Rare 23 Papers of 2014-2015 Not in 42 Files (Golden) - Chandkians-6th by Amlodipine Besylate

You might also like

Download as pdf or txt
Download as pdf or txt
You are on page 1of 257

FCPS PART 1

PAST PAPERS
PUBLISHED AT:

www.MedicosRepublic.com

KEEP VISITING OUR BLOG FOR MORE USEFUL STUFF ON FCPS,


USMLE, MRCP AND PLAB! ☺
BEST REGARDS,
DR. AURANGZAIB BALOCH
FOUNDER/ADMINISTRATOR OF MEDICOSREPUBLIC
1

Rare 23 Papers of 2014-2015 not in 42 Files(Golden) & Chandkians-6th


Compiled by : Amlodipine Besylate

(Index-Check list)Total=(2941)
=-= Year 2015-=-=-(1660 MCQS)
1. Surgery 10th June 2015 by Omair Arshad Dar(193) Page#1
2. Surgery 22 April 2015 by Dr.Irshad Baloch(177) Page#38
3. Surgery 1st Jan 2015 BY Surgeon Nash(60+) P#50
4. Medicine 10th June 2015 by Dr.Sumera Memom(75) P# 55
5. Gynae/Obs 31st Dec-2015 by Dr.Asifa (100+92)57
6. Gynae Obs Paper of June 10th, 2015 by Dr.Haya Khan(174)66
7. FCPS 4th Feb 2015 by Dr.BadalKhan / Nasar Yaqoob (187)P#80
8. Radiology 10 june 2015 by Ulnar Nerve(144)P#91
9. Radio June 2015 by Asma Warsi(95) p#99
10.Ophthalmology 31st Dec-2015 by Anita Malik(85)p#119
11.Psychiatry (conventional) November 2015 by Dr.Rabeea Iesar.(100)p#131
12.Psychiatry August 2015 by Dr.amjad/Dr Rabeea Iesar.(72)p#138
13.ENT 18th November 2015 by Zehra Aqeel Nizami(59) p#143
14. Psychiatry 16th August 2015 by Dr Amjad Dahani(47)p#148

=-=Year 2014-=-=-(1281 MCQS)

1. Medicine 12th Feb 2014 by Dr .Fahad (192) Page#152


2. Medicine August 2014 Online by Dr.Zara Bashir (148) page#178
3. Medicine 12th November 2014 by Amin Raza (197) page#195
4. Medicine November 2014 by Mitral Valve(195) page#207
5. Medicine Online 28th August,2014 by Dr.Sidra Majeed (27)page#225
6. Surgery 12th Febuarary 2014 by Dr. Asad-Ur-Rehman (200)page#227
7. Radiology 12th November 2014 by Sheeraz Ahmed Sheikh(159)#232
8. Pathology 4th June 2014 Paper by Umbreen Hashim (80) p#245
9. Ophthalmology June 2014 by Abdul Sami Abbasi(83) p#253
=-=-
th
Note :-Only following 4-Papers of 2014 available in Chandkians-6 Edition
th th
June 2014 medicine,Surgery 12 November 2014,12 Febuarary 2014 Gyane/Obs, Radio june2014
=-=-=-=-=--=-=-=-=-=-=-=-=-=-=-=--=-=-=-=-=-=-=-=-=-=-=--=-=-=-=-=-=-
1. Surgery 10th June 2015 by Omair Arshad Dar(193)
Surgery 10th June 2015 by Omair Arshad Dar(193)
1.A female pt with bleeding from breast.no
mass found on examination.no axillary
lymph node palpable.wats the diagnosis

A.Fat necrosis
B.Intraductal papilloma(Answer)
2

C.ductal CA
D.Fibrocystic disease
2.Regarding Adipose tissue

A.insulin cause lipolysis


B.inc in size by dividing adipocytes(Answer)

C.Growth hormone cause deoosition of fat


dont remember other options
3.Insulin causes

A.dec ketogenesis(Answer)
B.inc lipolysis
C.inc glycogenolysis
4.pnemothorax

A.lung collapse chest inward


B.lung collapse chest owtward(Answer)

5.Breast lymph drainage of inner lower


quadrant except (a long scenario actually)

A.Pectoral Nodes
B.inferior phrenic nodes(Answer)

C.internal mammary nodes


D.Apical Nodes
6.Pain Temp Crude touch fibres of face go
to thalamus through trigeminal via
A.Trigeminothalamic tract(Answer)

B.Trigeminothalamic lemniscus
C.Dorsal
D.Anterolateral tract
7.DOC for systemic fungal infection
3

A.Amphotericin B(Answer)

B.Ketoconazole

C.Iconazole
D.Nystatin
8.Decrease surfactant cause

A.increase pulmonary compliance


B.decrease pulmonary compliance(Answer)

C.decrease surface tension


9.Regarding Spleen
A. Vertically 12cm
B. On da right free margin of lesser sac
C. More echogenic than liver on ultrasound
D.if ligamentous support become lax it
becomes free called wandering spleen(Answer)

10.Thorn prick...abscess formation..most


likely organism

A.Strept Pyogenes
B.Staph aureus(Answer)

C.Pseudomonas aeruginosa
1.D? 2.B? 3.A

4.B 5.B 6.A?


7.A 8.B 9.D
10.B
11.15yr child came in peads clinic with
history of sore throat & fever for 10 days.
now he presented with chest pain not
related to respiration. what is the most
4

probable cause:

a. lungs

b. pleura
c. myocardium(Answer)

d. pericardium
e. costochondral joints
12.regarding middle colic artery which one
is correct,

A.supplies mailny left colic flexure,


B.is a branch of SMA, (Answer)

C.supplies cecum only


13.sickle cell scenario.boy having hb 5.6
deeply jaundiced, ,plt 260000 ndtlc 10000

a aplastic crisis

b painful crisis
c sequestration crisis
d hemolytic crisis(Answer)
14.Pregnant lady appropriate

A.inc ferritin
B.inc TIBC(Answer)

C.inc iron
15.Pregnant lady normocytic
normochromic anemia appropriate
A.plasma volume expansion(Answer)

B.iron def anemia


16.Aids associated skin lesion
A.Kaposi sarcoma(Answer)
5

B.BCC
17.Sertoli cells appropriate

A.secrete testosterone
B.secrete FSH
C.secrete LH
D. provide nourishment and glucose
E.Forms Blood Testis barrier(Answer)
18.Ca channels disease

A.Myasthenia Gravis
B.Lambert Eaton syndrome'(Answer)

C.Duchenne Muscular Dystrophy


D.Beckers myotrophy
dont remember exactly
19. Proteinuria RBCs RBC casts wats

damaged

A.Proximal tubule
B.Glomeruli(Answer)

C.Urinary Bladder
20.serum conc is 10/ml...urinary conc is
100/ml...urine flow rate is 2ml/min wats

clearance?

A.2ml/min
B.20ml/min(Answer)

C.200ml/min
D.0.2m/min
11.C 12.B 13.D?
14.B 15.A
6

16.A 17.D? 18.B?


19.B 20.B?
21.after total gastrectomy wat occurs

A.iron def anemia


B.Pernicious anemia(Answer)

22.best host defence system in alveoli

A.lymphatics
B.alveolar macrophages(Answer)
C.nasal cilia
23.breast surgery done pt cant comb hair
due to paralysis of serratus anterior.nerve
involved

A.suprascapular nerve
B.long thoracic nerve(Answer)
24.DVT most common
A.prolonged bed rest(Answer)

B.After surgery
C.inherited thrombophilia
25.14 yr old child with 8%creatinuria BP

180/110.. wat is damaged

A.glomeruli
B.JG cells(Answer)

C.Macula densa
26.after gynecological surgery which
anaerobic beta lactamase producing
organism causes infection
7

A.Bacteroides

B.Ecoli

C.Strept pyogenes
27.RMP is mainly by
A.k efflux(Answer)

B.na/k pump
28.blockage of alpha recepters(Answer)

A.vasoconstriction
B.vasodilation(Answer)
29.dec arterial PO2

A.CO poisoning
B.anemic hypoxia
C.hyoxic hypoxia(Answer)
30.Elastic fibers most thick in

A.Aorta(Answer)

B.renal
C.brachial
21.B 22.B 23.B
24.A 25.B
26.B? 27.A 28.B

29.C 30.A
31.4th Heart sound due to
A.Atrial contraction causes ventricular wall(Answer)

vibration
B.aortic rebound
C.verntricular filling
32.human is intermediate host in
8

A.Schistosomiasis
B.Haydatid cyst(Answer)

33.which doesnt cross BBB


A.levodopa
B.Dopamine(Answer)
34.2yr old female child pale n not thriving
dec hb 4.2 anisocytosis poikilocytosis
hepatospleenomegaly

A.leishmaniasis
B.thalasemia major(Answer)

C.fe def anemia


D.sideroblastic
35.aneurysm of 3rd part of subclavian
artery which triangle involved

A.Anterior
B.Posterior(Answer)

C.Muscular
36.week 2

A.epiblast
B.epiblast and mesoderm
C.Epiblast and hypoblast(Answer)
37.Regarding Thymus

A.Arch of aorta is anterior to it


B.has single lobe
C.extends from below thyroid to T4 vertebral
level(Answer)
38.Regarding NK cells
9

A.kill virus infected cells without prior


sensitization(Answer)

39.thyroid CA operated. 3cm mass which


has congo stain. wats best marker to
monitor
A.calcitonin(Answer)

B.AFP dont remember other options


40.knee cant b flexed and hip cant be

extended which muscle damaged


A.rectus femoris
B.sartorius
C.semitendinosus(Answer)

31.A 32.B 33.B


34.B 35.B?

36.B 37.B? 38.A


39.A? 40.C
41.Regarding Popliteus

A.medially rotates femur on tibia


B.Assists in knee extension
C.lies b/w lateral meniscus and lateral

collateral ligament
D.Cuases Flexion of Knee(Answer)
42.Meckels diverticulum
A.sometimes contain acid secreting mucosa
B.located in proximal part of ileum(Answer)
43.regarding cardiac cycle
A.Aortic notch his due to aortic valve(Answer)
10

B.60% Filling in diastole,


C.max filling in mid atrial contraction,

dont remember other options


44.tubuloglomerular feedback supported
by
A.dec peritubular na conc(Answer)

B.inc peritubular blood flow


C.inc tubular urine flow
45.paracetamol
A.has very weak antiinflammatory activity
46.a plumber has history of dyspnea and
cough now PH 7.4 bicarb 19 PCO2 31 PO2
62. wats diagnosis

A.compensated met acidosis


B.compensated resp alkalosis(Answer)

C.mixed resp alkalosis and met alkalosis


D.met acidosis
E.uncompensated resp alkalosis
47.case of small cell CA n cushingoid
features

A.ACTH(Answer)
48.pt has dec pupil size and droopy eyelid

A.home adie pupil]


B.occulomotor injury
C.argyl robertson pupil
D.horner syndrome(Answer)
49.pt rt eye cant look laterally
11

A.tumor in frontal lobe


B.cavernous sinus infection(Answer)

C.lateral blow to eye


50.elderly patient with pneumonia admitted
in ward...6 hrs later died
A.inc vascular permeability(Answer)

B.dehydration
C.shock

41.B? 42.A 43.A


44.A? 45.A
46.B? 47.A 48.D
49.B 50.A
51.urinary osmolality 1200 serum
osmolality 310 diagnosis

A.SIADH(Answer)
52.urethra damage at urogenital diaphragm
urine will leak into

A.Retropubic space
B.sup perineal pouch(Answer)

C.medial thigh

D.ischiorectal fossa
53.regarding Transitional epithelium

A.pseudostratified
B.Stratified columnar
C. Stratified with outer cells bigger n rounder(Answer)
54.during a histology practical test a
medicalstudent observes under
12

microscope a tubular structure having a


small lumen lined by transitional

epithelium supported by connective tissue


lamina propria and three layers of smooth
muscle with adventitia as outer most layer
what anatomical structure is it?

A.prostate
B.Renal pelvis

C.Urethra
D.ureter(Answer)

E.urinary bladder
55.Submandibular gland surgery nerve
damaged
A.marginal branch of mandibular branch of(Answer)

facial nerve
B.lingual nerve
56.70 age female with previous history of
MI hb5 TLC 7000 Plt 120000
pt 13 aptt 35 wats best for her

A.whole blood

B.FFP
C.platelets
D.RBC concentrate(Answer)
57.regarding Cerebellum
A.receive preprioceptive info from spinal cord
B.receive cortical info from cortex(Answer)

dont remember other options


13

58.infarct near post interventricular


septum.artery involed

A.LCA
B.RCA(Answer)

C.Rt marginal branch


D.LCX
E.Rt interventricular
59.regarding dilute urine

A.positive water clearance(Answer)


B.negative water clearance
dont remember exactly
60.floor of 4th ventricle's superolateral
relation
A.superior cerbellar peduncle(Answer)

B.middle cerebellar peduncle


C.inferior cerbellar peduncle
D.pyramidal tracts
51.A 52.A? 53.C
54.C 55.A
56.D 57.A 58.B

59.A 60.D?
61.regarding atrial fibrillation

A.p waves prominent


B.regular rhythm
C.pulsus deficit(Answer)
62.Most sensitive cardiac marker

A.creatine kinase
14

B.Troponin T
C.Myoglobin(Answer)

D.LDH
63.uterine leiomyoma. endometrium
around the mass has few glands and a lot
of stroma...wat has happened to
endometrium around fibroid

A.metaplasia
B.hyperplasia(Answer)
C.atrophy
D.dysplasia
64.alpha sympathetic stimulation

A.pupils constrict
B.pupils dilate(Answer)

65.parasympathetic stimulation

A.pupils dilate
B.pupils constrict(Answer)

C.inc heart rate


D.dec GI motility
66.Bile duct relation with duodenum

A.supraduodenal retroduodenal infraduodenal


intraduodenal(Answer)

B.omental supraduodenal intraduodenal


C.omental supraduodenal infraduodenal
67.Renal artery

A.interlobar lobar segmental arcuate


B.lobar interlobular segmental arcuate
15

C.seegmental interlobar arcuate


D.lobar segmental interlobar arcuate
68.Dorsum of noise supplied by
A.ethmoidal branch of ophthalmic artery(Answer)

B.a branch of facial artery


69.Free radicals formed by radiation they
will damage
A.DNA(Answer)

B.cell membrane
C.mitochondria
70.virus cause disease

A.direct action
B.altering protein synthesis
C.membrane damage

61.C 62.B 63.C?


64.B 65.B
66.A 67.A 68.A?
69.B 70.B
71.pt having of hepatoma developed
dependent edema and dilated abdominal

veins wats the cause

A.hepatic vein obstruction


B.IVC obstruction(Answer)

C.portal vein obstruction


72.female pt with difficulty
swallowing..wrinkles which appeared on
her skin vanished...difficulty eating with
16

hands...no joint pain..wats the diagnosis

A.dermatomyositis
B.CREST(Answer)
73.Regarding Medial geniculate body

A.receive visual afferents


B.attatched with thalamus(Looks correct)
C.located on ventromedial aspect f thalamus(Answer)
74.inc ejection fraction causes

A.dec EDV
B.Dec ESV(Answer)
75.Macrocytic anemia

A.Folic acid deficiency


B.intrinsic factor deficiency(Answer)

C.hemolysis
76.SA node location
A.superior aspect of sulcus terminalis(Answer)

B.lower part of crista terminalis


77.Regarding lignocaine

A.cause prolonged seizures


B.toxicity can be prevented by using 2%

preparation
C.can be prevented by hydrocortisone
injection
78.Femoral artery pulse is felt

A.apex of femoral triangle


B.midpoint of inguinal ligament
C.mid-inguinal point(Answer)
17

79.tumor suppressor gene


A.p53(Answer)

80.Diabetic nephropathy 1st manifestation

A.inc blood urea nitrogen levels


B.inc creatinine
C.inc urinary albumin(Answer)

71.B 72.B 73.B?


74.B 75.A

76.A 77.A? 78.B?


79.A 80.C
81.Mesothelioma most common cause

A.tobacco smoking
B.asbestosis(Answer)
82.Gastric lymphoma most common cause

A.H-pylori(Answer)

B.atrophic gastritis
83.Host defence against neoplasm
A.apoptosis(Answer)

B.good health
C.no family history
84.most common cause of neoplasm
A.overexpression of proto-oncogenes(Answer)

B.non-lethal injuryof genes


85.IVC is formed at
A.L5 vertebra(Answer)
86.female with exophthalmos inc HR
sweating antibodies present
18

A.Hashimoto
B.Graves(Answer)

87.Regarding Down syndrome


A.inc incidence with inc maternal age(Answer)
88.Dorsal rami supply

A.Flexors of trunk
B.extensors of trunk(Answer)
89.female undergoing hysterectomy..there

is a chance of damaging ureter while


ligating which artery

A.Internal ilac artery


B.Uterine artery(Answer)
90.pt underwent trauma now has spastic
lower limbs exaggerated lower limb

reflexes Babinski sign is positive biceps


jerk is absent whr is the injury
A.C5-C6(Answer)

B.C2-C3
CL1-L2
81.B 82.A 83.A?

84.B 85.A
86.B 87.A 88.B
89.B 90.A
91.Prostate arterial supply
A.Inferior vesical artery(Answer)

B.superior vesical artery


C.internal ilac artery
19

92.localized edema cause


A.allergy(Answer)

93.Regarding Red cells

A.larger than all WBCs


B.Mean MCV is 85 fl(Answer)

C.contain remnants of mitochondria and


endoplasmic reticulum
94.myeloperoxidase is present in

A.lymphocytes
B.RBCs
C.Neutrophils(Answer)
95.Succinylcholine toxicity in individuals
having deficiency of

A.Acetyl cholinesterase
B.pseudocholinesteae(Answer)

C.MAO
96.upper origin of external oblique forms
digitation with which muscle

A.Latissimus Dorsi
B.Pectorals major
C.Serratus anterior(Answer)
97.Platelet rich plasma is stored at

A. 0 deg
B. 4 deg
C. 22 deg(Answer)

D.40 deg
98.Lady had breast implant 2 yrs ago since
20

then she is having firmness and shape


distortion something . Now the implants

are removed and implant fluid is leaking


inside. Which cells will most likely be
present

A. Neutro
B. Mast
C.Eosinophil
D.Giant cell(Answer)
E.Plasma cell
99.Remnant of gubernaculum in females

A.spermatic cord
B.Testis
C.Round ligament of ovary(Answer)

D.prostate
100.vessel having 80% saturation in fetus
A.umbilical vein(Answer)

B.aorta
91.A 92.A 93.B?
94.C 95.B

96.C 97.C? 98.D


99.C 100.A
101.Sciatic nerve is damaged now which
nerve supplies sole of foot
A.saphenous nerve(Answer)

B.sural nerve
C.common peroneal nerve
21

102.trauma to upper limb pt cant feel


sensation in little finger and paralysis of

dorsal interossei which nerve is damaged


A.Ulnar nerve(Answer)

B.anterior interosseus nerve


C.median nerve
103.Fracture of surgical neck of humerus.
nerve damaged

A.median nerve
B.axillary nerve(Answer)

C.upper brachial plexus


104.trauma to shoulder axillary nerve
damaged pt cant abduct arm

A.above head

B.0 to 20 deg
C.20-90 deg(Answer)
105.which muscle is flexor of elbow joint
and supplied by radial nerve
A.brachioradialis(Answer)

B.brachialis

C.Biceps brachii
106.which hormone causes extrahepatic
protein catabolism and hepatic protein
anabolism

A.Thyroid hormone
B.Epinephrine
C.Cortisol(Answer)
22

D.growth hormone
E.insulin
107.GFR is increased by

A.afferent artiolar constriction


B.dec plasma proteins(Answer)
108.lumen of allantois doesn't obliterate

A. Urachal sinus
B.urachal cyst

C.umbilical vein
D.urachal fistula(Answer)
109.abdominal contents pushed through
remains of processus vaginalis which
occurs
A.indirect inguinal hernia(Answer)

B.direct inguinal hernia


C.hydrocoele
110.left adrenal vein is drained into

A.IVC
B.left renal vein(Answer)

C.Inf mesenteric vein

101.B ? 102.A ?
103.B 104.B
105.A
106.C 107.B
108.D 109. A
110.B
111.Tissue not regenerated
23

A.kidney
B.adrenal medulla
C.lens(Answer)

D.skin
112.regarding tail of pancreas
A.in linorenal ligament(Answer)

don't remember the mcq exacty


113.no lymphoid nodules seen in

A.thymus(Answer)
B.Lymph node
C.spleen
114.CA urninary bladder caused by which
parasite
A.schistosoma hematobium(Answer)

B.clonorchis sinenis
115.pseudomembranous colitis caused by
which

A.clostridium perfringens
B.clostridium dificile(Answer)
116a.most common cause of gas gangrene

lower limb

A.clostridium birefrengens (this is the exact


spelling as in paper)
B.clostridium welchii(Answer)

C.clostridium septicum
I asked the examiner he said it must be
perfringens
24

116b.osteoblastoma most common site

A.diaphysis
B.Metaphysis(Answer)

C.epiphysia plate
D.epiphysis
E.Body of vertebra
117..difference between primary and
secondary peristaltic wave

A....Primary has oropharangeal phase(Answer)


B.primary wave relaxes lower esophageal
sphincter
C.secondary propels food forward
don't remember exact mcq
118.joint b/w pubic bones allowing slight

movement during birth of baby

A.Synchondrosis
B.syndesmosis
C.Symphysis(Answer)
119.Heart contractility depend on
A.ca++ ions(Answer)

120.Regarding thyroid gland


A.change in epithelium occurs with function(Answer)

111.C 112.A 113.A


114.A 115.B
116a.B? 116b.A? 117.A?
118.C 119.A 120.A
121.fast adapting phasic receptors
25

A.Pacinian(Answer)

B.meisners

C.Baroreceptors
122.rapidly adapting receptors
A.pacinian(Answer)

B.Baroreceptors
C.free nerve endings
2 similar questions were asked one in both

papers a and b
123.which of the following inhibits the
muscle to stretch beyond limits

A.golgi tendon organ


B.muscle spindle(Answer)
124.Dead space does not change in

A.standing
B.old age
C.shallow breathing(Answer)

D.Deep inspiration
E.Tracheostomy
125.chlorthiazides cause

A.hyperuricemia(Answer)

B.hyperkalemia
C.hypercalcemia
126.max triglycerides found in

A.VLDL
B.chylomicrons(Answer)

C.LDL
26

D.HDL
E.IDL
127.most common source of SYSTEMIC
arterial emboli

A.femoral vein thrombosis


B.pulmonary vein thrombosis
C.right atrial thrombus
D.portal vein thrombus
E.left ventricular mural thrombus(Answer)
128. pyruvate is intermediade between
A.glucose and acetyl coA(Answer)

B.acetoacetic acid and actyl coa


C.Fat metabolism and acetyl coA
D.carbohydrates and proteins
129.pt with back pain lytic lesions in
vertebra and ribs on xray.. most
appropriate investigation for diagnosis

A.MRI
B.Immune electrophoresis(Answer)

C.CT scan
130.patient with history of multiple
myeloma now has hypercalcemia , polyuria
and confusion urine osmolality is
310mosm/l
wats the reason of polyuria

A.ADH cant act on tubules


B.dec ADH formation
27

C.psychogenic polydipsia
don't remember the actual stems

121.A 122.A 123.B


124.D 125.C
126.B 127.E
128.A? 129.B
130.A?
131.pulmonary atery supply

A.Bronchi
B.Bronchioles
C.Alveoli(Answer)
132.Regarding bronchopulmonary
segment

A.airated by primary bronchus

B.airated by secondary bronchus


C.airated by tertiary bronchus(Answer)

D.not in touch with all other segments


133.55 yr old diabetic hypertensive
hypothyroid DOC for HTN

A.Propanolol

B.Verapamil
C.Captopril(Answer)

D.Nefidipine
136.A 45 yr old female presents with two
years history of pruritis, months history of
jaundice. On clinical examination she has
bruises xanthomas and
28

hepatosplenomegaly. The investigation


which will confirm the diagnosis would

be:

A. ALT
B. Alkaline phosphatase
C. Anti-mitochondrial antibodies(Answer)

D. CT abdomen
E. ultrasound abdomen.
137.sarcoidosis histological picture
A.Ganuloma with asteroid bodies(Answer)

B.non-caseating granuloma
138.ADH formed in

A.posterior pituitary
B.supraoptic nucleus(Answer)

C.ventromedial nucleus
139.42 yr old female and dyspareunia and
midcycle pain n bleeding her cervix was
hypertrophic.. vagina and uterus normal
which initial investigation should be done

A.colposcopy with biopsy


B.cervical smear(Answer)

C.endoscopy
140.metaplasia is caused by
A.chronic irritation(Answer)

131.C 132.C 133.C


136.C 137.A?
138.B 139.B 140.A
29

(plz dont mind numbering error ☺)


141.most common cause of PRIMARY HCC

A.hep C
B.alpha 1 antitrypsin deficiency
C.Aflatoxins
d.hep b more than hep c(Answer)
142.Two liters Normal saline infusion

A.dec urinary sodium

B.inc blood volume


C.dec intracellular volume
143. patient bp 90/50.. why is isoflurane
contraindicated
A.dec cardiac contractility(Answer)

B.vasodilation of blood vessels


144.Regarding popliteus

A.assist in knee extension


B.cause medial rotaion of femur on tibia
C.located in between lateral meniscus and
lateral collateral ligament(Answer)

D.supplied by obturator nerve


145.Otic ganglion is located immediately
below
A.Foramen ovale(Answer)

B.Foramen rotundum
C.Foramen lacerum
146.DIC initiated by
A.thromboplastin(Answer)
30

B.Factor 13
147.Tachyphylaxis

A.Rapid dec in drug action(Answer)

B.Gradual dec in drug action


148.Fastest speed of impulse in

A.ventricular fibers
B.AV node
C.AV bundle
D.purkinje fibers(Answer)
149.a student asked a pt to clench his
teeth then followed his masseter inferiorly
on lower madibular marginal he felt a
pulsation which artery was it

A.Lingual
B.Facial(Answer)

C.Maxillary
D.External carotid
150.pt cant open mouth which muscle
injured

A.Masseter

B.medial pteryogoid
C.lateral pterygoid(Answer)

D.Buccinator
E.Platysma
141.A? 142.B 143.B?
144.A 145.A
146.A 147.A 148.D
31

149.B 150.C
151.Abdominal migraine cause

A.IMA
B.SMA(Answer)
152.Middle colic artery is a branch of
A.SMA(Answer)

B.IMA
C.Aorta
153.There are no blood agglutinins in
which blood group

A. ABB.
AB+(Answer)

C O+
154.Insulin increased by

A.Glucagon(Answer)

B.Growth hormone
155.pt has Gonococcus infection what is
quicked way to check it

A.ZN stain
B.Gram stain(Answer)

C.Culture
156.pt took cotrimxoazole developed black
urine reason?
A.G6PD(Answer)

B.Malaria
157.Whcih prolongs the life of corpus
luteum
32

A.testosterone
B.endogenous FSH LH
C.Endogenous HCG(Answer)
158.Which artery has branches that are
end arteries

A.Coronary artery
B.Central artery of Retina(Answer)
159.pt has Christmas disease...factor 9 not

available wats best substitute


A.Cryoprecipitate
B.FFP(Answer)

C.Whole blood
160.Metaplasia
A.functional change in cells(Answer)

B.change in size and shape of cells


151.B 152.A 153.B
154.A 155.B
156.A 157.C
158.A? 159.B
160.B
161.pseudomonas causes shock
A.endotoxin(Answer)
162.which are bipolar neurons

A.posterior root ganglion cells


B.anterior horn cells
C.posterior horn cells(Answer)

D.Olfactory cells
33

163.memory function of which lobe?

A.parietal
B.Temporal(Answer)

C.Frontal
D.Occipital
E.insula
164a.GH increased by
A.excercise(Answer)

B.hyperglycemia
164b.Pellagra caused by def of

A.Vit B1
B.Vit.B6
C.nicotinic acid(Answer)
165.which is caused by DNA virus

A.influenza
B.infectious mononucleosis(Answer)
166.cervical lymphadenopathy...reed stern
berg cells seen
A.hodgkin lymphoma(Answer)
167.pt with normal internal genital organs

with ambiguous genitalia karyotype 46 XX


A.Adrenogenital syndrome(Answer)

B.Craniopharyngioma
168.Area 3,1,2
A.All somatic sensation(Answer)
169.Basal ganglia excitatory pathway
neurotransmitter released
34

A.serotonin
B.Dopamine(Answer)

170.Pregnant lady with rheumatic heart


disease ...most dangerous complication
for her

A.MR
B.AR
161.A 162.D 163.B

164a.A 164b.C 165.B


166.A 167.A 168.A
169.B 170.B
171.Which hormone causes smooth
muscle constriction

A. Aldosterone
B.ADH(Answer)
172.Ethics principle

A. Autonomy Beneficence. Non-maleficence


Equity
173.Regarding mean

A.it is some of individual value divided by total


number of values(Answer)

B.its a good measure of central tendency


174.Brocas Area

A.Sup temporal lobe


B.afluent aphasia(Answer)
175.infraorbital artery is a branch of
A.maxillary(Answer)
35

B.facial
C.ophthalmic
176.painful swelling at site of amputation
A.neuroma(Answer)

B.ganglioneuroma
177.Sulfur containing amino acid

A.Arginine
B.Cysteine(Answer)

178.pt on oral anticoagulation which test


for monitoring
A.PT(Answer)

B.APTT
179.pyogenic meningitis
A.dec glucose(Answer)

180.Axillary sheath from

A.pretracheal sheath
B.prevertebral fascia(Answer)

C.carotid sheath
171.B 172.A 173.A
174.B 175.A

176.A 177.B 178.A


179.A 180.B
181.Anterior duodenal perforation whr
does the fluid accumulate

A.lesser sac
B.left paracolic gutter
C.rt subphrenic space(this is best if answer below not given)
36

D.left subphrenic space


there was no option of right paracolic gutter
(Answer) is right post subphrenic in supine and right paracolic gutter in standing
182.hormone responsible for oxidative
phosphorylation and inc in body
temperature
A.Thyroid hormone(Answer)

B.Growth hormone

C.Insulin
183.Pt had severe headache...CSF showed
blood tinge

A.sub-dural hemorrhage
B.sub-arachnoid haemorrhage(Answer)
184.young pt had diarrhea PT deranged

platelets normal hb normal


A.Vit k deficiency(Answer)
185.Edema is caused by

A.dec hydrostatic pressure


B.inc osmotic pressure
C.blocked lymphatics(Answer)

186.lower esophagogastric junction and


fundus removed wat will happen
A. loss of receptive relaxation(Answer)

B.inc gastrin production


C.inc acid production
187.Regarding Trachea
A.from upper border of C6-T4(Answer)
37

B.Recurrent laryngeal nerve in front


dont remember other options
188.pt with rt sided weekness of facial
muscles cant blow air wats the reason of
weakness
A.facial nerve palsy(Answer)
189. Regarding Vibrio cholera caused
diarrhea

A.inc chloride channels(Answer)


B.causes dec absorption of food which leads
to osmotic diarrhea
190.first response for bleeding

A.wound contraction
B,Vasocconstriction(Answer)

181.E 182.A 183.B


184.A 185.C
186.A? 187.B? 188.A
189.A 190.A
191.potent chemotactic factor

A.C3a
B.C5a(Answer)
192.Disease transmitted by mosquito
A.Filariasis(Answer)
193.Left coronary artery divides into
A.LCX and anterior interventricular artery(Answer)

=-=-=-=-=-=-=-=-=-=-=-=-
38

2. Surgery 22 April 2015 by Dr.Irshad Baloch(177)

Q1. sephaneous n terminal br of


A.femoral ANS b.obturator
c.sciatic
Q2. adductor muscle hamstring muscle supplied by which nerve
A . tibial ANS B.abturator
C.femaoral D.common peroneal
Q3. gracillis nerve supplied by ,
A.obturator nerve ANS b. femoral
c. common peroneal
Q4. adductor brevis supplied by,,
A. ant br of obturator nerve ANS
b.femoral c.common peroneal
Q5. fracture of surgical neck of humers, artery damaged,,
a.axillary nerve B.post.circumflex humeral ANS
Q6. common bile duct parts,,
a. omental, retroduodenal, paraduodenal
B. omental, retroduodenal, paraduodenal, intaduodenal
c.retroduodenal, paraduodenal, intraduodenal
Q8.loss of small muscle of one hand damage,,
A. C8,T1 ANS b.ulnar
c.radial d. C5 C6
Q9. inion landmark for what ,,
a.glabella b.pterion
C.extrnal occipital prominence ANS d.internal occiptal groove
Q10. gastroduodenal artery branch of,,
A. right hepatic artery ANS b.celiac c.splenic
Q11. regarding tramadol true is ,,
a.kappa nd meu b.strong kapa c.weak kappa
d.strong meu E.weak meu agonist
Q12.ketorolac analgesia most preferable,,
a.Thoractosy patient with ozzing blood from wound
b. hernoraphy wid patient of peptic ulcer
C.Non diabetic cholecyctectomy ANS
d.Asthmatic patient having inguinal hernia repair
Q13.Radiotherapy most sensitive to,,
a.craniopharyngioma b.pineal tumor
c.midbrain glioma D.glial glioma ANS [confirm it ]
Q14.bilateral exopthlmos, antithyroid antibody presen,,
A.graves ANS b.hashimotos
Q15.epitheial changes in endocervix in squamous tissue due to ,,
A. metaplasia ANS b.dysplasia
Q16.normal rasonance area in,,
A. right 2nd to 4th midclvicular line ANS
b. right 4 to 6 midclavicular
c.left 2nd to 4 mid clavicular d.left 2 nd to 4 midaxillary
Q17.internal carotid a damage, bleeding, at what vertebral tubercle u will press to stop
bleeding,,
a.3rd b.4th
39

c.5th D.6th ANS e.7th


Q17.human intermediate host ,,
A. hydatid cyst ANS
Q18.cysticercosis caused by,,
A. ingesting eggs of tenia solium ANS b.ingesting cysts of tenia solium
c. eating uncooked meat d. eating uncooked mutton
Q19. which drug has more effect on coxII than cox I ,,
a.aspirin B.celexocib ANS
c. nalproxane d.ketorolac
Q20. Internal oblique muscle aponeurosis,,
a.form conjoint tendon b. post boundary of inguinal canal
c. external spermatic fasia D.conjoint tendon ANS [confirm it]
Q21. RCA after ginving marginal branch obstruction affect ,,
a.SA node B. AV node ANS
c. apex of heart d.right atrium
Q22. penile urethra rupture ,,
A. scortum under colles fascia ANS b.tunica vahinilis
c.thigh d.deep fascia
Q23.which one is benign tumor,,
A. warthin ANS
Q24.middle rectal artery branch of, ,,
A.internal iliac ANS b.femoral
c.common iliac d. superior mesentric
Q25.right testis tumor nodes drain into ,
A. paraaortic ANS b. righ deep inguinal
c.superficial inguinal
Q26.Chlamydial infection causing chlamydia psittacosis is,,
A.obligate intracellular ANS b.DNA derived from host
Q27.scenerio of granuloma wid yellow pigment ,,
A. actinomyecets ANS
Q28. painful papule at labia ,,,
a. hsv II A.hsv I ANS [ confirm it by urself]
c. vz d. CMV
Q29.most perinium and pelvis nerve supplied by,,
A.pudendal ANS b.femoral c.obturator
Q30.maintain flexor tone arm by ,,
a.corticopinl b.corticobulbr
c.reticulo spinal D.rubro spinal ANS
Q31.lower motor neuron lesion...
A.muscle wasting ANS b.hyperreflexia
c.positive babinski d. spasticity
Q32.hemisetion with total spinal section differentaiad by ,,
a.paraplegia in extension b. degre of reflex activity
c.sensory loss permanat D.spinal shock ANS
Q33.which one is most common exocrine pancreas tumor ,,
A.ductal adenocarcinoma ANS b.acinar adenocarcinoma
c.mixed d.intraductal
Q34.delayed action of insulin,,,
a.glucose inside cell b.potasium inside cell
c.inhibit protein catabolism d. protein synthesis
E. stimulate mRNA transcription for lipogenesis [confirm it by urslf ]
40

Q35.gas having mximum diffusion capacity in body fluids,,,


A. CO2 ANS b.CO
c.He d.O2 e. N
Q36.dec PO2 in blood..
a.CO poisoning B.hypoventiltion ANS
c.anemia d.cyanide poisong e.hypoventilation
Q37.A person living on river side since 2yrs, having blindness, opthalmologist indentify which
larva,,,
A.onchcerosis ANS b. wucheria c. loa
Q38.during surgery thyroid nerve present to a distance to superior thyroid artery,
A.externl lyryngeal ANS b.internal laryngeal c.recurrent
Q39.antrum of stomach removed wht happen ,
a.dec compliance b.inc acid output c.increase gastrin
Q40.margins of TB ulcer,
A.undermined ANS b.everted c.inverted
Q41. ulcer due to hypertension,,
a.decibuts b.marjolins
C. martorell ANS d.bazin
Q42. PTU:
A. Inhibits synthesis of thyroid hormones b.lt homonymous hemianopia
B. Inhibits release of thyroid hormones
Q43. lt rt optic tract lesion causes,,
a.right homonymous hemianopia b.left homonymous hemianopia
Q44.Internal laryngeal nerve passes through,,

a.Arytenoid n cricoids cartilage b.Cricoid n thyroid

C.Thyroid n hyoid ANS

Q45. prgnanacy related senerio, PT and APTT inc (too long scenario but answer DIC :
A.DIC ANS
Q46.whichr used for screening congenital abnormality,,
A.Estrol ans [confirm it] b.estrione c.estradiol
Q47.phenyl over alcohol anaesthesia ,,
a. effect detected soon B.less painful ANS
c.phenol most potent
Q48.patient developed hemolysis due toG6PdD deificiency nxt happen,,
A.self limiting ANS b.hemolysis with G6PD c.autosomal domint
Q49. Primitive streak mesoderm cells migrate around the cloacal membrane to form ant.
Abdominal wal cell, if ths migration does not hapen around the cloacal membrane will
ruptureand open caudly as.
A. exstrophy of bladder ANS b.imperfrate anus
c.hypospedia d.ectopic anal opning
Q50.liver development ,
a.from dorsal mesentry proximal midgut
B.from ventral mesentry proximal foregut ANS
c. from ventral mesentry distal foregut d. ventral mesentry proximal midgut
Q51.which one drain into left brachiocephalic vein,,,
a. sup thyroid b.middle thyoid
C.inf thyroid ANS d. facial
41

Q52.64 year Old man with posterior cervical lympadenopathy which is firm and mobile. Hb 13-
14 hct 40 wbc 7230 plateletes 250000 no hepato splenomegaly seen biopay showed numerous
small monomophic lyphocytes,,

a.infectious mononucleosis b. hodgkin lymphcyte predominant ANS


c.CLL d.reactive hyper plasia
Q53.hb 10, O2 sturation 90, PO2 60 , ask abt to calculate O2 contents,,
a.10 B.13 ANS c.18 d.21
Q54.girl with h/o of gum bleeding, giant platelets , plt count 85k, ,,,
A.vonwillibrand ANS b.bernaud solier
c.familial thrombocytopathy d.thrombocytopenia e.ITP
Q55.thiazide and loop diuritc given work simultaneously in excretion of Na and chloride from,,
a.ascending loop of henele b.proximal tubult
C.distal tubule ANS d.descending limb
Q56. required for transport of B12 in blood,
a.intrinsic factor b.TC 1
C. TC2 ANS d.TC 3
Q57.terminal ileum removed what happens,,
A.large amount of water in feces ANS b. decreased amino acid absorption
c.increased bile acid and entero hepatic circulation d.increased fat absorption
Q58.protein content less in glomerular filtrate occur due to ,
A.pore size and negative charge ANS
b.rapid active transport absorbtion
Q59. increase in aldosterone due to inc in,
A. HIGH K ANS b.angiotensin 2
Q60.35 year old man bilateral crepts, on xray chest heart fills completely,mass in rt ventricl ,
what wil b the main cause..
a.mesothelioma b.rhabdomyosarcoma
c.angiosarcoma d.myxoma
Q61.about flexor hallucis longus,,
a.tendion can be felt immediately post to medial maleolus
b.inserted on first metarsal base
Q62.superficial parotid painless tumor, psudopoid growth, contain no capsule on histopatho ,,
A.pleomorphic ANS b.warthin
c.mucoepidermoid d.cyst adeno carcimona
Q63.Hypertensive ulcer
a.bazin B.Martorel ANS
c.curling d.decubitus

Q64.Reflexes correctly marched,,


A.Bicep c6 ANS b.triceps C8
c.abdominal L5 S1 d.knee L4 L5
Q65.Rt ovarian vein drains into,,

A. IVC ANS b.renal vein

Q66.regarding posterior communicating artery,,


a.connect post cerebral to internal carotid
b.connect post and middle cerebral
c. pass anterior to occulomotor and connect internal carotid with post cerebral
D.Its coonects internal carotid with posterior cerebral ANS
42

e.connect post cerbellar inernal carotid

Q67.ADH is inhibited by,


A.alcohol ANS b.increased plasma osmolarity
c.angiotensin 2 d.dec ECF
Q68.large Bolus stuck after eating what will happen next,

a. Its never happens b.swallowing centre activated

C.secondary persiatlisis will be generayed by intrinsic nervous sytem to remove it ANS

Q69.whn will u do dilaysis whn gfr decreases by %,,

A. 5 ANS b.20 c.30

d.40 e.50

Q70.feature of malignancy,,
a.Invasion B.metastasis ANS [confirm it]
c. high N C ratio
Q71.Death of Gas gangrene is,,,

A.Toxemic shock ANS b.excessive necrosis

Q72. big scenario of ischemia but in end asking finding in brain ,

A. Liquefactive ANS b.coagulative


c)caseous
Q73.infection related to pretracheal layer will spread to,,
A.Ant mediastinum ANS b.sup mediastinum
c.post medistinum
Q74. Regarding active transport about drugs which is incorrect,,

a.saturable b.require energy

C. all drugs tranpoted by this mechanism d.involves cariers


Q75. organism causing cerebral malaria,,
A. p. falciparum b.p. vivax
c.p. malaria
Q76.p.falciparum disease,,
A. black water fever
Q77.regarding piturtry gland true is
A.vein drain directly in dural sinuses b.partly derived from foregut
c.related to ethmoidal sinus d.optic chiasm lie anteriorly
Q78. Patient with fluid loss now show metabolic acidosis e hypokalemia. most fluid loss will be
through:
A. Stomach
B. Colon
C. Pancreas
D. Duodenum
E. Jujenum
43

Q79. Inversion of foot by which muscle:


A. Peronues Longus
B. Peroneus Brevis
C. Peroneus tertius
D. Tibialis Posterior

Q80. Anemia wid Blast Cells in peripheral blood:


A. Iron deficiency anemias
B. Erythroblastosis fetalis
C. Folate deficiency
D. Bone marrow depression

Q81. Most common cause of Hepatocellular CA in developing countries:


A. HEP B, C
B. Hep C
c. Hep B

Q82. Closure of lips involve: or unable to close lips which muscle involved:
a. Depressor anguli oris
b. Depressor labii inferioris
c. Depressor labi superioris
d. Orbicularis oris
e. Lateral pterygoid

Q83. Superior Thyroid artery arise from:


A. First branch of Subclavian Artery
B. External Carotid Artery
C. Internal Carotid Artery

84. Incubation period of which disease is longest:


A. Measles
B. Rubella
C. Chicken pox
D. Infectious Mononucleosis
E. Mumps

Q85. Regarding CSF true is,,


A. total 150 ml
B. production rate 20 ml/hr
C.produced by ependymal cells of choroid plexus
D. Arachnoid villi are visible
Q86. Age estimation at 5th to 12th week is done through:
A. Biparietal diameter
B. Crown rump length
C. Crown heel length
D. Abdominal circumference
87. A sprinter during running had injury to his ankle. He can stand on his toes but with severe
pain. There is a visible ecchymoses around ankle joint:
44

A. Plantaris tendon

Q88. Amoeba causes lesions in which part of gut:


A. Terminal ileum
B. Cecum
C. Ascending colon
D. Transverse colon
E. Sigmoid and rectum

Q89. Buccopharyngeal Membrane:


A. Stomodium from Foregut
B. Nose from Pharynx

Q90. Pretracehal fascia infections spreads to:


A. Anterior mediastinum
B. Posterior mediastinum

Q91. Shivering center is located in:


A. Anterior hypothalamus
B. Posterior hypothalamus

Q92. A 30 year old woman in azad kashmir was injured mildly in an earthquake that killed many
people apart from her distant uncle. She would most probably be in,
A. Social crisis
B. Personal crisis
C. Situational crisis
D. Developmental crisis

Q93. While a surgery on Submandibular gland which has more chances of injury:
A. Superior thyroid artery
B. Lingual nerve
C Mandibular nerve
D. Facial nerve

Q94. Which ligament supports odontoid process of axis on atlas,


A. Posterior longitudinal ligament
B. Anterior longitudinal ligament
C. Tranverse ligament
D. Ligamentum nuchae

Q95. C7 Vertebrae has:


A. Longest spine
B. Broad body

Q96. conversion of fibronogen into fibrin what will b detected in blood

A. Prothrombin

Q97. Percentage of Plasma out of ECF:


A. 30%
B. 20%
45

C. 50%

Q98. Newborn with a Erythroblastosis Fetalis having Blood group B postive what is best for
management:
A. Any blood with Rhpostive
B. B Negative

Q99. If a person is taking Protein in his diet which will be necessary:


A. Riboflavin
B. Thiamine

Q100. Group of young men bathing n beech next day develop blisters on back shoulder limbs
region cause is:
A. Mast cell destruction
B. Endothelium venules destruction

Q101.Renal Column:
A. Collecting Duct
B. Interlobar artery
C. Interlobular artery
D. Minor calyces

Q102.Function exclusively linked with vit E:


A. Carbohydrate metabolism
B. Endothelial protection
C. Skin integrity
D. Germinal epithelial protection
Q103.Deep to post digastric and near palatoglossus a structure runs obliquely upwards,
A. Facial artery
B. Lingual artery
C. Maxillary artery
D. Sup thyroid
Q104.Nucleus of general visceral efferents of tenth nerve to palate muscles is present in:
A. Locus ceruleus
B. Tractus solitarius
C. Nucleus ambiguous [ confirm it by urself]
D. Nucleus raphe
Q105.Regarding pneumotoxic center following is true:
A. Increases inspiratory potential
B. Fires dynamic respiratory drive

Q106. What is correct about thyroid:


A. Supplied by 3 paired arteries
B. Surrounded by prevertebral fascia
C. Isthmus attached to cricoid
D. Venous drainage in bracheocephalic vein

Q107. Regarding Pituitary gland true is :


A. TSH
B.TRH
46

Q108. Blood group A+ transfusions reaction due to:


A. A+
B. B+
C. O-
D. A-

Q109. H2O2 found in:


a.mitochondira
B.peroxisome ANS [confirm]
c. Lysosome
D. RER

Q110. ICAM and VCAM are for:


A. Leucocyte adhesion

Q111. Breast atrophy in young female is due to:


A. Estrogen
B. Estrogen and Progesterone
C. Progesterone

Q112. in man causes cancer is:


A. Protooncogen

Q113. Essential amino acid:


A. Phenylalanine

Q. Joining of ICA and Basilar artery:


A. Posterior communicating Artery

Q114. Regarding sympathetic effects Beta Agonists:


A. Increase HR

Q115. Poliomyelitis:
A. Anterior horn cell defect

Q116. Regarding P Value:


A. Checks probability
B. Percentile something.

Q117. Spread of renal tumor will be through:


A. Renal Vein

Q118. Retention of C02:


A. Respiratory failure

Q119. Antiseptic for skin: not come


A. Phenol with clorhexidine
Cant recall other statements but this seemed the best answer as Phenol and Clorhexidine are
used for skin disinfectants.
47

Q120. Cardiac disease scenario wad there and they asked about the fluid accumulation,
proteins more than 3.2 g/dl:
A. Exudative
B. Transudative

Q121. Scenario was given about massive splenomegaly wid lymphatic obstruction:
B.Leishmanias
B. Chornic Malaria

Q122. regarding Gray matter foot processes:


A. Protoplasmic Astrocytes

Q125. Cause of global blindness:


A. Herpes Simplex
B. Rubella
C. Chlamydia Trochamatis

Q126. Which is normal HB:


A. HbA
B. HbF
C. HbC

Q127. Thalesemia carrier percentage in Pakistan:


A. 5%
B. 10%
C. 15%

Q128. Surgery for removal of ovary done which is damaged:


A. Internal Iliac Artery
B. External Iliac Artery
C. Uterine Artery

Q129. In respiratory Alkalosis renal compensation is:


A. NaHC03 excretion

Q130. During mitosis chromosomes double in which phase?


A. Prophase
B. Metaphase
C. Anaphase
D. Telophase
E. Interphase

131. Example of observational study:


a. Cohort
b.cases (include case reports )
c.mata anylasis

Q132. most two common pathway of axillary lymph drainage:


A. central and apical
B. Supraclavicular and posterior
48

C. Pectoral and lateral


D. anterior and posterior

Q133.true about csf is


A.produced by chroid plexus which is cauliflower n shap coverd by smooth muscle and seen by
naked eye
B.daily production is 500 ml per day
C.hyperosmolar thn plasma

Q134.the maximum strengh of wound healing occurs in


A. 3 month ANS
Q135. suboccipital n supply which muscle
a.capitis b. lattisimus dorsi
c. trapezius D.rectus capitis post ANS
Q136. regarding intercostobrchial nerve true is ,,
a. loss of nerve supply b.post side of arm
C. 2nd intercostal sensory ANS [confirm it]
d.loss to preganglionic sympethaetic
Q137.Normal rasonance are,,
A. rt 2nd to 4th midclvicular line ANS b.rt 4th to 6th midclavicular
Q138. in asian population most common cause of macrocytosis
A. liver diease ANS [confirm it] b.hemolysis
c.hypothyrodism d. alchol
Q139.corticoteriod mechanism anti inflammatory
A. inhibit phospolipase A2 b.inhibit cycloxygenase
c.inhibit lipoxygenae d) action on leukotriene B e. inhibit lukotriene D
Q140. scenerio about sacrioilac joint pain, loss of lumber ladorosis negative ana,, 22
A. HLA b27 ANS b.rA
Q141.a person standing at same place for 1hour, what wl dec
a.diametr of aorta B.central venous pressure ANS
c.rennin d.adh
Q142.true regarding breast peau de orange appearance except,,
A.orange brown appearance due to necrosis
b.dimpling of skin due to lymphatic obsturstion c. subcutenous edema
Q143.enzymes for abcess formation
A.neutrophil ANS b.serum c.lymphocyte
Q144.A lady presented with with leg swelling for last five months and no cough nd fever, on
investigation thre were bilateral plueral effusion and right atrium enlargemt...ast 200 ,alt250 ldh
dearranged
a.recurrent thromboembolism b.rheomatid arthritis
C.good posture ANS [confirm it] d.renovascular hypertension
e.chronic asthma
Q145.Estrogen, progesterone and testosterone k receptors act through.. ligand chanel
involves,,
a.g coupled b.tyrosine kinase
C.increase transcription ANS [confirm it]

Q146. great tronchetric fracture, cant walk properly painful swelling, next complication
A.pulm thromboembolism ANS b.soft tissue tumor
Q147.post triangle relations
Q148.acute appendicitis scenerio, on pathologicl emanination
49

A.neutrophil n muscular wall ANS


b. lymphoid hyperplasia with giant cell in muscular wall
Q149. old man develepod discomfort in abdomen with enlargement massive
Splenomegaly(3000 gm),,,
a.sickle cell b.Mononucleosis
c.portal Hypertension D.myelofibrosis ANS

Q150.During exercise decreased TPR is due to


a.Skeletal muscle arterioles dilation due local metabolites
b.due to increase coronary flow to brain
c. increase splanchnic flow
Q151.man in hot sunny environment fell...finding na 122 due to
A. excessive sweating ANS b.decrease water intake
c.decrese reabsorption
Q152. lady presented with persistent asthenuria ( dilute urine) defect,,
a.high blood urea
B. low urine nacl ANS [confirm it]
c. fixed specific gravity or plasma osmolarity
Q153. about pancrease true is ,
a. they are inferior to stomach
b. sup mesenteric pass anterior to uncinate process
Q154. leukocyte adhesion factor is ,,A. integrin ,b. selectin
Q155. 35 year old man bilat crepts, on xray chest heart fills completely,a mss in rt ventricl
a.mesothelioma b.rhabdomyosarcoma
c.angiosarcoma D.myxoma ANS
Q156. if P53 is mutated wht wil happen A. cell survival ANS
Q157.2ndary cartilagenous joint is A .Pubic symphysis ANS
Q158.Oesophegotomy done, nw stomach blood supply is
a.lt gastric artery B. rt gastric artery ANS [confirm it]
Q159.regarding acetylcholine true is
Q160.regarding acetylcholine true is
Q161.protein synthesis correct squence is
Q162.jejunum blood supply is
A. SM artery ANS b.im artery
Q163.non epithelil origion A.sarcoma
Q164.one question abt bone
Q165.dicumarol side effects -- A. Increase BT ANS
Q166.regarding breast tumor correct is
Q167.pt and dr relation ,, -A.reassurance ANS
Q168.increase amino acid,glucose and fatty acid n blood
A.Cortsol ANS
Q169.in children spinal cord level is,, A. L3 ANS
Q170.hip extension and leg flexion by ,, A.semitandinius
Q171. gun shot at lateral leg wid foot drop ,site of injury is ,,A.neck of fibula ANS
Q172.long thoracic nerve supply to,,, A. serratus anterior ANS
Q173. transplant 5 years back ,maximum chances of what,,
Q174.female wid nausea,icterus ,hep n pregnancy ,which hep wil b dangerous n future a.HEP E
Q175. regarding c1 vertebrea true is,, A. no body ANS
Q176.prolactan antagonist,, A. dopamine ANS
Q177. in malnutrition pt ,wht substance u wil check to measure estimate malnutrition
A. Protein ANS b.vitmins ,c.mineral
50

=-=-=-=-=--=-=-=-=-=-=-=-=-=-=-=--=-=-=-=-=-=-=-=-=-=-=--=-=-=-=-=-=-
3. Surgery 1st Jan 2015 BY Surgeon Nash(60+)
1.A 31 year old young man has heaviness in his scrotum for six months. On examination a solid 5cm mass is
palpated. On labs his alpha feto protein level was 81ng/ml while bHCG level was 15 IU/l. Same side orchiectomy
was done and smooth glistening mass was seen. Which of following could be the cause
1. Leydig cell tumor
2. Embryonal carcinoma ans
3. Choriocarcinoma
4 Teratoma
2.Ligament between dens of axis and process of atlas
Ant. Longitudinal
Post. Longitudinal
Transverse
Laigament flavum
3.Most common position of appendix
1. Paracecal
2. Retrocecal ans
3. Pelvic appendix

4.Regarding elbow joint


Capsule deficient posteriorly
2. Covered all arpund by muscles
3. Hinge joint ans
4. Supination occurs here
5. Joimt between humerus, radius, ulna

5.Lymph drainage of lower medial quadrant of breast will NOT drain into
1. Pectoral nodes ans
2. Medial sternal nodes
3. Supraclavicular nodes
4. Inf phrenic modes
5. Apical nodes

6.Appendicular artery is branch of


Post cecal
Iliocolic ans

7.most suitable nsaid in lactation


, brufen ans
, ketorolac,
sulindac

8.Which tumor involves extension to nerve sheath


1. Myxomas
2. Adenomas
3. Hemangiomas
4. Carcinoma expleomorphic adenoma
9.what is correct about thyroid gland 1)supplied by 3 paired arteries 2) surrounded by prevertebral fascia 3)
isthmus attached to cricoid 4) veous drainage in bracheocephalic vein ans 5) forgot the last option
10.A needle when inserted 2cm below and lateral to pubic tubercle will be on
1. Obturator nerve ans
2. Ilioinguinal nerve
51

11.A 52 year old man underwent coronary artery bypass graft two weeks ago and now presents with purpuric
patches on skin. On labs his PT and aPTT were normal. Following is true
Heparin induced thrombocytopenia
2. Drug induced thrombocytopenia ans
12.Regarding pneumotoxic center following is true
Increases inspiratory potential
2. Fires dynamic respiratory drive ans
13.About ankle joint
1. Supplied by superficial artery
2. Supplied by superficial paroneal nerve
3. Fibrocartilage on hialine surfaces
4. lateral malaeolar ligament on medial side
5 deltoid is in maleolar side ans
14.Which one combination is correct
1. Thiopental reduces heart rate
2. Profofol increases cerebral blood flow
3. Morphine reduces apnoeic threshold ans
4. Ketamine relaxes bronchial smooth muscles
15.Medial geniculate body
1. Receives oculomotor fibers
2. Optic tract fibers
3. Fibers ascend to thalamocortical areas ans
4. Solely to thalamis
5. Progect to frontal lobe
16.Loss of abduction and adduction of fingers and adduction of thumb with intact skin sensations involves
1. Recurrent branch of median
2. Deep branch of ulnar ans
17.Heterophile antibody is seen in
1. Infectious mononucleosis ans
2. Herpes simplex
3. Gonorrhea
18.Lesser omentum
Encloses hepatic artery and hepatic duct
2. Does not attach to liver
3. Attaches to second part of duodenum
4. Is a bilayer ans
19.Sensory supply of uterus passes through which ligament
1. Broad ligament ans
2. Round ligament
3. Cardinal ligament
4. Uterosacral ligament
5. Uterocervical ligament
20.Amoeba causes lesions in which part of gut
1. Terminal ileum and cecum ans
2. Ascending colon
3. Transverse colon
4. Descending colon and sigmoid
5. Sigmoid and rectum
21.Deep to post digastric and near palatoglossus a structure runs obliquely upwards
1. Facial artery
2. Lingual artery ans
3. Maxillary artery
4. Sup thyroid
22. Nucleus of general visceral efferents of tenth nerve to palate muscles is present in
52

1. Locus ceruleus
2. Tractus solitarius
3. Nucleus ambiguous ans
4. Nucleus raphe
23.Function exclusively linked with vit E
1. Carbohydrate metabolism
2. Endothelial protection
3. Skin integrity
4. Germinal epithelial protection ans
24.While a surgery on submandibular gland which has more chances of injury
Superior thyroid artery
2. Lingual nerve
3 Mandibular nerve
4. Facial nerve (marginal branch) ans
25.Which ligament supports odontoid process of axis on atlas
1. Posterior longitudinal ligament
2. Anterior longitudinal ligament
3. Tranverse ligament ans
4. Ligamentum nuchae
26.Cardiax myxomas are most common in
1. Left atrium ans
2. Right ventricle
3. Infundibulum of pulmonary artery
27.Otic ganglion is between mandibular nerve and tensor tympani. It's superior border is formed by
1. Foramen lacerum
2. Foramen ovale ans
3. Foramen rotundum
4. Styloid process
28.Inion is
1. Part of pterion
2. Posteroinferior prominent part of occipital bone ans
3. Posterior to frontal bone
4. Internally to occipital bone
29.Closure of lips involve
1. Depressor anguli oris
2. Depressor labii inferioris
3. Depressor labi superioris
4. Orbicularis oris ans
5. Lateral pterygoid
30.A sprinter during running had injury to his ankle. He can stand on his toes but with severe pain.. There is a
visible ecchymoses around ankle joint
Torn tendoculcaneus
2. Torn plantaris tendon ans
31.Thrombosis of right coronary atery after right marginal artery has branched off will result in infarction of
SA node
2. AV bundle
3. Apex of heart
4. Infundibulum of pulmonary artery
5. AV node ans
32.Age estimation at fifth to 12th week is done through
Biparietal diameter
2. Crown rump length ans
3. Crown heel length
4. Abdominal circumference
53

33.MELAS syndrome
Loflar
Inclusion
Granuloma
Cyst
red ragged borders
34.Highest number of alpha resceptors are present in
1. Trigone
2. Neck of bladder and proximal urethra ans
3. Penile urethra
4. Membranous urethra
5. External urethral opening
35.Loss of inversion
1. Paroneus longus
2. Paroneus brevis
3. Flexor digitorum
4. Tibialis posterior ans
36.A 31 year old young man has heaviness in his scrotum for six months. On examination a solid 5cm mass is
palpated. On labs his alpha feto protein level was 81ng/ml while bHCG level was 15 IU/l. Same side orchiectomy
was done and smooth glistening mass was seen. Which of following could be the cause
1. Leydig cell tumor
2. Embryonal carcinoma ans
3. Choriocarcinoma
4 Teratoma
37.A 30 year old woman in azad kashmir was injured mildly in an earthquake that killed many people apart from
her distant uncle. She would most probably be in
1. Social crisis
2. Personal crisis
3. Situational crisis ans
4. Developmental crisis
38.Anterior epithelium of cornea is
1. Stratified squamous keratinized
2. Simple squamous
3. Simple columnar
4. Stratified squamous non keratinized ans
5. Simple cuboidal
39.Incubation period of which disease is longest
1. Measles
2. Rubella
3. Chicken pox
4. Infectious mononucleosis ans
5. Mumps
40.Clavipectoral fascia
A. Part of endothoracic fascia reflecting anteriorly
B. Encloses pectoralis major
C. Encloses pectoralis minor ans
D. Extends superiorly and encloses scalenous anterior
E. Option i forgot
41.18 year old girl has fever tenderness in right ileac fossa. On labs WBC 17000cells/mm3. Appendicectomy was
preformed. Which of the following would be seen on histology
1. T lymphocytes in wall of appendix
2. Polymorphonuclear neutrophills ans
3. Glissening mass at apex of appendix
42.Which of following has highest chances of developing endometrial cancer
54

1. Simple hyperplasia
2. Complex hyperplasia without atypia
3. Complex hyperplasia with atypia ans
4. Squamous metaplasia
5. Chronic endometritis
43.CSF
1. Formed by ependymal cells
2. Formed 500ml/day ans
3. Formed only 30percent by chorioid plexus
4. Arachnoid villi are visible
44.A young boy with inc appetite weight gain and sleep problems. On examination he is having small genitals
1. Hypothyroidism
2. Pineal tumor
3. Craniopharyngioma
4. Narcolepsy
5. Cushing disease

45.Chronic Fungal infection with rhinosinusitis causing medial erosion and granulomas
1. Cryptococcosis
2. Mucormycosis
3. Rhinophycomycosis
4. Histoplasmosis
5. Blastomycosis
6,aspergillus (ans

Simpler bcqs about


bladder cancer schistosoma
Cholangioca clinorchis
Pubic symohysis secondary cartilaginous joint
Malignancy metastasis
Exudate more than 3gm protein
Itp labs
Brachial plexus multiple twisted questions and same for heart blood supply
VWF scenario

1.pretracheal fascia infection spread to...ant mediastinum


2.pituatry tumour cause... hemianopia of bilateral nasal half of visual field defect
3.cardiac infection e protein > 3.2 g/ DL...exudative pattern
4.protein diet required... ascorbic acid
5.n resp alkalosis renal compansation...decrease hco3 reabsorption
6.glucose transport...carrier mediated
7.50 yr old e lt hoarsense...mediastnal lymph node e Lt recurrent laryngeal involvmnt
8.daily production of csf... 500 mg/dl
9.group of young men bathing n beech next day develppe blebs on back shoulder region cause is...mast
cell destruction
10.abt meningitis...
11.for nutritional status measurement ...albumin
12.renal column...a)arcuate artey b)minor calcyes c)collecting tubule d) interlobar n interlobular art
13.pt e fluid loss now show metabolic acidosis e hypokalemia...a)stomach b) colon c)panrease d)
duodenum
14.1st cervical process...odontoid process
15.retention of CO2...resp failure

=-=---=-=-=-=-=-=-=-==-=-=-=-
55

4. Medicine 10th June 2015 by Dr.Sumera Memom(75)


10 JUNE 15 MEDICINE AND ALLIED MIXED PAPER 1 N 2
1. location of SA node:
2. prick thorn injury: staph aureus
3. cells which are bipolar: olfactory cells
4. lobe involved in memory: temporal
5. parasympathetic stimulation: meiosis
6. fastest conduction in: purkinje fibres
7. RMP due to: Na/K pump
8. gas gangrene causing organism: clostridium welchii
9. cardiac contractility: Ca++ ions
10. superlateral wall of fourth ventricle: inf cerebellar peduncle
11. cerebellum: receives fibres from motor cortex
12. end artery: central artery of retina
13. most sensitive marker of MI: myoglobin
14. severe angina pain due to: aortic stenosis
15. heart sound S4 due to: ventricular vibrations due to atrial systole
16. axillary sheath extension of: pretracheal fascia
17. infraorbital artery is the branch of: maxillary artery
18. muscle openin the jaw: med pterygoid muscle
19. pyruvate is the intrmediate in: glucose and acetylcoA
20. sulphur containing amino acid: cystine
21. thyroid hormone in: uncoupling n oxidative phosphorylation
22. which inc the insulin secretion: glucagon
23. leiomyoma related ques: neoplasia
24. tumor suppressor gene: p53
25. viruses cause damage by: altering protien synthesis
26. watr diuresis: +ve free water clearance
27. gap junctions: low resistance
28. mean: adding all the values and divide them with total no of values
29. ques regarding transitional epithelium
30. NaCl: inc blood vol
31. four basic pillars or ethics: autonomy,confidentiality something like that
32. enzyme myeloperoxidase oresent in: neutrophills
33. neoplasia due to: inc epressions of proto oncogenes
34. scenario on Vit k def
35. common cause of HCC: hep C
36. GFR inc when: dec plasma proteins
37. tachphylaxis:
38. local cause of edema: allergy
39. cause of edema scenario: lymphatic obstruction
40. cause of systemic emboli: mural thrombi fom left ventricle
41. pulmonary aretery supplies: alveoli
42. blood group having no agglutibins: AB+ve
43. alveoli defence: alveolar macrophages
44. does not crosses BBB: dopamin
45. diabetic nephropathy diagnosed by: urine albumin
46. Regarding transitional epithelium: Stratified with outer cells bigger n rounder
56

47. organ with no lymhoid nodules: thymus


48. most potent chemotactic agent: C5a
49. alpha receptors: mydriasis

50. scenario on primary biliary cirrhosis: antimitochondrial antibodies


51. which does not regenerate: lens
52. pallegra is due to def of: nicotinic acid (vit b3)
53. ADH released from: supraoptic nucleus
54. what results from gastrectomy: pernicious anemia
55. in the beginning of 2nd week of development: epiblast n hypoblast
56. thyroid follicle: changes epithelium acc to follicular activity
57. metaplasia: functional change in cells
58. pap smear scenario: neoplasia
59. left adrenal vein drain into: lef renal vein
60. rapidly adapting: pacinian copuscles
61. joint b/w pubic bones: symphysis
62. high CO failure: beri beri
63. investigation about gonococcus: gram staining
64. allantois: urachal cyst
65. 80% oxygen saturation in: umbilical vein
66. lp done and bloody CSF in: subarachnoid hemorrhage
67. dorsal post ramus supplies: extensors of trunk
68. best test to differentiate obs n restrictive lung diseases: FEV1/FVC ratio
69. dead space remain unchanged in: standing/ deep inspiration
70. protective against neoplasia: atoptosis
71. max water nd salt reabsorption in: jejunum
72. highest triglycerides in: chylomicrons remnants
73. bronchopulmonary segment supplies: tertiary bronchus
74. treatment of mild cong heart failure: ACE inhibitor-captopril
75. hookworm doesnot cause: macrocytic anemia
thanks remember me in ur prayers
DR. SUMERA MEMON
=-=-=-=-=-=--=-=-=-=-=-=--=-=-==-
57

5. Gynae/Obs 31st Dec-2015 by Dr.Asifa (100+92)


1:atrophy of breast in adult female
(Estrogen)
2:transpyloric plane
(Lower border of L1)
3:body of hyoid
(C3)
4:area receiving inhibitory signals from cerebral cortex but nt sending output back
(Substantia nigra)
5:1st sign of puberty in female
(Thelarche)
6:5th week of gestation,rubella infection causing
(Cataract)
7:boy came with fracture,on examination blue sclera found
(Collagen)
8:inscision at mc burney
(Deep circumflex illiac)
9:cause of edema in renal failure
(Albuminuria+na retention)
10:if factor 9 not available,alternate is
(Fresh frozen plasma)
11:pregnant women died in RTA
(Fortitious death)
12:pain in right shoulder gall blader pathology referred to
(C3,c4,c5)
13:which contain posterior cutaneous branch(c1,c6,c7,T1)
14:coccygeal ligament upper limit from
(s1,L1,coccyx)
15:side effect of biguanide
(Diarrhea n flatulence)
16:full term pregnant women with intra uterine death,DIC
(Evacuate uterus)
17:after delivery women in pph
(Ergometrine)
18:most definitive diagnosis of TB
(AFB)
19:dead space decreased in
(Tracheostomy)
20:fisherman with echymosis,gum hypertrophy deficiency of
(Vitamin C)
21:most common cause of hospital acquired infection
(Staphylococcus aureus)
22:child with git disturbance,macrocytic anemia best diagnosis
(Serum vitamin B12,intrinsic factr antibodies)
23:inferior layer of urogenital diaphragm
(Perineal membrane)
24:urethral sphincter is content of
(Deep perineal pouch)
25:trigone of bladder in female lies in
(A:upper surface of urogenital diaphragm
b:middle of urogenital diaphragm
C:below the urogenital diaphragm)
58

26:amniotic infection can be diagnosed


( IL-6)
27:true about kidney
(Renal vein pass anterior to aorta & renal artery)
28:sigmoid colon
(Starts at pelvic brim i marked)
B:supplied by left colic atery)
29:thyroid
(Isthmus lies over 2,3,4 tracheal rings)
30:thyroid gland
(Drains into deep cervical lymph node)
31:uretric stone on left side near pelvic brim
(Sigmoid mesocolon)
32:anus lymph drainage
(Medial group of superficial inguinal lymph node)
33:vaginal hymen histology
(Stratified squamous epithelium)
34:normal man with well developed secondary sexual characteristics,normal external genitalia,semen production
normal but with azoospermia.
(Sertoli cells)
35:prolactin inhibited by
(Prolactin inhibited factor)
36:after hypophysectomy effects
(Principal cells of thyroid)
36.physiological responce of GH
(A:direct effect on chondrogenesis
B:inc amino acids uptake in cell
C:uptake of glucose in tissues)
37:tension of muscles
(Golgi tendon organ)
38:length of muscle(muscle spindle)
39:normal cycle of respiration
(2 sec inspiration,3 sec expiration)
40:women with repeated abortions
(Lupus anticoagulant)
41:old man with type 2 diabetes,diabetic foot with ulcer,sensation loss kch esa tha
(Angiopathy+ neuropathy)
42:volatile anaesthetic with ca chanel blocker what effect on patient
(Hypotension i marked
B.hypertensn
C.bradycardia)
43:which autonomic substanc responsible for prolactin inhibition
(Dopamine)
44:abundant igG
(IgG)
45:scenario of achondroplasia
(Autosomal dominant)
46:a patient died on 6th day of MI,autopsy,clots in pericardial sac(cardiac temponade)
47:corticospinal tract while descend decussate at
(Pyramids)
48:gluconeogenesis by
(Glucagon)
49:stress hormone
59

(Cortisol)
50:female diagnosed withbreast carcinoma axillary lymph node palpable,which quadrant involved most likely
(Upper outer quadrant)
51:phrenic nerve anterior to which muscle
(Scalene anterior)
52:NMDA receptor
(Glutamate,GABA,serotonin)
53:mg is abundant in
(Bones)
56:post 1/3 of A/V septum
(Rt coronary artery)
57:processus vaginalis
(Testis
B:process of parietal peritoneum
C:covers ductus deferens)
58:old lady with breast ca,osteoporosis
(Tamoxifen)
59:old man died with stroke,autopsy brain shows
(Liquefactive necrosis)
60:binding of two or more molecule together
(Synthetase)
61:common cause of infertility in man
(Klinefilters syndrome)
62:dynamic support of uterus
(Pelvic diaphragm)
63:women pregnancy test +ve,pain in right illiac fossa(ectopic pregnancy)
64:zona glomerulosa secretes
(Aldosterone)
65:in ICU iatrogenic infections can be prevented by
(Hand washing)
66:role of prostaglandin in cervical ripening
Collagen hydration)
67:length of right main bronchus
(2.5cm)
68:thirst increased by
(Inc osmolarity+dec plasma volume)
69:cough with bilateral crepitations,normal floura on gram stain,recovered in 2 weeks with no sequelae
(Influenza type A)
70:carbidopa potentiates effect of levodopa
(dec peripheral degradation)
71:regarding hyaline cartilage
(Present in nose septum
B:in epiglottis
C:cartilagineous modification)
72:tubular organ lined by stratified squamous epithelium,elastic
(Vagina)
73:testis lymph drainage
(Para-aortic lymph node)
74:pain in back radiates to medial maleolus
(S1)
75:oxy-hb dissociation curve left shift
(CO poisoning)
76:regarding epithelium(simple squamous presnt in areas ov gaseous exchange)
60

77:pregnant full term,on way to hospital 50miles away,suddenly died(amniotic fluid embolism)
78:essential fatty acid
(Linoleic acid)
79:thyroid hormone
(Increases free fatty acids)
80:urogenital diaphragm attached laterally to
(Ischio ramus+pubic ramus)
81:to prevent tetatus in population what should be given
(Tetanus toxoid)
82:renin secretion can be decreased by
(Angiotensin 2)
83:sympathetic discharge in circulatory failure acts through
(Vasomotor centr)
84:in anaemia turbulence of blood in circulation due to
(Dec viscosity of blood)
85:subacute endocarditis caused by
(Streptococcus viridans)
86:pus n abscess contains
(Dead neutrophills)
87:epitheloid cell origin
(Monocytes)
88:christmax disease
(Def of factor 9)
89:cavernous sinus is related to which cranial nerve
(Abducent nerve)
90:an old patient came with lesion of pre central gyrus of left side
(Right hemiplegia)
91:increase consumption of carbohydrates will lead to deficiency of
(Vitamin B1-thiamine)
92:if two groups in which one group having vomiting other is not,wht yu called to present this data
(Nominal
B:parametric-i marked
C:ordinal
D:ratio
E:rate)
93:parasympathetic
(Relaxes intestinal sphincter)
94:proto oncogene converted to oncogene by
(Amplification of genes by viruses)
95:alpha receptor present in
(Iris )
96:metaplasia is referred as
(Change in character of epithelium)
97:major blood buffer
(HCO3-bicarbonate)
98:in pregnancy women feel respiratory distress due to inc in progestrone
(Dec PCO2
B:inc in tidal volume 20-30%
C:inc in minute volume 30-40%)
99:post streptoccal glomerunephritis
(Sub epithelial deposits)
100:pregnancy related edema is due to
(Dec serum albumin)
61

Remember me in yur prayers�


Good luck for future gynaecologists

2ND Post
Gynae n obs 31st dec,2015
Remaining questions paper 1n2 mixed
DR.Asifa Hussain

3 ques were repeated in both question with a change in statemnt


1:toxoid are used to prevent which of the following infections
(Tetanus)
2:pregnant lady came on 28 week of gestation on 1st antenatal visit with no any complain,normal
examination,lab shows low hb with few abnormal cells,what yu further advise
(Hb electrophoresis
B:cbc
C:mcv,mch,mchc
D:serum ferritin/iron)
3:in which condition 25 hydroxyD is not converted to 1,25 dihydroxy D
(CRF)
4:long scenario-cant recall full statemnt,ovaries cant ascend to abdomen due to
(Uterus)
5:a women with normal menstrual cycle of 32 days most likely to ovulate
(18th day)
6:surgicaly devided segments of lungs with tertiary brochi called
(Broncho pulmonary segmnt)
7:which is benign tumor
(Warthin)
8:epithelial origin of carcinoma most likely will contain
(Keratin)
9:autosomal dominant
(Variable expression)
10:glucocorticoids causes
(Lymphocytopenia)
11:co-relation coefficient r
(+o.5
B:_0.5
C:0.0
D:+1
E:_1(my engineer sis said it should b _1� ,confirm it urself)
12:hypophsis pituitary related posterior superior to
(Sphenoid sinus)
13:during excercise dec blood supply to
(A:brain
B:heart
C:kidney
62

D:skin)
14:pointed injury on figure cause redness imediate effect is(axonal reflex)
15:question regarding pleomorphism
16:pregnant lady came with 103f,fever,chills and dark urine
(Falciparam malaria)
17:regarding fetal heart
(Septum primum+septum secundam join n convert common atria to right n left atrium)
18:tubular structure,intestinal epithelium with lymphoid tissue in submucosa
(Appendix)
19:different % given,51% asian,20% european,etc hw to present this data
(Pie chart)
20:pregnant lady died on RTA
(Fortitious death)
21:whom yu called a healthy person
(A:a person who is married lives with children happily
B:a person who had never get ill ever in life
C:a person who is socially active,occupationally active,hv no any chronic disease,free of norms
of life(i marked)
D:a person who is dominant in society,every person praising of him)
22:indirect bilirubin increase in
(Methyldopa..i marked
B:clonidine
C:hydrochlorothiazide)
23:which of the following hav adenyl cyclase activity
(A:norepinephrine
B:clonidine
C:dopamine)
24:full term pregnant lady came with high grade fever,chills,watery discharge, most likely
(Chorioamnionitis
B:UTI..i marked
C.dengue
D:falciparam malaria
E:ectopic pregnancy)
25:action of drug on body
(Pharmacodynamix)
26:gonadotropin effect
(Inc LH n FSH)
27:premalignant lesion of skin
(Compound nevus.i marked
B:seborrheic keratosis
C:spider telengiectasia)
28:graft rejection
(HLA mismatch)
29:drug tolerance
(A:inc with inc in dose
B:inc with dividing doses
63

C:)hate dis question


30:patient came in ER with hypoxia,PH 7.36,pco2 30,HCO3 16
(A:compensated metabolic acidosis
B:compensated respiratory acidosis)
31:primordial germ cells derived from
(Endoderm)
32:inc in 2 3 BPG
( inc tempt
B:glycolysis)
33:patient with heart,kidney trabsplant got infection shows halos on microscopy most likely
(Cryptococcus)
34:acute radiation effect
(Desquamation of skin)
35:aggresive carcinoma
(Melanoma)
36:child came with fever n jaundice
(aLT+ bilirubin)
37:scenario of microcytic anaemia
38:pregnant lady came with c/o prominent veins on body n skin lesions,same she noticed with
her uncle having hepatitis C
(Palmer erythema+spider nevi)
39:a long scenario,lady with ryt loin pain,on examination normal,ct shiws adrenal mass 3cm,hw
to confirm it is malignant
(Invasion)
40:diffusion of solute depends upon
(Surface area)
41:during 5th week of developmnt
(Heart has definitive form
B:limb buds appear
C:stomach rotates)
42:actinomycosis (abscess n granuloma)
43:which can be palpated through lateral vaginal wall
(Ureter)
44:gamma glibulin produced by
(Plasma cells)
45:partirution caused by(fetal cortisol)
46:sensitive test for MI
(Trop T)
47:hernia protrude by hassel bach triangle
(Direct inguinal hernia)
48:inc lipid solubility
(Rapid action)
49:which vitamin help in clotting
(Vitamin c..vit k was not in option)
50:GFR inc by(dilatation of afferent arteriole)
51:maternal blood separated from fetal
64

(Syncytotrophoblst+endothelium
B:cytotrophoblast+endothelium other options i dnt remember)
52:bladder lymph drainage
53:common cause of death in primigravida in our country
(Sepsis i marked
B:haemorrage
C:amniotic embolism
D:prematurity)
54:45 yr old lady c/o weight gain,T3,T4,tSH diff values were given in normal range
(A:weight gain i marked
b:hypothyroidism)
56:G6PD (bitten cells)
57:cells donot regenerat(lens)
58:primigravida with obstructed labor which is mostly damage
(Levator ani i marked
B:sphincter urethrae
C:urogenital diaphragm)
59:gastric lymphoma most commonly cause
(H.pylori)
60:patellar reflex
(Contraction of quadratus femoris)
61:Mgso4 given in eclampsia act through
(GAba i marked
B:glutamate
C:serotonin)
62:halothane given in anaesthesia with
(Nitrous oxide)
63:parasympathetic bcq dnt remembr options as there were 3 bcq on parasympathetic stimulation
64:bcq on lysosomes
65:mast cell produces
(Heparin,proteases)
66:universal recipient(AB +ve)
67:q on hamartoma
68:2nd pharyngeal arch derivative
(Stylohyoid ligament)
69:melanocyte
(Neural crest cells)
70:musxle relaxation
(GT0)
71:severe diarrhea
(Inc k+,inc Na+,dec Na)
72:pregnant lady with hb 9.8gm/dl, with normocytic picture
(Physiologic change)
73:right surface of heart(costosternal)formed by
(Right ventricle)
74:XXX has how many bar body
65

(2 bar body)
75:Hb 5gm/dl ,MCV 107
(Vit B12 deficiency)
76:diagnosis of TB
(AFB)
77:female with hb dec,MCV dec with 105 ferritin
(Thalasemia trait
B:microcytic anaemia)
78:breast lobules separated by
(Interlobar stroma)
79:local anaesthetic action is by
(By blocking Na+ channels)
80:glucose cross placenta by
(Facilitated diffusion)
81:adenoma
(Glandular origin)
82:uterus prolapse complication
(Metaplasia+ulceration
B:cervical dysplasia+discharge)i marked

83:ca feature (invasion)


84:inguinal canal superior border formed by..roof
(Conjoint tendon)
85:in venous blood (inc HCO3)
86:maximum blood in
(Veins)
87:cardiogenic shock most likely due to
(Haemorrhage)
88:red infarct
(Organ with dual blood supply)
89:in shock characteristic feature by
(Tissue hypoxia)
90:toxin produced by streptococcus pneumonia
(Streptolysin)
91:pulse pressure
(Inc in stroke volume)
92:dialysing fluid feature
(More glucose than plasma)
8 bcq em unable to recawl
Forgive me for human errors.plz remember me in prayers.dua for my result.Allah will succeed
yu awl.GOOD luck

=-=-=-=-=-=-=-=-=-=-=-=-=-=-=-=-=-=-=-=-=-=-=-=-=-
66

6. Gynae Obs Paper of June 10th, 2015 by Dr.Haya Khan(174)


1
This document is the property of FB group: Gynae Obs Past Papers FCPS-1
Gynae Obs Paper of June 10th, 2015
(Compiled and Edited by Dr.Haya Khan)
1. Ureteric bud not reaching mesenchyme leads to
a) Kidney without ureter pancake kidney
b) Kidney without ureter
2. Longest pre erthrocyte stage is longer in
a) Plasmodium Falciparum
b) Plasmodium Ovale
c) Plasmodium Vivax
d) Plasmodium Malariae
3. No agglutinin is present in blood group
a) AB+
b) AB –
c) O+
d) O-
4. Excretory system of kidney is derived from
a) Ectoderm
b) Endoderm
c) Lateral plate mesoderm
d) Intermdiate mesoderm
e) Splanhnic mesoderm
5. Platelets transfusion does not transfuse
a) DIC
b) Previous multiple blood transfusions
6. 4th heart sound is produced by
a) Vibration of the AV valves
b) Filling of the ventricles
2
This document is the property of FB group: Gynae Obs Past Papers FCPS-1
7. Cloacal membrane
a) Arises from the caudal end of hindgut
b) Derivative of Endodermal plus ectodermal
8. Dead space remains unchanged in
a) Shallow breathing
b) Deep breathing
c) Standing
d) Old age
e) Tracheostomy
9. Patient is unable to open the mouth due to paralysis of
a) Medial pterygoid
b) Lateral pterygoid
10. What is correct about broncho pulmonary segment
a) Supplied by primary bronchi
b) Secondary bronchi
c) Tertiaary bronchi
67

d) Not directly related to each other


e) Each bronchopulmonary seg is supplied by a separate veinof its own
11. Characteristic of actinomyces
a) Abcess
b) Granuloma
c) Cancer
d) Abscess and granuloma
12. One of them is an end artery
a) Central retinal artery
b) Coronary artery
13. A carcinoma of vagina below hymen( vaginal orifice) is likely spread via the lymphatics into
the
a) Inferior mesentric nodes
b) Internal and external iliac nodes
3
This document is the property of FB group: Gynae Obs Past Papers FCPS-1
c) Internal iliac nodes
d) Medial group of horizantal superficial inguinal nodes
e) Vertical group of superfial inguinal nodes
14. A scenario on pernicious anemia – Vit B12 deficiency after resection of the colon
15. Iron deficiency scenario in which MCV, MCH and MCHC were low.
16. A scenario of thalassemia in which the patient had moderate splenomegaly
17. Human placental lactogen
a) Single chain having 191 amino acids
b) Molecular wt 22 kg
18. Tachyphylaxis
a) Diminish unresponsiveness adapted quickly
b) Diminish unresponsiveness adapted slowly
c) Caused by morphine
19. Harmone that causes synthesis of protein in liver while its lysis in peripheral tissue
a) Cortisol
b) Insulin
c) Glycagen
d) Thyroid
20. Powerful vasoconstrictor
a) Bupivacaine
b) Cocaine
c) Tetracaine
21. Ureter is narrowest at
a) Ischial spine
b) Ischial tuberosity
c) Iliosacral joint
d) Sacral promontory
4
This document is the property of FB group: Gynae Obs Past Papers FCPS-1
e) Perinatal mortality
22. Values were given we had to solve it using the formula for maternal mortality
23. Harmone which causes increase in cAMP
68

a) GH
b) LH
c) ADH
24. Muscles are
a) Origin is mobile
b) Fixed at insertion
c) Ligaments are fibrous
25. Intra embryonic obliteration of urachus
a) Urachal cyst
b) Urachul fistula
c) Urachal sinus
26. True about rectum is
a) Straight structure
27. True about pudendal nerve
a) Passes through great sciatic notch
b) Supply internal anal sphinter
c) Supplies muscles of perineum
28. GnRh is not inhibited by
a) Carcinoma
b) Estrogen
c) Progesterone
29. Maximum pressure dissipated is
a) Aortic arterial
b) Arterio arterial
5
This document is the property of FB group: Gynae Obs Past Papers FCPS-1
c) Aertrio capillary
30. Cerebellum
a) Coordinates all movements
b) Recieves afferents only from cerebral cortex
c) Recieves sensory afferents
d) Proprioception
31. Rapidly adapting
a) Pacinian corpuscles
b) Arterial barorecpetors
c) Messiener corpuscles
32. 4th ventricle floor lateral border is formed by
a) Pons and medulla
b) Inferior cerebellar peduncle
c) lateral cerebellar peduncle
d) Medial cerebellar peduncle
33. Ureter is supplied by all except
a) Gonadal arteries
b) Renal arteries
c) Common iliac arteries
d) Inferior mesenteric arteries
34. Resting membrane potential is maintained by
a) Potassium efflux
69

b) Na+/K+ pump
35. Oral contraceptive causes what in pulmonary artery
a) Infarction
b) Embolism
36. Uncoupling of oxidative phosphorylation and heat production is the related to
a) Cortisol
b) Adrenaline
6
This document is the property of FB group: Gynae Obs Past Papers FCPS-1
c) Thyroxine
37. Myleperoxidase is an enzyme present in
a) Neutrophil
b) Basophil
c) Lymphocytes
38. Old age HB Dec, RBC increased, WBC increased, platelets increased with hypeplastic bone
marrow with weakness and lethargy, what’s your diagnosis
a) CRF
b) Megaloblastic anemia
c) Acute leukemia
d) Myleofibrosis
39. A girl 10 years with HB 5 MCV 5,MCH 15, MCHC 18 with splenomegaly
a) Iron def anemia
b) Anemia of chronic diseases
c) Beta thalessemia major
40. 40 years lady with 2 year history of pruritis ,for the last 6 months having xanthoma,
hepatospleenomegaly, which investigation will confirm Diagnosis
a) ALT
b) AST
c) U/S
d) CT scan
41. AFP is not raised in
a) Hepatitis
b) Pancreatic carcinoma
c) Liver carcinoma
42. New cases per 100,000 in a given time
a) Incidence
b) Prevalence
43. A specific group of patients selected and their outcome is checked by
(I don’t remember if this was an mcq – please help me correct it)
7
This document is the property of FB group: Gynae Obs Past Papers FCPS-1
a) Stratified sampling
b) Cross control studies
c) Cohort
44. Pubic bones are an example of
a) Symphisis joint
b) Syndesmosis
45. One of the following does not regenerate
70

a) Adrenal gland
b) Cornea
c) Lens
d) Epithelium
46. Ureteric bud does not grow towards metanephric cap causes
a) Absent ureter
b) Absent kidney
c) Polycystic kidney
d) Pancake kidney
47. Milk presents in the breast during pregnancy with normal prolactin level but no lactation it is
due to inhibitory effect of
a) Estrogen
b) Progesterone
c) Oxytocin
48. Antimicrobial and protective effect in alveoli is produced by
a) IgA
b) Alveolar macrophages
c) Mucin
49. Pulmonary artery supplies
a) Tertiary bronchi
b) Secondary bronchi
c) Alevoli
8
This document is the property of FB group: Gynae Obs Past Papers FCPS-1
50. Primary hepatocellular carcinoma is caused by all of the following except
a) Hepatitis C
b) Alfa 1 antitrypsin deficiency
c) Toxins in food
d) Aflatoxin
51. Vaginal lymphatic drainage below the hymen is to
a) Superior inguinal lymph nodes
b) Aortic nodes
c) Inferior mesenteric nodes
d) Superior mesenteric nodes
e) Femoral lymph nodes
52. A patient can’t store food in vestibule, can’t blow, cannot frown and cannot clench his teeth,
which nerve has been damaged
a) Facial nerve
b) Mandibular nerve
c) Vagus nerve
d) Maxillary division of the mandibular nerve
53. Inferior orbital branch is a terminal branch of
a) Facial artery
b) Maxillary artery
c) Transverse facial artery
54. Lengthening of a muscle is controlled by
a) Golgi tendon
b) Muscle spindle
71

55. Alpha blocker effect


a) Mydriasis
56. Benign tumor
a) Keratocanthoma
b) Warthin
9
This document is the property of FB group: Gynae Obs Past Papers FCPS-1
c) Wilms
57. Aggressive tumor
a) Squamous cell carcinoma
b) Melanoma
c) Compound nevus
d) Basal cell carcinoma
e) Intraepithelial Paget's
58. leiomyoma overlying endometrium shows increased fibrosis with scanty glands/ducts in the
stroma
a) Atrophy
b) Hyperplasia
c) Dysplasia
59. Metaplasia
a) Change in function of the cells
b) Change in size and shape
60. Amniotic fluid embolism
a) During labor and postpartum
b) Soon after delivery
c) Postpartum
d) Complicated labor like abruptio placenta
61. Inguinal canal in female contains
a) Iioinguinal nerve and round ligament
62. Bladder does not have
a) Trigone
b) Detruser
c) Ureteric opening
d) Pubovesical
63. Nerves in pelvis
a) Obturator
10
This document is the property of FB group: Gynae Obs Past Papers FCPS-1
64. Anaphylactic Shock (What was the correct stem plz)
a) Arterial dilation
b) Endotoxin
c) Increased capillary permeability
d) Venous dilation to increase venous return
65. Fetal adrenal gland produce
a) Testosterone
b) Progesterone
c) DHEA
d) Pregenolone
72

66. Parturition is caused by


a) Fetal cortisol
b) Oxytocin
67. Cardiac output increases in
a) 1st trimester
b) 2nd trimester
c) After delivery
d) 3rd trimester
68. Gonococci obtained from exudate most easy test to diagnose is
a) Gram staining
b) ZN staining.
c) Culture
69. 40 years lady with post coital bleeding and inter menstrual bleeding on examination
hypertrophic cervix, first investigation to do
a) Cone biopsy
b) Cervical smear
70. Which is not appropriate
a) Widal in 1st week
b) VRDL not specific for syphilis
11
This document is the property of FB group: Gynae Obs Past Papers FCPS-1
71. Sjorgen most sensitive
a) Anti DS DNA
b) Anti SSA
c) ANA
72. Systemic thrombi mostly arise from
a) left art mural thrombi
b) left ventricle
73. Sensitive for cardiac muscles
a) Myoglibin
b) ADH
c) Trop T
74. Inscisionall hernia from within outwards
a) Transversls fascia
b) External oblique
c) Internal oblique
75. Hesselbalch triangle gives way to
a) Indirect inguinal hernia
b) Direct Inguinal hernia
76. ADH
a) Supra optic nucleus
77. SA node
a) Superior part of sulcus terminals
b) lower part crista terminals
78. Increased lipid solubility means
a) Increased entry into cell
b) Rapid action
79. Diuresis
73

12
This document is the property of FB group: Gynae Obs Past Papers FCPS-1
a) Negative water clearance
b) Decreased Na excretion
c) Osmalility diuresis
80. Causes vasoconstriction
a) ADH
b) Dopamine
81. Cannot cross BBB
a) Levodopa
b) Dopamine
82. Pellagra is caused by
a) Niacin
83. Difference between malignant and normal cells such that malignant cells repair the shortening
of chromosome so that they can continue dividing. Enzyme responsible
a) Telomerase
b) Reverse transcriptase'
c) Isomerase
84. Tumor supper gene
a) P53
b) Bcl
c) M-Myc
d) L-Myc
85. An person fall had abrasion and bleeding first effect will be
a) Platelets adherence
b) Vasoconstriction
86. Memory center is present in
a) Temporal
b) Frontal
c) Parietal
13
This document is the property of FB group: Gynae Obs Past Papers FCPS-1
87. Succinylcholine
88. Increase in BT ,Increase CT with diarrhea
a) CRF
b) Vitamin K deficiency
c) Chronic liver disease
89. 4th heart sound is produced by
a) Ventricular filling
b) Atrial filling
90. Breast implants with inflammation, the type of cell found are
a) Giant cells
b) Mast cell
c) Plasma cells
91. Axillary sheath is derived from
a) Prevertebral fascia
b) Pretracheal fascia
92. There is decreased pO2 in arteial blood than pulmonary artery because
74

(Correct the options and the stem please)


a) Diffusion to occur
b) Due to dead space
93. If 28 day cycle on day 21 progesterone will be
a) 5ng
b) 10ng
c) 18ng
d) 20ng above
94. Pillar of medical ethics
95. local cause if edema
a) Allergy
14
This document is the property of FB group: Gynae Obs Past Papers FCPS-1
b) Arterial dilation
c) Portal Hypertension
96. Pseudohermaphroditism with 46XX
a) Turner syndrome
b) Klienfelter syndrome
c) Adrenigenital syndrome
d) Mixed gondal dysgenesis
97. Effect of oral contraceptives
a) Prevention of ovarian carcinoma
b) Prevention of vaginal carcinoma
c) Treatment of cervicitis
98. Quartan malaria
a) Plasmodium malariae
b) Plasmodium vivax
c) Plasmodium falciparum
99. Sorbitol
a) Alcohol sugar
100. Pyruvate intermediate between
a) Glucose and acetyl coA
b) Acetyl acetate and coA
c) Glucose and carbohydrates
101. Cause of insulin secretion
a) Glucagon
b) Somatostatin
c) Beta blockers
102. GFR is increased by
a) Increase in GFR pressure
b) Decrease in hydrostatic pressure
15
This document is the property of FB group: Gynae Obs Past Papers FCPS-1
c) Decrease in oncotic pressure
103. Gas gangrene is caused by
a) Clostridium perfringens
b) Clostridium welchi
c) Clostridium septicum
75

104. Platelets cannot be given in


a) Autoimmune the on Oct to pennies
b) Spleenomegaly
105. Amino acid with sulphur group
a) Cysteine
106. Contents between lower layer of urogenital diaphragm and perenial membrane
a) Sphyncter urethrae
b) Bulbourthral glands
c) Superficial transverse perennial membrane
d) Neck of bladder
107. lymph drainage of medial breast lobe
a) Medial mediastinal lymph nodes
108. lymph drainage of upper outer quadrant of the breast
109. Cervical lymph drainage
110. Below knee amputation irregular mass after sometime
a) Neuroma
b) Ganglioma
c) Neuroganglioma
111. Area 3,1,2 is associated with
112. Transtional epithelium
a) Cilaited
16
This document is the property of FB group: Gynae Obs Past Papers FCPS-1
b) Pseudostratified
c) Stratified cuboidal
d) At the base is flatter while more disatally cuboidal
113. Bipolar cells are found in
a) Nasal epi
b) Anterior horn
c) Posterior horn
114. lobes of breast through many terminal ducts open into
a) lactiferous duct
b) lactiferous sinus
115. Kidney
a) Lobules constitute the parenchyma
b) Adrenal gland and kidney are separted by parirenal fat
c) Basement membrane is thicker then other basement memranes in the body
116. Oxygen consumed by brain(Cerebral Cortex) per minute
117. Total gastrectomy cause
a) Pernicious anemia
b) Iron deficiency anemia
c) Megaloblastic anemia
118. Pseudo membranous colitis caused by
a) Clostridium difficile
119. An MCQ about the diameter of pelvic outlet
120. Patient has creatinine 7, BP 210/130 the pathology involves
a) Peritubllar arterioles
b) Macula densa
76

c) JG cells
121. Bronchopulmonary segment is aerated by
a) Tertiary bronchus
17
This document is the property of FB group: Gynae Obs Past Papers FCPS-1
b) Secondary bronchus
c) Has its own vein
122. Definition of Proband (And MCQ from Biostats)
123. Left adrenal gland drains into
124. Connection between cells with protein channels and low electric resistance
a) Tight junctions
b) Gap junctions
c) Desmosomes
125. Free radical injury produces pathology by affecting
a) Cell wall
b) DNA
c) RNA
126. 60 year old woman a known case of Ischemic heart disease, Carcinoma breast has now
come with osteoporosis. Which drug you will prescribe to treat her
a) Beta blocker
b) Raloxifene
127. Most aggrassive tumor
a) Squamous cell carcinoma
b) Melanoma
128. Most lethal hepatitis in pregnancy
a) Hep A
b) Hep B
c) Hep C
d) Hep D
e) Hep E
18
This document is the property of FB group: Gynae Obs Past Papers FCPS-1
129. Alpha one receptor supplies
a) iris
130. 0.5% bupivacain in 100 ml soultion contains
a) 100mg
b) 200 mg
c) 300 mg
d) 500mg
131. In osteoporosis which of the fallowing is defective
a) Bone collagen
b) Ostoblasts
c) Osteoclasts
d) Structure of PTH
132. Respiratory tract is supplied by bronchial and pulomnary arteries. What artery
supplies the alveoli?
133. Functional layer of endometrium consists of
a) Stratum basale
77

b) Stratum compactum and stratum spongiosum


c) Stratum basale and compactum
134. Which one of the following will not regenerate
a) Retina
b) lens
c) Cornea
d) Epithelium
135. When one molecule of glucose is completely oxidized to carbon dioxide and water no
of ATP molecule formed
a) 10
19
This document is the property of FB group: Gynae Obs Past Papers FCPS-1
b) 20
c) 30
d) 38
136. Which dont have agglutinin
a) O +
b) O –
c) Ab +
137. Dilation of pupil by
a) Sympathetic
b) Parasympathetic
138. Incidence
a) No of new cases in a particular area at a particular time
139. Action due to alpha receptor
a) Increased GIT motility
b) Sphyncter contraction
140. The structures included in the upper fascial layer and the lower fascial layer of
urogenital diaphragm are
a) Sphyncter uretherae
b) Vaginal sphincter
141. Inferior layer of uroginital diaphragm
a) Camper’s fascia
b) Colle’s fascia
c) Perinneal fascia
d) Perinneal membrane
e) Scarpa’s fascia
142. Hessel Bach triangle... Indirect ingunal hernia
143. Gonococci easily detected on gram staining
20
This document is the property of FB group: Gynae Obs Past Papers FCPS-1
144. Cause of emniotic fluid embolism
a) C-section
b) Normal delivery
c) Post partum hemorrhage
d) C/S+ post partum
145. Fast impulse passes through
a) Sa node
78

b) Av node
c) Intermodal pathways
d) Pukinje fibers
146. Diabetic nephropathy is best diagnosed by
a) Ultrasonography
b) Plasma albumin
a) Which hormone produced by fetus is required to start labour
a. Cortisol
b. Corticosterone
c. Estradiol
147. The compound containing the highest content of triglycerides is
a) LDL
b) Chylomicrons
c) VLDL
d) HDL
148. Which of the following does not delay the complication of brain tumor
(Please help correct the stem)
a) Brain cancer
b) Gilioma
c) Brain edema
21
This document is the property of FB group: Gynae Obs Past Papers FCPS-1
149. 25 year old female come with 34 weak gestational amenorrhea HB being 9gm/dl. Her
blood picture shows normocytic normochromic picture
a) Megaloblastic anemia
b) Iron deficiency anemia
c) Blood volume expansion
150. 4th heartsound is produced by
a) Atrial systole
151. Proband
a) It is 1st family member who is affected from disease with no previous history of genetic
diseases who need treatment.
152. Which is not a cause of hepatocellular carcinoma
a) Hep A
b) Hep B
c) Hep C
d) Hep H
e) Hep D
153. Urter is supplied by all arteries except
a) Renal Artery
b) Vesical Artery
c) Inferior Mesenteric
154. Inferior rectal artey is a branch of
a) Internal pudendal artery
155. A lady diabetc and asthmatic was taking oral hypoglycaemic drugs after an attack of
asthma and drug intake her bsl rises it is due to
a) Beta blockers
b) Steroids
79

156. Cause of edema


a) Blockage of lamphatics
22
This document is the property of FB group: Gynae Obs Past Papers FCPS-1
b) Inrcreased hydrostatic pressure
157. local cause of edema
a) Allergy
158. Maternal mortality - You should know the formula
159. A patient with pruritis for 4 months, jaundice, hepatosplenomegaly , the best
investigation to find the cause is
a) Ultrasound
b) CT Scan
c) Alkaline phosphatase
160. The cause of primary hepatic carcinoma
a) Alpha antitrypsin 1 deficiency
b) Aflatoxin 1 in diet
161. Tumor suppressor gene
a) p53
162. Unlikely about anaphylactic shock
a) Endotoxin release
b) Increased venous return
c) Arteriolar dilation
163. Viruses produce damage by denaturing proteins
164. A question about single blinding
165. Pregnant lady HB 9 normo chromic - Physiological change/Plasma volume expansion
166. Gas gangrene in super limb - Clostridium welchi
167. Atrial fibrillation - Pulse deficit
168. A person picks a thorn and abscess is formed, organism involved - Staphylococcus
Aires
169. Axillary sheath is the extension of
a) Pretracheal
23
This document is the property of FB group: Gynae Obs Past Papers FCPS-1
b) Prevertebral
c) Superficial cervical fascia
170. Arterial emboli mostly arises from
a) Femoral
b) Mural left artrium
c) Right atrium
171. Which causes release of insulin
a) Glucagon
b) Thaizides
c) Potassium
172. Patient brought to emergency - stiff neck, spinal tap was bloody
a) Subarachnoid hemorrhage
173. Arterial supply of prostate?
174. Diarrhoea after jejunostomy
a) Osmotic ,b) Secretory, c) Both
80

=-=-=-=-=-=-=-=-=-=

7. FCPS 4th Feb 2015 by Dr.BadalKhan / Nasar Yaqoob (187)

Q1.Diabetic ketoacidosis pt managed bt died.most likely infection is


a.mucur [ANSWER] b.candida c.E.coli
Q2.The specific feature of Apoptosis
a. cell shrinking [ANSWER] b. karyolysis
c. hyperchromasia d. cell swrlling
Q3.A tumor with all germ layers
a. teratoma [ANSWER] b. mixed mesodermal tumor
c. cysts
Q4.An obese man with h/o atherosclerosis underwent laparotomy due to bleeding in abdomen n
during surgery, his small gut was purple n sup mesenteric artry was blocked , so what is dx
a. wet gangrene [ANSWER] b. dry gangrene
c. thrombosis d. volvulos
Q5. ADH is inhibited by
a. alcohol [ANSWER] b. aldosteron
c. rennin
Q6. Wound of surgery with green discharge is due to
a.pseudomonas [ANSWER] b. salmonella
c. candidiasis d. mycobacterium
Q7. Hiv associatd with
a.decreased CD4 [ANSWER] b. cytotoxic T cell
Q8. Open wund healing includes
a.myofibroblast [ANSWER] b.fibroblast
c. macrophages d. lymphocyyes
Q9. S1 is lost so there will b
a. ulcer on medial n lat sole b. loss of sensation on lat leg
c. flexion of ankle affevted d.loss of ankle jerk [ANSWER]
Q10.Cardiac out put depends on
a. venous return [ANSWER] b. exercie
c. stroke volume or [ANSWER] d. EDV
Q12.Following has highest clearance value
a. inulin b. para amino hippuric acid [ANSWER]
c. K d. Na
Q13.Apex of heart is
a.about 8cm from mid line [ANSWER] b. in 3rd intercostal space
c. inguinal canal d. pulmonary trunk
Q14.FRC is sum of
a. vital capacity plus tidal vol b. vital capacity plus exp reserve vol
c. ERV and RV [ANSWER] d. can b measured on spiromrtry
Q15. A girl of 11 yr was reffered to x ray department for xray of elbow joint to know age ,
a. capitulum b. trochlea
c. med epicondyle d. lat epi condyle [ANSWER]
Q16. quadrate lobe
a.functionally part of rt lobe b.drains into left hepatic duct [ANSWER]
c.lies bte inf vena cava and lig venosum.
Q17. Intrinsic pathway is activated by
a.contact of tissue collagen when it comes in contact [ANSWER]
81

b. factor X
Q18.Which is not a 'basic tissue of body'?
a.nerves b.muscles
c.blood [ANSWER] d.epithiliuem e.connective tissue
Q19.Cervical rib attached to trverse process of cevical rib will compress:
a.C5 b. C8 c.T1 [ANSWER]
Q20. Aortic aneurysm which structure not compressed:
a. Esophagus b. Trachea
c.left primary bronchusd d.Thoracic duct e. Phrenic nerve [ANSWER]
Q21. Resting membrane potential acieved by:
a.diffusion of potassium outside the cell b.Na K pump [ANSWER]
c.is same for all types of cell
Q22. What is median if values are 20 20 25 30 35 40 25 30:
a.27.5 [ANSWER] b.25 c.30
Q23. Dead organisms in vaccine :
A. Measles B. Mumps
C. Tetanus d.pretussis [ANSWER]
Q24. Esophagectomy done , stomach is mobilized to upper portion of thorax .. To
anastomosis with
phayrnx, Which artery is responsible for stomach supply now
a.left gastric [ANSWER] b.Right gastric
c.Left gastroepiploic d.Short gastric vessels
Q25. Patient is pregnant and suddenly faints due to less blood ot brain or something.
She'll be
positioned in left lateral and not right lateral to avoid compression to which abdominal
structure:
a.IVC [ANSWER] b.SVC c. Liver
Q26. Post op wound having greenish pus discharge organism involved:
a. Staph Aureus b.pseudomonas [ANSWER]
c. Streptoccocus Virid d. Kliebsella
Q27. Common carcinogen of Ca cervix:
a.HPV [ANSWER] b. HSV
c. Chronic irritation d. IUD
Q28. Chronic Myeloid Leukemia
a.chromosome 9&22 [ANSWER] b. chromosome 11& 22
C. chromosome 14 & 22
Q29.common Post op infection in hospital cause by which organism :
a. Staph Aureus [ANSWER] b.pseudomonas
c. e.coli d. Kliebsella
Q30. Steroids drug action on cells:
a.act on genes [ANSWER] b. Adenyl cyclase
c. CGmp d. Ip3
Q31.claw hand produced by unopposed action of
a.flexor digitorum longus and flexor digitorum profundus
b.extensor digitorum longus and extensor digitorum indices
c.extensor digitorum and flexor digitorum profundus [ANSWER]
Q32.blood supply of head of humerus is
82

a.anterior circumflex artery [ANSWER] b.posterior circumflex artery


c.suprascapular artery d.subscapular artery
Q33. Organelle have microtubules in its structure:
a. Golgi apparatus b. Mitochondia c.centriole [ANSWER]
Q34.blood supply of head of femor is
a.femoral artery and obturator artery b.
Q35.function of anterior cricuiate ligmant is
a.prevents forward sliding of the tibia on the femur [ANSWER]
b.prevents backward sliding of the tibia on the femur
Q36.if iodine is nt used for formation of thyroid hormorne,then iodine excreted which site
of the
body
a.stool b.urine [ANSWER] c.salivary gland
Q37. Mucus not secreted by:
a. Larynx b. Trachea
c.terminal bronchioles [ANSWER] d. Nasopharynx
Q38.coagulative necrosis occurs due to
a.ischmia [ANSWER]
Q39. A man of 70 has recurrent UTI cause (No clue was mentioned regarding
bedridden):
a. Instrumentation b.outflow obstruction [ANSWER]
c. Diverticulum d.catheterization
Q40. Plasma Protein Binds With All Except:
a.oxygen [ANSWER] b. CO2
c. Iron d. Bilirubin e. Thyroxin
Q41. Which drug cause tachycardia in therapeutic doses:
a.morphine [ANSWER] b. Fentanyl
c.afentanyl d.pethidine
Q42. Highest bioavailability of anti emetics:
a.metaclopromide [ANSWER] b. Qdansetron
Q43.f there is abnormal metabolism of amino acid.which of the following amino acid will
appear in
urine
a.alanine b.tryosine c.tryptophan
Q44.nerve supply of extensor digiti minimi
a.deep radial nerve [ANSWER] b.superfcial radial nerve
Q45. Pregnant lady anaemic, Best lab investigation:
a. S. ferritin b .CBC [ANSWER] c. TIBC
Q46. HIGHEST % OF PROTEIN IN WHICH:
a. VLDL b.HDL [ANSWER]
c. LDL d. Chylomicrons
Q47.ruttor cuff is formed by
a.supraspinatus,infraspinatus,teres minor and subscapularis [ANSWER]
Q48. Decrease Insulin will leads to:
a.Increase activity of Lipoprotein Lipase b.ketogenesis in liver [ANSWER]
Q49.the structure which prevents muscle from tearing under a constant presuure
83

a.ruffinis endings b.golgi tendon organ [ANSWER]


c.merkle cells d.muscle spindle
Q50. First heart sound:
a. Atrial systole b.isovolumic contraction [ANSWER]
c. Isovolumic relaxation d. Rapid ejection
Q51.regarding golgi tendon organ it
a.senses dynamic length of muscle b.senses muscle tension [ANSWER]
c.alpha motor neuron stimulation d.involved in reciprocal innervation
Q52. 55yrMan With Weakness And DysphagiaTarget Cells, Blood Profile With :
a. AOCD b.iron deficiency anemia [ANSWER]
Q53.optic chiasma lesion causes
a.bitemporal hemianopia [ANSWER] b.right homonymous hemianopia
c.left homonymous hemainopia d.ipsilateral temporal hemianopia
Q54. Antioxidant Vitamin:
a.vit A b.vit E [ANSWER] c. vit C
Q55. Which of the following does not increase in stress for surgery:
a.ADH b.insulin [ANSWER]
c. Glucagon d. ACTH
Q56. Biguanides side affects:
b. Elevated ALT b.diarrhea and flatulence [ANSWER]
Q57. Patient with Gastrectomy presents with Anemia after One and half year, what will
you give:
a. Vit C b.VIT B12 [ANSWER]
c.Iron d. Blood transfusion
Q58. Data is collected for some clinical trial based on presence or absence of Vomiting.
What type
of data it is
a. Ordinal b.nominal [ANSWER] c. Parametric
Q59. Aphasia (Non-fluent) after stroke or Trauma which area of Brain is involved:
a. Wernickes b.brocas [ANSWER]
c. Temporal region d. Pyrimidal gyrus
Q60. What cytoskeletal structure connects Extracellular matrix to Intracellular:
a. Integrins b.cadherins [ANSWER]
c. Intermediate Filaments d. Microtubules
Q61. Wich Antiemetic has high bioavailability:
a. Prochlorthiazide b.metoclopramide [ANSWER]
c. Ord[ANSWER]etron d. Droperidol
Q62. Cardiac defects in fetus of mother suffering from rubella infection will occur if:
a. 3 rd month b. 4 th month
c.5th to 10th week [ANSWER] d. 7 th month.
Q63.in chorea which nucleus effected
a.putamen b.caudate [ANSWER] c.globus pallidus
Q64.in athetosis which nucleus effected
a.putamen b.caudate [ANSWER] c.globus pallidus
Q65. Downs Syndrome:
a. Extra chromosome
84

b.increase incidence with increasing mother age [ANSWER]


Q66. Pulmonary vasoconstruction occurs due to:
A. Raised PCO2 b.reduced systemic po2 [ANSWER]
C. Reduced pH
Q67. Most imp buffer in blood:
a. Hb b.HCO3 [ANSWER]
Q68. Which vitamin act as cofactor in oxidation of fatty acids:
a. Vit B1 b.biotin [ANSWER] c. Vit C
Q69.xytocin and vasopressin originated from:
a.Neurohypophysis b. Adrenal medulla c.hypothalmus [ANSWER]
Q70. If FACTOR IX not available:
a. Cryoprecepitate b.FFP [ANSWER]
c.platelets d.whole blood
Q71.in von willebrand factor which one is given
a. Cryoprecepitate [ANSWER] b.FFP
c.platelets d.whole blood
Q72.Lt kidney is not related anterioly to
A. Diaphragm [ANSWER] b. splenic flexure colon
c.3rd part of deudonum
Q73.On xray,shadow of heart,right border is formed by
a.Right atrium b. Ascending aorta
c.Left ventricle d.SVC [ANSWER] e.IVC
Q74right border of heart is formed by
a.Right atrium [ANSWER] b. right atrium and svc
c.Left ventricle d.SVC e.IVC
Q75.whic artry occlusion cause posterior 1/3rd of inter ventricular septum m.i
a. lft c artry b.rca [ANSWER]
c. inter ventricular artry
Q76.golgi tendon organ is associated wth:
a.measurement of length in muscle [ANSWER]
b.measurement of tension developed in muscle [ANSWER]
Q77.Metaplasia
a.Increse in numbr of cell b.Size Increse
c.transformation of one with other epithelium [ANSWER]
Q78.patient with seizures hypotonia anticalcium antibodies present what is the
diagnosis
a.duchene b.beckar c.lambert eaton syndrome [ANSWER]
Q79. which of the following drain into superior mesentric lymph node?
a.Jejunum [ANSWER] b.4th part of duodenum
c.Desending colon d.cecum
Q80.Fever chills 10 days. Pain in chest on lying.structure involved
a.pleura b.pericardium [ANSWER]
c.myocardium
Q81.The muscle that causes eversion of foot.
a.Tibialis post b.peroneus longus [ANSWER]
Q82.Burkits lymphoma?
85

a.CMv b.EBv [ANSWER]


c.Malaria d.Hiv
Q83.Post synaptic sympathatic mainly secretes
a.Acetylecholine b.nor epinephrine [ANSWER]
c.adrenalline
Q84.muscle is stretched,relaxation will occur due to:
a.muscle spindle b.golgi tendon organ [ANSWER]
Q85.Cervical carcinoma s associated wd?
a.Trichomonas vaginalis b.HPV [ANSWER]
c.Candida albican
Q86.Parotid gland
a.Lat pterygoid lies anterior to pterygoid
b.ext carotid passes through superficial gland
c.facial nerve supplies it
Q87.Broder, s classification of tumour
a.degree of differentiation [ANSWER] b.degree of mitosis
c.grading on histopathology
Q88.mucus not secreted by
a.larynx b.terminal bronchioles [ANSWER]
c.trachea d.nasopharynx
Q89.Effect of drug on a body
a.pharmakokinetics b.pharmodynamics [ANSWER]
Q90.Most common known cause of hepatocellular carcinoma in developing countries
a.Hepatitis B b.Hepatitis C
c.both hep b and hep c [ANSWER]
Q91.Carpal Tunnel Syndrome
a. Anesthesia of thenar muscles b.wasting of thenar muscle [ANSWER]
c.Injury to ulnar nerve d.Injury to radial nerve
Q92.Gluteus medus and minumus cause
a. Adduction and lateral rot b.abduction and medial rotation[ANSWER]
c.Extension d.Flexion e. Standing from sitting
Q93.para sympathetic stimulation cause ...
a.periphral vaso dilation b.decrease heart rate [ANSWER]
c.thick and viscus saliva sectetion
Q94.widest epidural space??
a.just below foraman magnum b.L2 [ANSWER] c .T12
Q95.Which one is used for beta oxidation of fatty acifds and CHO
a. biotin [ANSWER] b. pantathenic acid c.vit B12
Q96.Which is not blood supply of scalp
a.occipital artery b.maxillary artery [ANSWER]
c.supra orbital artery d .supra trochlear artery
Q97.Pancreatitis in fluid accumulation??
a.Sub diapgrm b.Sub phrnic
c.Para colic gutter [ANSWER] d.lesser sac
Q98.Extent to which tissue disturbed by occlusion of its venous or arterial supply does
not
86

depend on
a.anatomical pattern of vascular supply
b.velocity of blood in tissue [ANSWER] c.general state of blood
Q99.Anion gap
a.difference b/w measured cation and unmeasurd anion [ANSWER]
Q100.infective endocsrditis diagnostic investigation?
a.ASO titre b.blood culture [ANSWER]
Q101.Essential fatty acid?
a.linoliec acid [ANSWER] b.palmatic acid
Q102. a pt with histry of bleeding gums n wounds prolongd bleeding time and PT
28SEC (
control 12sec) wats the diagnosis?
a.heamophelia b.christmis
c.won wiliband disease d.factore 7 deficiency [ANSWER]
Q103.drug which is analgesic but not anesthetic
a.desflurane b.isofurane c.nitrous oxide [ANSWER]
Q104.if circumflex branch of left coronary artery is blocked which area will b effected,,,
a.anterior surface of left ventricle b.posterior surface of left ventricle [ANSWER]
c.interventricular septum
Q105.regarding cimitidine intake what effect causes
a. Agranulocytosis b.inibition of hepatic enzymes [ANSWER]
c.anti estero genic effect
Q106.a drug causing ↑GFR will do it by following mechanism
a.↓ing bLOOD flow
b.increasing hydrostatic pressure
Q107.Plasmodium falciparum causes:
a.Hemolytic anemia b.black water fever [ANSWER]
Q108.axone arise from conical shape part of neuron cell body called.
a.perikaryot b.axon hillocks [ANSWER]
c.Dandrites
Q109.peptic ulcer profusely bleeding from posterior wall o duodenum...which artery
involed?
a.gastroduodenal [ANSWER] b.gastroepiploic
Q.110.Epithelium of kidney is developed from
a. endoderm only b. ectoderm only
c. mesoderm only [ANSWER] d. endoderm + mesoderm
e. ectoderm + mesoderm
Q111.deep inguinal ring is present in
a.transversis abdominis muscle b.transversalis fascia [ANSWER]
c.internal oblique muscle
Q112.wash red bloood cell use to abolish
a.hypersensitivity [ANSWER] b.Infection
c.Transfusion reaction
Q113.Scrotal carcinoma - spread to
a. Deep inguinal b.Superficial inguinal [ANSWER]
c.Para-aortic
87

Q114.parathyroid glands are developed from


a.1st pharyngeal pouch, b.2 pouch
c.3rd pharyngeal pouch d.4th p pouch
[ANSWER] both 3rd and 4th correct
Q115.In IV drugs abusers
a. Libman Sacks endocarditis b.bacterial endocarditis [ANSWER]
c. Mitral valve carditis d. Rheumatic fever or heart disease
Q116.true abt vertebral column
A.cervical nerves originate above crresponding vertebra [ANSWER]
B.vertebral artery arches ant part of post arch of axis
Q117.papillary carcinoma of thyroid occurs due to
a.radiation [ANSWER]
Q118.seminoma in male ,if same origion occurs in overy then its called
a.yolk sac b.embrynal c.dysgerminoma [ANSWER]
Q119.which of the following is premalignant lesion
a.lichen planus [ANSWER]
Q120.ewing sarcoma occurs which site of bone
a.diaphysis [ANSWER] b.epiphysis c.metaphysis
Q121.spermatogenesis is stimulated by
a.fsh b.testosterone
c.LH d.FSH and testosterone [ANSWER]
Q122.an old diabetic develops sensory abnormalities of left foot and ulcer of big toe .the
pathogenesis of this condition is best described by
a.angiopathy with neuropathy [ANSWER] b.accelerated atherosclerosis
c.microangiopathy d.vascular occlusion with infection
Q123.a pt is having generalized pain,and taking nsaid drugs,nw developed swelling n
big toe,
wht will b present in fluid
a.mono sodium urate crystals [ANSWER]
Q124.type 1 hypersensity reaction
Q125.cardiac reserve
a.increase in athelet
Q126.regarding neutrophil correct statment is
a.migrate into and out of blood stream [ANSWER] b.more phagocytic n blood stream
c.decrease n infective carditions d.decrease whn corticosteroid given
Q127.regarding vit k
a.is a water soluble b.produced by intestinal bacteria [ANSWER]
c.present n large amount n human and cow milk
Q128.human placenta has
a.amnion on its fetal side [ANSWER] b.chorio allantoic placenta
c.get separated along the stratum spongiosum
d.get seperated whn is rupture of may uterine arteries
Q129.regarding vertebra
a.31 pairs of spinal nerve
b.each cervical nerve arises above the crossponding vertebra [ANSWER]
Q130.regarding oculomotor nerve
88

a.arising from trigeminal gangalion


b.all the fibers going to sphincter pupillae muscle parasympathetic [ANSWER]
Q131.ulcer caused by anarobic staphyloccus
a.arterial ulcer b.venous ulcer c.marjulan ulcer
Q132.epididymis drain into
a.vasdeferens [ANSWER] b.ejaculatory duct c.seminal vesicle
Q133.person naked n room,temperatue 21 centigrade ,humidity present,loss of heat by.
a.insensible perspiration b.sweating
c.conduction and radiation [ANSWER] d.urination
Q134.a pregant lady has bile duct obstruction,which s marker best
a.gama glutamyl transferase [ANSWER] b.LDH
c.ALT d.alkaline phosphatase
Q135.regarding lesser sac
a.double peritonal layer [ANSWER]
Q136.congenital catract caused by
a.rubella [ANSWER] b.
Q137.lidocaine mechanism of action
a.blocks na channels [ANSWER] b. block k channels
Q138.extent to which tissue distrurbed by occulsion by its venous or arterial supply
doesnt
depend on
a.velocity of blood in tissue
b.anatomical pattern vascular supply
c.general state of blood
Q139.when head of fetus became exactly half of the length at crl
a.3 month [ANSWER] b.4th mont c.7th month
Q140.malignant neoplasia which rarely metastasized
a.basal cell carcinoma [ANSWER] b.breast carcinoma
c.bronchial carcinoma
Q141.most appropriate about clavipectoral fascia
a.coers pectoral minor [ANSWER] b.covers pectoral major
Q142.lysosome has secretion against bacterial iron, bkz thy contain
a.hydrolases [ANSWER] b.oxidases
Q143.normal t cells and low b cells and mild anemia
a.bruton b.aplastic anemia c.thallasemia
Q144.cricoid cartilage
Q145.blood supply of heart is regulated by
a.local metabolites [ANSWER]
Q146.coronary artery corpus callosum forms
a.join 2 hippocampus b.posterioly forms a ridge
c.join parts in same cerebral hemisphere
Q147.about small intestine true is
a.valvae connivents most numerous in ileum
b.mesentry start from the rite of L2
c.valvae connivents 5mm wide
d.small intestine 6-8 meters long [ANSWER]
89

Q148.right horizontal semicircular canal stimulated


a.head tilt to right b.head tilt to left
Q149.end product of purine metabolism
a.uric acid [ANSWER]
Q150.basic scavengers
a.macrophages [ANSWER]
Q151.great cerebral vein does not drain into
a.occipital vein [ANSWER]
Q152.acute inflamation protein more thn 3
a.exudate [ANSWER]
Q153.regarding gap junctions
Q154.recurrent ulcers n mouth
a.atrophic stomatitis [ANSWER]
Q155.orofecal transmission
a.hep e [ANSWER]
Q156.elderly with scenario of malnutrition the most unlikely is
a.thromocytopenia [ANSWER]
Q157. cause of death in clostridium infection
a.toxemia [ANSWER] b.toxic shock syndrome
Q158.vocal cords atrophy
Q159.correct about aorta
a.lies to rt of cisterna chyli b.divides at L5
c.give branches to parities at L1 d.gives renal arteries at L2 [ANSWER]
Q160.a girl has midline neck swelling ,histology shws normal thyroid tissue ,which
epithalium
a.simple columner wid ciliated cells
b.cuboidal follicular [ANSWER] c.stratifed squamous
Q161.a child came with bleeding , which investagation will u do
a.aptt [ANSWER] b.bleeding time
c.clotting time d.pt
Q162. farmer wid migratory lesion on foot
a.leismania b.strongyloides stercoralis
c.cutaneous larve migran [ANSWER]
Q163.nerve loss in urogenital ,which nerve is severed
a.obturator b.pudenal [ANSWER]
Q164.true abt throacic duct ,drain into
a.right brachiocephalic b.svc
c.at confluence of left subclavian and left jugular [ANSWER]
Q165.a man comes ist time to pak,develops fever thn coma,on investagation 10 percent
platelets ,wbc dec
a.memingoencephalitis b.dengue
c.falciparum [ANSWER]
Q166.main class associated wid antigen presenting celles
a.mhc1 b.mhc2 [ANSWER]
Q167.about posterior triangle correct statement is
a.apex is mid of clavicle b.formed by ant border of sternocleidmastoid
90

c.contains subclavian,three trunks of brachial plexus [ANSWER]


Q168.tumor n children with blue cells and high levels of catecholamines
a.nephroblastoma b.neuroblastoma [ANSWER]
c.wilms tumor
Q169.abt trichomonas vaginalis correct is
a.can b a commensal n vagina b.sexually transmitted [ANSWER]
c.treated wid quinolones
Q170.gustarty sweating is caused by
a.submandibular excision b.superficial parotectomy [ANSWER]
c.wisdom tooth extraction
Q171.regarding thyroid
a.lympatic drainage n deep cervical lymph node [ANSWER]
b.isthmus is level of c4 and c5
c.its covered by prevertebral fascia
Q172.secondry center of ossification is
a.epiphysis [ANSWER] b.diaphysis
c.metaphysis
Q173.which of following predisposes carcinoma of billery system
a.clonarchis sinensis [ANSWER] b.taenia solium
Q174.which of the following is hypovoliume
a,oliguria [ANSWER]
Q175.starling effect
a.increase venous return [ANSWER]
Q176.slow growing tumor of parotid gland
a.pleomorphic adenoma [ANSWER]
Q177.propulsive movement of intestine effected ,due to lesion of
a.aurbach [ANSWER] b.myntric pleuxes
Q178.absolute lymphocytes with b sore throat
a.infectious mononucleosis [ANSWER]
Q179.K excretion n diet distal tubules
a.distal tubules [ANSWER]
Q180.narrowest part of urethra
a.external meautis [ANSWER]
Q181. ca prostate spread by
a.vertibral plexus [ANSWER]
Q182.INH
a.pyrodoxine [ANSWER]
Q183.after non lethal mi in 4 minutes
a.arrhythmias [ANSWER] b.complete recovery
Q184.Artenoid cartilage a.give attachment to inferior constricter [ANSWER]
Q185.opsonization occurs a.help of c3b [ANSWER]
Q186.subdural hematoma a.diploc fracture [ANSWER]
Q187.fracture of 9th and 10th ribs will cause injury of which organ
a.spleen [ANSWER] b.pancrease
=-=-=-=-=-=-=-=-=-=-=-=-=-=-=--=
91

8. Radiology 10 june 2015 by Ulnar Nerve(144)


1.. Ion involved in cardiac muscle contraction 1
Na
Ca
2 Resting membrane potential maintained by 1
Na influx into cell
Cl outflux from cell
K outflux from cell
Na K Atpase
3Blood group with no agglutinins 1
AB+
AB-
O-
4 Gas gangrene 1
Cl bifrengen
Cl welchii
Cl septicum
5 Infra orbital artery is a branch of 1
Maxillary A
Facial A
6 pregnant lady what parameters increase 1
TIBC
Ferritin
Iron
7 pregnant lady,normocytic normochromic anemia ,, cause1
Plasma expansion
Folic acid def
Iron deficiency
8 tumor with most aggressive characterisrics(someyhing like that)
BCC ( if melanoma not in option)
Sq. CC
(ACTINIC KERATOSIS,ACANTHOMA,MELANOMA) not sure these were the excat options or not
9 LH hormone effects2
Lydig cell
sertoli cells
10 Lydig cell effects2
Helps in spermatozoa formation
Help pushing ejaculatory fluid from seminal vesicles to ejaculatory duct
(cant remember other options)
11 boy with hyperphagia,obese,short stature,mental retardation,,short fingers2
Prader willi ( trisomy 15 )
Down synd (
Angelman syndrome ( trisomy 15)
12 after total gastrectomy1
Pernicious anemia
Iron def
Folic acid deficiency
13 Na absorption occur at
Jejunum
Ilieum
Colon
14 pulmonary artery supply1
Alveoli
Tertiary bronchioles
Respiratory bronchioles
15 80% oxygen saturation1
Umbilical vein
IVC
Umblical artery
16 dorsal thoracic nerve
92

Lattismus dorsi
17 DVT most commen cause1
Prolong immobilization
Anti thrombin 3
18 hepatocellular carcinoma most common cause1
Hep c
Heo b
Aflatoxin
19 shortest incubation period1
Hep a (10-15 days)
Hep e ( 40 days mean)
hep b
hep d
20 most epidemic1
Hep e
Hep d
Hep a
hep b
21 mean1
Sum of all observations divided by number of observations
Same as average/mode
22 ca cervix,, lumph node involved2
Int iliac
Pudendal
Inguinal
23 anorectal canal lymph drainage2
Obturator
Pudendal
24 dec arterial PO2`1
Hypoxic hypoxia
CO poisoning
Anemic hypoxia
25 jugular foramen
Part of occipital bone
Last 3cranial nerves pass through it
26 In posterior cranial fossa
Straight sinus run inf to cerebellar vermis
Basilar artery pass infront of pons
27 14 years old child with 8% creatinine, bp increase.. damage in 1
Juxtaglomular complex
Macula densa
28 Down syndrome1
Inc incidence with inc maternal age
29 receptor to prevent muscle overstretching 1
Golgi tendon
Muscle spindle
Pacinian corp[uscle
30 posterior division of spinal nerves are1
Extensors of trunk
Extensors of limb
Abductors of limb
31 highest cintent of triglycerides1
VLDL
HDL
Chylomicrons
32 atrial fibrillitation??1
.p waves prominent
.regular rhythm
.pulsus deficit

33 regarding heart1
LCA divides into interventricular and circumflex artery
93

RCA supplies sa node av node,both bundle branch


34 inf wall MI??
35 ESOPHAGEAL ANEURISM INVOLVES2
Left atrium
Left auricle
Right atrium
Left ventricle
36 Right atrium2
related to med surface of left lung
forms right surface of heart
37 increase ejection fraction causes dec in1
ESV
EDV
38 Systemic emboli source1
Left ventricular mural thrombi
Femoral/ popliteal vein
Pulmonary artery
39 transitional epitheliu1
Stratified with round epical cells??(don’t remember the stem.)
40 leomyoma with few glandular tissue1
Hyperplasia
Neoplasia
Atropht
Metaplasia
Dysplasia
41 bodys natural cancer protectuion1
Apoptosis
42 scenario about silicon breast implant,, cells 1
Plasma cells
Giant cells
Neutrophil
43 does not regenerate1
Cornea
Lens
Suprarenal gland
44 adrenal medulla damage,most common cause
Trauma
45 end arteries1
Coronary
Sup ophthalmic
Central retinal artery
46 secrete oxytocin1
Supraoptic nucleus of hypothalamus
47 women with 2 years pruritis,6 months jaundice.. diagnostic test 1
ALP
Anti mitochondrial antibodies
AST
48 short vertebral column,,,, ?/
Achondroplasia
Down syndrome
49pallegra caused by deficeiency of1
Nicotinic acid
Biotin
50 dead space remain unchanged by1
Standing
Old age
Deep inspiration
Shallow inspiration
Tracheostomy
51 cancer is due to 1
Over expression of proto oncogens
52 virus cause damage by1
94

Altering cell protein synthesis


53 scenario,, parietal lobe atrophy, effects
Short term memory
54 about muscles
Origin is proximal and movable
Insertion is distal
Round tendon
Belly fleshy throughout lenghth
Aponurosis
55 alpha receptor stimulation causes
Bronchodilatation
Vasoconstriction
Inc cardiac contractility
56parasympathetic stimulation 1
Contraction of smooth muscles of iris
57 sa node1
Upper part of sulcus terminalis
Lower part of cresta terminalis
58 Memory center 1
Frontal
Parietal
Temporal
59 boy playing cricket had cut on elbow,initial mechanism to control bleeding1
Vasoconstriction
60 axotomy2
Swelling at both end of cut
Orthograde function intact ???
61 difference between primary and secondary intention repair
Epithelial proliferation
Granulation tissue
Keloid
61thyroid
Epith changes during different phases of synthesis
62 young boy,petechial rashes,chronic diarrhea,1
Vit k deficiency
Liver failure
Renal failure
63 4th heart sound1
Rapid ventricular filling (s3)
Atrial systole
64axillary sheath1
Prevertebral layer
Pretracheal layer
sup cervical
65 pseudomembranous colitis1
Cl difficle
Cl perfringens
Cl botulinum
66 poorly cross BBB1
Dopamine
carbidopa
phenytoin
nd
67 2 week1
Epiblast and hypoblast
Epiblast only
Hypoblast only
68does not have lymph nodes1
Thymus
Spleen
69 man had a thorn prick,develop abcess1
Staphylococcus aureus
70 Regarding CSF1
95

pyogenic memingitis,, dec sugar


tb meningitis
fungal meningitis dec sugar
71 4 pillars of medical ethics1
Beneficient,non malificient,autonomy,Justice,
Informed cosent,confidentiality,proper treatment,,Followup
72 indicator of myocardial cell damage, (was ir specific or sensitive)1
Myoglobin (sensitive)
Troponin T (specific)
Lactate dehydrogenase
CK
73 causes insulin release 1
Glucagon
Secretin
Growth hormone
74bronchopulmonary segment aereated by1
Tertiary bronchioles
Sec bronchiole
Primary
75 pneumothorax
Ipsilateral lung collapse and chest wall spring out
76 preganglionic parasympathetic fibers of glossopharyngeal supply2
Parotid submandibular
Sublinguall
Lacrimal
77 pancraetic perforation fluid go to2
Lesser sac
Para colic gutter
Left kidney
Spleen
78 left adrenal vein .ant relation2
3rd part of duodenum
Head of pancreas
Sup mesenteric A
79 appendicitis, lymph nodes involved2
Sup iliac
Inf iliac
80 levator ani nerve supply 2
81 about gall bladder2
Right and left hepatic artery don’t unit?/
Right left hepatic duct unit to form bile duct
Gall bladder supplied by hepatic artery proper
82 ureter 2
Double ureter ??/
Double ureter more common in male
Most constricted at renal pelvis
83 portal duct relation to head of pancreas2
Posterior to ir
Embedded in it
84 epiploic foramen2
85 GFR inc with1
Inc hydrostatic pressure in bowman capsule
86 asbestosis?
87 stab wound,after intercostals pierce2
Endothoracic fascia
Paraital pleura
Visceral pleura
88 brocas area`1
Damage causes Non fluent aphasia
Send impulses to wernickes area
Located in superior temporal gyrus
89 area 3,1,2,,1
96

Somatosensory
90 blood in CSF aspiration came from1
Subarachenoid space
91 water dieresis1
92 methylproxidase presnt in1
Erythroblast
Plasma cells
Neutrophils
93 uncoupled oxidative phosphorylation,heat production by1
Cortisol
Thyroxin
94 RBCs1
Mean MCV 80 fl
Larger than all wbcs
Contain remnants of mitochondria and endoplasmic reticulum
95 L4 damage
ABSENR BABINSKI
Absent dorsiflexion of foor
Absent planterflexion
96 no resistance to flow,, tubules prsesnr1
Gap junction
Desmosomes
Hemidesmosome
97 scenario of patient with small cell carcinoma lung, hormone present
Acth
Insulin
98 hydrocele
Fluid from peritoneal cavities
Orchitis
99 sulphar containing amino acids`1
Argentine
Cysteine
100 lymph drainage from medial side of breast
Internal thoracic
101 pyruvate is intermediate between1
Glucose and acetyl COA
Acetic acid and acetyl coA
102 metaplasia
Functional change in epithelium
Change in size an shape
103 cremasteric muscle nerve supply2
genital branch of genitofemoral nerve
104 blood supply of first part of deudenum2
Right gastric artery
Left gastric artery
Superior gastrodudenal artery
105 3rd part of duodenum anterior relation2
Superior mesenteric artery
106 left adrenal gland drain into`1
Left renal vein
Ivc
107 testes lymph drainage2
Paraaortic lymph nodes
108 lymph from duodenum first goes to 2
Splenic lymph nodes
Pancreaticodudenal
Superior mesenteric
Celiac nodes
109 portosystemic anastomosis2
Umbilical veins with veins of ant abdominal wall
110fibers from cerebral cortex project to 2
Reticular formation
97

Pons
Cerebellum
111best chemotactic`1
C5a
C3a
C5b
Arachidonic acid metabolites
112 right crus of diaphragm
Right phrenic nerve pass through caval opening
113 neuro scenario2
114 neuro 2
115 neuro2
116 amyloidosis?2
117 femur fracture,, amylodosis type
Local amyloidosis
Reactive amyloidosis
118 right brachiocephalic artery from which pouch
rd
3 pharyngeal pouch
st
1 pharyngeal pouch
4th pharyngeal pouch
119 facial nerve which arch2
2nd brachial arch
st
1 pharyngeal arch
120 muscle involved in opening mouth`1
Lateral pterygoid
Medial pterygoid
Platysma
121 Int laryngral nerve pass to larynx by piercing between2
Between cricoids and thyroid
Between throid and hyoid
Between hyoid and arytenoids
122 lumen of allantois not obliterated1
Urachal fistula
Urachal sinus
Urachal cyst
Mickels diverticulum
123 tear to pelvic diaphragm,muscle involved2
Levator ani muscle
124 fast adapting fibers`1
Pacinian corpuscle
125 baby 8 monhs, difficulty in respiration,releved in prompt up position,resp sound not present on left
lung,investigations show intestinal loops in left thoracic cavity,cause2
Left lung absent
Eventration of diaphragm
Diaphragmatic canal something??
127succinyl cholene toxicity increase by dec1
Pseudocholinesterase
Acetylcholinesterase
128 water dieresis1
th
129 superolateral border of 4 ventricle formed by1
Inf cerebellar peduncle
Superior cerebellar peduncle
Middle cerebellar peduncle
130 tachyphylaxis rapid decrease in drug action
131 Parasympathetic action1
Dec bronchial secretions
Inc heart rate
Decrease intestinal motility
132 median nerve damage2
134 INVESTIGATION OF CHOICE FOR DIABETIC NEPHROPATHY1
Serum creatinine
Urine protein
98

135 lymph from first web space of hand drain to2


Supraclavicular nodes
Infraclavicular nodes
Popliteal nodes
136 cut tendon of biceps.cut extends medially,what will be effected2
Brachial artery,
median nerve
Ulnar nerve
137 anterior dislocation of shoulder,damage2
Anterior circumflex artery
Axillary nerve
138.joint b/w pubic bones allowing slight movement during birth of baby1
Synchondrosis
syndesmosis
Symphysis
139 Two liters Normal saline infusion 1
dec urinary sodium
inc blood volume
dec intracellular volum
140 Fastest speed of impulse in 1
ventricular fibers
AV node
AV bundle
purkinje fibers
141 pt has Gonococcus infection 1
ZN stain
Gram stain
Culture
142which are bipolar neurons 1
posterior root ganglion cells
anterior horn cells
.posterior horn cells
Olfactory cells
143 painful swelling at site of amputation 1
Neuroma
Ganglioneuroma
144.Edema is caused by 1
dec hydrostatic pressure
inc osmotic pressure
blocked lymphatics
=-=-=-=-=-=-=-=-=-=-=-=-=-=-=-=-=-=-=-=-=-=-=-=-=
99

9. Radio June 2015 by Asma Warsi(95)


FCPS-I Radiology Paper-II June 2014
This paper may contain some incomplete options. Please kindly correlate by checking and confirming
with your colleagues. Tried my level best to keep it confine to paper2 only.
Qs 1. Structure present between celiac trunk and superior mesenteric artery is?
A. Pancreas
B. Stomach
C. Spleen
D. Duodenum
E. Liver
Qs 2. Hepatoduodenal ligament transmits
A. Hepatic artery ?
B. Portal vein
C. Common bile duct
D. Hepatic duct
Qs 3. Mixed venous blood is found in
A. Left atrium
B. Left ventricle
C. Pulmonary artery
D. Pulmonary vein
E. Rt femoral vein
Qs 4. The bulb of posterior horn of left ventricle is made by?
100

A. Forceps major
B. Forceps minor
C. Thalamus
Qs 5. Structure passing anterior to head of pancreas
A. Aorta
B. Superior mesenteric vein
C. Superior mesenteric artery
D. Ivc
Qs 6. Mass in porta hepatis will compress
A. Pv
B. Cbd
C. Hepatic artery
Qs 7. Renal artery divisions sequence(none of the option seem to be appropriate or in a sequence)
A. Segmental, lobar, interlobar, arcuate
B. Interlobar arcuate lobar
Please learn the sequence. Cant recall the options
Qs 8. Lower limb
A. Femur and patella are in opposite position except full extension
B. Intrarticular fibrocartilage
Qs 9. Which loops around arch of aorta
101

A. Left recurrent laryngeal nerve


B. Azygous vein
C. Left vagus nerve
D. Subclavian
Qs 10. Serotonin is formed from
A. Platelets
B. Liver
C. Brain
Qs 11. Trigone of urinary bladder
A. Uretric ridges are oblique
B. Found between uretric ridge and urachus
C. Develops from urogenital sinus
Qs 12. Cutaneous sensation loss on dorsum of thumb due to lesion of
A. Musculocutaneous nerve
B. Radial nerve
C. Median nerve
D. Ulnar nerve
E. Axillary nerve
Qs 13. Cataract is
A. Defect in protein synthesis of?
B. Denaturation of proteins
102

Qs 14. Semen
A. Main bulk from seminal vesicular secretion
B. 10ml after several days of abstinence
C. Inc content of glucose
Qs 15. Knee jerk loss due to lesion of
A. L1
B. L2
C. L3
D. L4
E. L5
Qs 16. Most radiosensitive tumor is
A. Lymph node
B. Cartilage
C. Brain
Qs 17. Ureter most constricted at
A. Transverse process
B. Ischial spine
C. Ischial tuberosity
D. Sacroiliac joint? (not sure abt this option)
E. Sacral promontry
103

Qs 18. Extension of hip flexion of knee(asim shoaib bcq)


A. Semitendinous
B. Sartorius
Qs 19. Common peroneal nerve causes dorsiflexion of foot. What else it do?
A. Eversion
B. Inversion
C. Plantar flexion
D. Cutaneous sensation on foot
Qs 20. Which passes through pelvic brim
A. Ureter
B. Uterine artery
C. Urethra
Qs 21. Bile duct has following compartmentAgain none of the option had correct answer
A. Supraduodenal intraduodenal retroduodenal infraduodenal
B. Retroduodenal intraduodenal infraduodenal
C. Infraduodenal retroduodenal paraduodenal
Qs 22. Structure dividing heart into four segments
A. Endocardial cushions
B. Aorticopulmonary segment
Qs 23. Middle rectal vein drains in
104

A. Internal iliac
B. Smv
C. Imv
D. Pv
Qs 24. Complete ossification of (four peices?) of body of sternum occurs at
A. 6mnths
B. 1yr
C. 21yrs
D. 15yrs
Qs 25. Upper body of sternum has
A. Juglar notch can be palpated or start from here
B. Body of sternum and xiphoid fuses here
C. Upper t2 costal cartilage can be found here
D. Lies against t3??
Qs 26. Reticulocytosis
A. Sle
B. Polycythemia
C. Hemolysis
Qs 27. Hemolytic anemia in
A. Bronchiogenic ca
B. Aml
105

C?
Qs 28. A man cut his wrist. Which structures are damaged passing superficial to flexor retinaculum
A. Flexor digitorum superficialis and ulnar artery
B. Ulnar nerve and ulnar artery
C. Median nerve and ulnar nerve
D. Flexor digitorum superficialis and median nerve
Qs 30. At shaft of humerus which is damaged
A. Profunda brachi artery
B. Posterior circumflex humeral artery
C. Anterior humeral artery
Qs 31. Skin of glans penis drains in? (snell review bcq)
A. Superficial inguinal lymph node
B. Internal iliac
Qs 32. Regarding lower limb lymphatic
A. Superficial inguinal drains scrotum
B. Big toe lymph node entrapment causes enlargement of inguinal lymph node
C. Inguinal lymph node causes elephantiasis?
Qs 33. Which opens directly in SVC?
A. Azygous vein
B. Ivc
106

C. Pulmonary vein
D. Brachiocephalic vein
Qs 34. Which valve of heart is commonly diseased? (feb2014 qs paper2)
A. Aortic
B. Mitral
C. Pulmonary
D. Tricuspid
E. Coronory sinus
Qs 35. Aortic aneurysm compresses
A. Thoracic duct
B. Aortic and thoracic duct
C. Esophagus
D. Azygous
Qs 36. Trypsinogen activated by
A. Enterokinase
B. Ph less than 7
C. Ph more than 7
D. Hco3
Qs 37. Thoracic duct
A. Joins aorta at t2
B. Valves
107

C. Drains right limb


D. Ends at right brachiocephalic vein?
Qs 38. Atrophy of right parietal lobe cause
A. Hemianesthesia
B. Right left disorientation
C. Sensory dysphasia
D. Dysdiadokinesia
E. Hemibellismus
Qs 39. Tail of pancreas
A. Lineorenal ligament
B. Hepatorenal ligament
Qs 40. Right renal vein drains in
A. Ivc
B. Aorta
C. Pv
D. Svc
Qs 41. Which connects liver with ventral bodywall?
A. Falciform ligament
B. Hepatoduodenal ligament
Qs 42. Sickle cell anemia
108

A. Hb S factor
B. Hemolysis?
Qs 43. Posterior intercostal are branches of
A. Internal thoracic artery
B. Intercostal artery
C. Thoracic aorta
D. Axillary artery
Qs 44. Which is a direct branch of subclavian artery directly anastomosis around shoulder joint?
A. Suprascapular artery
B. Dorsal scapular artery
C. Transverse cervical artery
D. Subscapular artery
Qs 45. Medial side of breast drains in
A. Internal thoracic
B. Axillary
C. Pectoral
D. Lateral nodes
Qs 46. A man got gunshot injury. He has planters going up, loss of power and no sensations below the
level of lesion. Where is the lesion?
A. Corticobulbar
B. Corticospinal and spinothalamic tract
C. Spinothalamic tract
109

D. Lmn
Qs 47. Neurovascular bundle present in btw
A. Internal and innermost intercostal
B. External and internal intercostal
C. Internal intercostal and endothoracic fascia
Qs 48. Diaphragmatic hernia is due to (asim Shoaib bcq)
A. Incomplete pleuroperitoneal membrane
B. Incomplete pleuropericardial membrane
Qs 49. Dysphagia for liquids due to
A. Neuromuscular incordination
B. Palatal palsy
Qs 50. Primary motor cortex sends fibers directly to
A. Lmn
B. Umn
C. Dorsal column of spinal cord
D. Motor nuclei of spinal cord
E. Anterior horn
Qs 51. Optic radiations are
A. Association fibers
B. Arcuate fibers
110

C. Projection fibers ??
D. Commisural fibers
E. Unmyelinated fibers
Qs 52. Spinal cord
A. Conus medullaris ends at l3 ref RJ Last
B. Spinal nerves are formed 2cm away from ant and post rami
Qs 53. Left gastric artery is a branch of
A. celiac artery
B. SMA
C. Splenic artery
D. Hepatic artery
Qs 54. Ligamentum teres
A. Obliterated umbilical vein
B. Found btw free margins of coronary ligament
Qs 55. A Girl before meal how to measure insulin levels (asim shoaib bcq)
A. blood glucose
B. Circulating glucagon
Qs 56. Internal capsule
A. Band of white fibers
B. Corticobulbar and corticospinal tracts present in post limb
111

C. Optic radiations present in supra lenticular?


D. Supplied by ant spinal artery
Qs 57. Autosomal dominant
A. NF
B. Cystic fibrosis
Qs 58. Mental artery is branch of
A. Inferior alveolar artery (mental branch of inf alveolar nerve. RJ last. I didn’t mark this option but
people say this could be the answer too)
B. Facial artery (submental artery. Ref Clinical Snells)
C. Lingual artery
D. Maxillary artery
Qs 59. Vibrio cholera
A. Asymptomatic
B. Iv and oral antibiotics given?
C. Causes small bowel perforation
D. Bloody diarrheae
E. Transmitted by water and food
Qs 60. Typhoid(rabia ali microbio section bcq)
A. Asymptomatic
B. Careers should be isolated and treated
C. Excrete organism in feces
112

Qs 61. An 8yrs old girl has facial pain and mass in mandible. Cause is
A. Ebv
B. Htlv-1
C. Hhv-8
D. HIV
Qs 62. Structures present anterior to left kidney is
A. Posterior portion of lesser sac
B. Terminal branch of left colic artery
C. Greater sac
D. Duodenum
Qs 63. Wrist drop is caused by lesion of
A. Radial nerve
B. Median nerve
Qs 64. Rectum
A. Complete muscular coat (RJ Last)
B. Has apoendices eppiplocae
C. Post to prostate
Qs 65. Appendix drains in
A. Superior mesenteric lymph nodes
B. Int iliac lymph node
113

C. Inferior mesentric lymph node


D. Celiac
Qs 66. Ivc arises at
A. L2
B. L3
C. L4
D. L5
Qs 67. Bile duct
A. Lies on right of hepatic artery
B. 6-8 cm/mm long? (dont rem)
C. Travels in hepatoduodenal ligament
Qs 68. Between ureter and peritoneum lies
A. Uterine artery
B. Iliac vessels
C. Ureter
D. Gonads
Qs 69. Osteprosis
A. Wide and thin trabecular bones
B. Affects long bones
Qs 70. During hysterectomy which structure is damaged
114

A. Ureter
B. Uterine artery
Qs 71. Nerve supply between anterior superior iliac spine and greater trochanter is
A. Iliohypogastric nerve
B. Lat cutaneous nerve of thigh
C. Superior gluteal nerve ??
D. Femoral nerve
Qs 72. Inguinal canal
A. Present btw ant sup iliac spine and pubic tubercle
B. Absent at birth
C. Iliohypogastric nerve passes through it
D. Conjoint tendon forms superior wall
Qs 73. Cause of infertility in male is
A. Hypospadiasis
B. Klinefilter syndrome
C. Cryptochordism
Qs 74. Hypospadiasis is due to
A. Genital fold
B. Genital tubercle
Qs 75. Circumflex artery supplies
115

A. Rt atrium
B. Left atrium
C. Left atrium and left ventricle
D. Rv and rt atrium
Qs 76. Visceral carotid sheath contains
A. Common carotid artery
B. External carotid artery
C. Internal juglar vein
D. Pharynx?
Qs 77. SLE except (asim shoaib medicine bcq)
A. HLA DR27
Qs 78. Clavipectoral fascia
A. Covers pectoralis minor
B. Suspensory ligament suspends it from chest wall
Qs 79. Geniohyoid muscle supplied by
A. Ansa cervicalis
B. C1
C. Facial nerve
Qs 80. Part of levator ani
A. Puborectalis
B. Rectococcygeal
116

Qs 81. Deep inguinal ring formed by


A. Facsia transversalis
B. Fascia iliaca
C. External oblique aponeurosis
D. Internal oblique
Qs 82. Arterial supply of prostate
A. Inferior vesicle artery
B. Internal iliac artery
C. Median sacral artery
Qs 83. In pregnancy what is increased
A. Serum ferritin
B. Serum transferin
C. Iron absorption from gut
D. Redcell mass (maybe the answer. Mentioned in GOLJAN)
E. MCH
Qs 84. Not attached to 12th rib
A. Psoas
B. Lateral arcuate ligament ??
C. Diaphragm
D. Quadratus lumborum
117

Qs 85. Right gastric artery is branch of


A. Celiac artery
B. Hepatic artery
Qs 86. Intraarticular muscle is (rabia ali bcq)
A. Semitendinous
B. Poplitues
C. Sartorius
Qs 87. Which one is not correlated
A. Leukemia and cyclophosphamide
B. Benzene and leukemia
Qs 88. Nasal polyposis least likely
A. Infection
B. Vasomotor instability
C. Autoimmune ?
D. Allergy
Qs 89. Which has both intra and extrauterine extension
A. Round ligament of uterus
B. Round ligament of ovary
Qs 90. Lumbar puncture which is pierced
A. Dura mater
118

B. Pia mater
C. Post ligament
Qs 91. Protrusion of jaw along with medial pterygoid is done by
A. Lateral pterygoid
B. Omohyoid
C. Geniohyoid
D. Temporalis
Qs 92. Filum terminale
A. Ends at s2?
B. ?
Qs 93. Burnt skin heals quickly due to
A. Granulation tissue
B. Underlying connective tissue intact
C.skin appendeges intact?
Qs 94. Calcitonin is released in tumor condition from
A. Thyroid
B. Parathyroid
Qs 95. Erythropoeisis at 4th month is done in
119

A. Liver B. Yolk sac C. Spleen D. Bone marrow

=-=-=-=-=-=-=-=-=-=-=-=-=-=-=-=-=-==-=-=-=-=-=-=-=-=-=-=-=-=-=-=-=-=-==-=-=-=-=-=-=-=-=-=-=-=-=-=-=-=-=-=

10.Ophthalmology 31st Dec-2015 by Anita Malik(85)


OPTHALMOLOGY 31ST DECEMBER 2015 MIX PAPER1 AND 2 by Anita Malik

Q1.DIAGNOSIS OF WIDAL TEST:

A,1:100 OF O ANTIGEN

B1:160 OF O AND H ANTIGEN

C.1:160 OF O ANTIGEN

Q2.LATERAL VENTRICLES SEPARATED BY:

ANS SEPTUM PELLUCIDUM

Q3.HLD-DR4

ANS RHEUMATOID ARTHTITIS

Q4.A MAN RECEIVED HEAD TRAUMA AFTER FEW DAYS DEVELOPED GRADUAL LOSS OF LEFT
SIDE OF VISION.. WHERE IS THE LESION?

A.SUPERIOR CEREBELLAR PEDUNCLE

B.INFERIOR CEREBELLAR PEDUNCLE

C.RIGHT ANGULAR GYRUS (ANS??? NOT SURE)

D.LEFT ANGULAR GYRUS

Q5.HORMONE RECEIVING INHIBITORY STIMULUS FROM HYPOTHALAMUS?

ANS PROLACTIN

Q6.A PATIENT HAD INJURY TO FACE PRESENTS WITH DISHED IN FACE DIPLOPLIA (SOME
SCENARIO LIKE THS)...FRACTURE TO WHICH BONE?

ANS MAXILLA

Q7A PATIENT PRESENTS WITH DIPLOPIA, ON LOOKING DOWN HIS EYE MOVES TOWARDS
MIDLINE, LESION OF WHICH NERVE
120

A.INFERIOR DIVISION OF OCCULOMOTOR

B.TROCHLEAR

C.SUPERIOR DIVISION OF OCCULOMOTOR

D.ABDUSCENT

Q8.REDUCTION IN BLOOD SUPPLY TO BRAIN OCCURS IN WHCH CONDITION?

A.SEIZURE

B.INTRAVENTRICULAR INJECTION OF NOREPINEPHRINE

C.HYPERBARIC O2

Q9.PATIENT EXPOSED TO HYPERBARIC O2 FOR SOME TIME WILL DEVELOP?

ANSSPONTANEOUS PNEUMOTHORAX?? DONT REMEMBER OTHER OPTIONS

Q10.LOCAL ANESTHETIC WITH SHORTEST DURATION OF ACTIOM?

ANS BUPIVACAINE?

Q11.BRADYCARDIA IS SIDE EFFECT OF WHICH ANESTHETIC AGENT?

A.KETAMINE

B.HALOTHANE

C.NO2

Q12.PRETERM BABY HAVING SURFACTANT DEFICIENCY WILL HAVE?

A.DECREASE COMPLIANCE INCREASE PRESSURE

B.INCREASE COMPLIANCE DECREASE PRESSURE

Q13.REGARDING ORBICULARIS OCULI?

A.OPENS EYE

B.ATTACHED TO LATERAL RAPHE

Q14.AT 5TH WEEK OF EMBRYOGENESIS?

ANS.GUT ROTATION
121

Q15.CHANGE OF EPOTHELIUM AT ANDOCERVICAL JUNCTION?

ANS.METAPLASIA

Q16.MOST COMMON SIDE EFFECT OF CHOLINERGIC DRUGS?

A.BRADYCARDIA

B.SALIVATION

Q17.2YR OLD CHILD WITH FEVER ARTHRALGIA BLINDNESS...DX?

A.JUVENILE ARTHRITIS

B.HENOCH SCHONLEN PURPURA

Q18.CONJUNCTIVITIS INVOLVING SYSTEMIC DISEASE IN WHICH CONDITION

A.VERNAL KERATOCONJUNCTIVITIS

B.HAY FEVER

C.ATOPIC CONJUNCTIVITIS

Q19.YOUNG PT A HIGH ALTITUDE SUDDENLY DEVELOPS CHEST PAIN AND LENS DISLOCATION...
CAUSE OF CHEST PAIN?

A,MARFAN

B.CARDIOPULMONARY FAILURE

C.AORTIC ANEURYSM

D.MI

Q20.CILIRAY BODY DERIVED FROM?

A.NEUROECTODERM ALONE

B.MESODERM ALONE

C.NEURO AND MESO

Q21.CILIARY BODY FEATURE (SOMETHING LIKE THT)


122

ANS.2 LAYER OF EPITHELIUM

Q22. PREGNANT LADY HB9, CAUSE OF ANEMIA

ANS.PLASMA VOLUME EXPANSION

Q23.EXCRUTIATING CHEST PAIN, CAUSE

ANS AORTIC ANEURYSM

Q24.COMMON CAUSE OF HEPATOCELULAR CA?

ANS.HEP C

Q25.LIVER ATTACHED TO PERITONEUM BY?

ANS:IVC

Q26.A SCENARIO OF MENSTRUAL DISTURBANCE, COLD INTOLERANCE.....DX?

ANS.HYPOTHYROIDISM??

Q27.A NAKED PT LYING IN A ROOM WITH TEMP 21C, HUMIDITY 80, MECHANISM OF HEAT
LOSS?

ANS.RADIATION AND CONDUCTION

Q28.ACUTE INFLAMMATORY CELLS IN BLOOD?

ANS.NEUTROPHILS

Q29.ANTICANCER DRUG REGRESS CANCER SIZE BY WHICH MECHANISM?

A.ATROPHY

B.APOPTOSIS

C.INFARCTION

Q30.A PT WITH MULTIPLE FRACTURE, BP90/70, INITIAL STEP?

ANS.VOLUME REPLACEMENT

Q31.OCCULOCARDIAC REFLEX INITIATED, WHAT WOULD BE INITIAL STEP TO REVERSE THE


STIMULUS?

A.IV ATROPINE
123

B.STOP THE STIMULUS

Q32.GLABELLA IS PART OF WHICH BOME?

ANS.FRONTAL BONE

Q33.NEOSTIGMINE PREFERRED IN WHICH DISEASE?

ANS.MYASTHENIA GRAVIS

Q34.MECHANISM OF NEOSTIGMINE?

ANS.INCREASE ACH AT END PLATE

Q35.DEAD SPACE DECREASE IN

A.TRACHEOSTOMY

B.SHALLOW BREATHING

C.STANDING

Q36.MOST SENSITIVE TO RADIOTHERAPY?

ANS.LYMPH NODE.

Q37.BASE OF MIDDLE CRANIAL FOSSA FORMED BY?

A.GREATER WING OF SPHENOID

B.PETROUS PART OF TEMPORAL BONE

Q38.Bunch of golden yellow colonies ...which organism?

Ans.staph aureus?

Q39.visual and auditory stimulus combine at?

A.Wenicke’s area

b.broca’s area

forgot other options

Q40.number of capillary fenestrations are highest at?

a.peripapillary area
124

b.ora serrata

c.equator

Q41.A pt with head trauma presents with floater (a scenario like that)

A.PVD

B.Hyphema

C.Hypopyon

Q42.a pt with head trauma complaints of loss of vision that becomes normal after 1 day?

a.vit hemorrhage

b.retinal detachment

c.macular edema

Q43.A pt presents with nystagmus, neurological examination was normal, lesion site?

Ans.cerebellar lesion?

Q44.Volume of aqueous humor?

Ans0.25-0.3

Q45.Retina thickest at?

Forgot options 

Q46.Blood supply of rods and cones?

Ans.Chorioapillaries

Q47.regarding optic chiasm?

Ans.junction of 2 nerves

Q48.Skin burnt, will heal itself by?

Ans.presence of granulation tissue

Q49.Identical twins?

Ans.graft not rejected even without immunosuppresion


125

Q50.Increase pulse pressure due to?

A.hypertension

B.increase cardiac output

Q51.Jugulodiagastric lymph node drains?

Ans.tongue and palatine tonsil

Q52.Parasympathetic supply to sphincter papillae

Ans.short ciliary nerve

Q53.Increase heart rate decrease?

Ans.end systolic volume

Q54.Clostridium tetani?

a.exotoxin has lethal effect

b.exotoxin effect cerebral cortex

Q55.ALL of the foolowing are autosomal dominant except

a.marfan

b.arachnodactyly

c.alpha1 antitypsin deficiency

Q56.In pneumothorax?

Ans.lung collapse and chest wall spring out

Q57.Cells per square mm in bowman’s layer?

Dont remember options 

Q57.extradural hematoma?

Ans trauma to middle meningeal artery

Q58.collagen?

Ans.lightly stauns with eosin


126

Q59.presence of normal tissue of body at location oher than its primary location?

Ans.choristoma

Q60.Lymphoid nodules are not present in?

Ans.thymus

Q61.pregnant lady consumed alcohol during pregnancy, in baby which organ would be likely
affected?

And.brain

Q62.GFR estimated clinically by?

Ans.creatinine clearance

Q63.Gastrin stimulates acid secretion by?

Ans by inhibiting both hcl and vagally mediated acid secretion

Q64.Confirmatory test of myasthenia gravis?

a.emg

b.tensilon test

c.antibodies to ach receptors

Q65.Complete oxidation of glucose produc total ATP?

ANS.38

Q66.Pretracheal fascia infection spreads to:

a.anterior mediastinum

b.posterior mediastinum

c.middle mediastinum

Q67.secondary centre of ossification?

ANS.EPIPHYSIS

Q68.major stress hormone?


127

ANS.ACTH

Q69.A population was taken for a study, it was subdivided into groups and further subgroups
were formed. Which type of study is this?

ANS.RANDOM STRATIFIED SAMPLING

Q70.A child with bowel disturbance, deficiency of vitamin B12, investigation to be done?

ANS.INTRINSIC FACTOR ANTIBIODY

Q71.A patient with chest pain, Ecg shows anterior wall MI, artery involved?

ANS.LAD

Q72.Oblique fissure?

ANS.T3 to 6TH costal cartilage

Q73.Epidural space widest at?

ANS.L2

Q74.Tumor with nerve extension?

ANS.CARCINOMA PLEOMORPHIC EXADENOMA

Q75.4TH HEART sound?

ANS.VIBRATION OF VENTRICULAR WALL IN ATRIAL SYSTOLE

Q76.Area for auscultation of tricuspid valve?

ANS.RIGHT LOWER STERNAL BORDER

Q77.PYOGENIC PERITONITIS

A.Bacteroids

B.E.Coli

Q78.Energy utilized in which order?

ANS.CHO, FAT, PROTEIN

Q79.Alopecia after TPN,deficiency of?


128

ANS.ZINC

Q80.Imp antioxidant?

ANS.GLUTATHIONE

Q81.Concave structure around nuclues with cisternae ?

a.golgi apparatus

b.rer

Q82.Peroxisomes?

ANS.CATABOLISM OF LONG CHANGE FATTY ACID

Q83. 2*2 TABLE?

ANS CHI SQUARE

Q84.Most commonly injured nerve during thyroidectomy?

ANS.EXTERNAL LARYNGEAL NERVE

Q85.Patient with DKA managed per died, cause?

ANS.MUCOR

=-=-=-=-=-=-=-=-=-=-=-=-=-=-=-=-=-==-=-=-=-=-=-=-=-=-=-=-=-=-=-=-=-=-==-=-=-=-=-=-=-=-=-=-=-=-=-=-=-=-=-=

11.Psychiatry (conventional) November 2015 by Dr.Rabeea Iesar.(100)


IN THE NAME OF ALLAH THE MOST GRACIOUS THE MOST MERCIFUL

PSYCHIATRY CONVENTIONAL 2015

Paper 1 neuro mcqs+paer 2

1.Cultural syndrome LATAH

In middle aged women


129

2.a child realizes that his mother is distict from him and can correlate

Personal identification phase?

3.alcohol is

Depressant

4.a youngboy 19 years old having mood changes ,treatment

LITHIUM

5.drug causin maniac elation

Chlorpromazine

6.buspirone hydrochloride

Non benzodiazepine anxiolytic

7.young man ,psychosis

Olanzapine

8.regardin NE

Most postganlionic sympathetic fibers secrete

9.regarding botulinum toxin

Respiratory muscle paralysis

10.hyperkalemia

Suxamethonium given in burn patient

11. pt. on antidepressant for 10 years,from 2 years drug is changed due to some untolerable effects,still he is
disturb due to unusual increase in movement,you should give following drug combine with the main treatment
regimen

Anticholinergic

12.which receptors block along with synaptic facilitation of amines…..

*Cholinergic?

13.sailor,southeast asia recent trip,dementia like features

*hiv related dementia

14.different type of EEG rhythms with background slow rhythm shown on strip.patient is a53 year old man with
profound dementia,myoclonus,died of bronchopulmonary pneumonia,autopsy shows spongiformencephalopathy
130

Your diagnosis

Creutzfeld Jacob

Hiv encephalopathy

Herpes simplex encephalitis

Alzheimer

15.proressive failing recent memeory

Alzheimer

16.neurofibrillary tingles

Alzheimer

17 12 years old .child eeg shows 3 sikes per min ,treatment should be

Carbamazepine

Phenytoin

Na valproate

18. 10 year old child with myoclonic seizures,treatment should be

Phenytoin

Carbamazepine

19.most teratogenic

Lithium carbonate

20.na=155 meqL

K=3

Ph=7.45

Most common cause

Increase aldosterone

21.most important regulator of na

ADH

ANP

Aldosterone
131

22.dec.anion gap

Metabolic alkalosis

Hyperaldosteronism

23.metabolic alkalosis

Intestinal fistula

24.which hormone inhibit gonadotrophin release from pituitary and increase in night

Melatonin

Prolactin

27.dmage to which structure

If t4 increase,t3 increase,tsh decrease but markedly increase on giving trh,patient with palpitation and sweating?

Thyroid

Pituitary

Hypothalamus

28.thalamus send axon to

Tapetum

Tectum

Medial geniculate body

29.trigeminothalamic pathway ascend to

Medial geniculate body??

30.all other pathways ascend from thalamus, end in

VPL

SOMATOSENSORY AREA OF CONTRALATERAL SIDE OF THALAMUS

31.VENTRAL SPINOTHALAMIC tract

Itch/ticle

Temp

Pain

Proprio
132

Vibration

32.dorsal column

Crude touch

Proprio

Vibration

33.which structure damage when there will be no motor,sensory loss,also reflexes intact

Broadmann area4

Vestibulospinal pathway

Rubrospinal

Spinothalamic

34.taste sensation in cerebral cortex supply by

35.foot area in cerebral cortex supply by

36.which area is responsible for conversion of written words into audible form

Broca

Wernike

Angular gyrus

37.administration of which drug would lead to pupillary dilatation and vessel contraction

Adrenaline

Salbutamoll

38.neurotransmitter released when

Ca release presynaptically

Ligand gated calcium channel open

39.REM SLEEP IS DECREASED BY

No drug

40.TREATMENT OF withdrawl of antipsychotic

Benztropine

41.as day passes decrease in activity increases,ptosis occurs


133

There will be antibody against ca channel receptors

42.neuromodulators

Release at post synaptic membrane to modulate response

Relee at post synaptic membrane

Release at synapse

Directly pour into blood

43.calcitonin

132 molecule

In Pagets disease cannot given

Increase calcium absorption in gastrointestinal tract

Can be iv administered

Release by pituitary

44.deficiency of dopamine result in

Parkinsonism

45.parkinsonism caused by dopamine deficiency of which precursor molecule

Tryptophan

Tyrosine

Histidine

Cystedine

46.carbidopa inhibit peripheral conversion of dopamine due to

Dopa decarboxylase

47.Dopa destroyed by which enzyme

48.barbiturates are

Enzyme inducers

49.which drug affect other drugs metabolism

Phenobarbitone

Alcohol
134

Aspirine

Respiredone

50.common in delirium and dementia

Loss of consciousness

Loss of intelligence

Loss of recent memory


th
Dilatation of 4 ventricle

51.old age delirium,dementia,ataxia,urinary retention

NPH

52.septum pellucidum

Fornix>ant.commisure>corpus callosum

Ant.commisure>fornix>thalamus

53.MGB

Attatched to main mass of thalamus

54.difficult return of sensation and emotion lesion in

Thalamus

55.pregnant lady

Propylthiouracil

56.back is supplied by

Dorsal rami

57.segmental fragmentation

Myelin degenerate at node of ranvier

58.sensation from face and taste sensation carried to thalamus

MGB

59.cerebellum

Spastic paralysis

60.left side pain and temp loss,rt.side dorsal column


135

Hemisection rt.

61. 40 years old,.profound memeory loss,painful vision loss,family history positive,chorea,athetosis,

Huntington

62.regarding pain,true is

Only stimulate nociceptive ending

Stimulate other endings as well

63.cells in cns like schwann cells in pns

Oligodendrocytes

64.mature defence is

ALTRUISM

65.pill rolling tremors

Parkinsonism

66.young patient tremor

Propranolol

67.priapism is caused by trazadone

68.which stimulate Ne RELEASE

AMPHTAMINE

69.PT. DIABETIC FUNDOSCOPY REVEALS COTTON WOOL SPOTS CAUSE IS

Dm

70.PT. DYSPNOIC ON LYING DOWN

RETROSTERNAL GOITRE

71.pt. NIDDM,surgery of abdomen,bed ridden,dyspnoic

Pulmonary embolism

72.decrease response to Achilles tendon reflex

Hypothyroidism

73.multiple fractures

Osteoporosis
136

74.cushing syndrome

Neutropenia

75.true hermaphrodite

XX/XY

76.stress hormone

ACTH,cortisol not given in options

77.true about thyroid hormone preparation

Iodine diffusion

78.most common cause of mental retardation in new born

Alcohol

79.recent memory loss

Amygdyla

80.alcohol lead to

Degeneration of mammillary body

81.APKD

Association with cerebral hemmorhage

82.sub dural hemmorhage

Sub dural veins

83.vagus nerve

Dorsal motor

Nucleus

84.rt. optic tract

Left homonymous hemianopia

85.pituitary tumor growing upward,

Bitemporal hemianopia

86.craniopharyngioma

Bitemporal hemianopia
137

87.decrease GABA

Increase movement

88.lesion in huntington in

Caudate nucleus

89.trinucleotide repeat

Huntington

90.No somatosensory loss with hemiparesis

Ant.part of post limb of anternal capsule

91.direction of hearing identified by

Inf. Colliculus

92.teeth grinding

Bruxism

93.imprinting

In birds

94.facial nerve

Greater petrosal nerve in temporal bone

95.physical dependence

Morphine

96.Regarding smaller unmyelinated axons and larger myelinated axons different is

Large are

*fast conduction velocity

*increase internal resistance per length

*decrease membrane resistance per cm2

97.cannot initiate movement,cannot stand from sitting position,lesion in?

98.sensation from body and limbs to thalamus

VPL??

99.NOTOCHORD
138

INDUCE OVERLYING ectoderm to form neural plate

100.wernike encephalopathy.

Good concepts of following required:

A lot of questions were from MEDIAL GENICULATE BODY

LATERAL GENICULATE BODY

VENTROPOSTEROMEDIAL TRACT

VENTROLATERAL TRACT

PAPER 1

2-3 questions were from

MUSCLE SPINDLE

GOLGI BODY

Remember me in prayers.

Dr.Rabeea Iesar.

=-=-=-=-=-=-=-=-=-=-=-=-=-=-=-=-=-==-=-=-=-=-=-=-=-=-=-=-=-=-=-=-=-=-==-=-=-=-=-=-=-=-=-=-=-=-=-=-=-=-=-=

12.Psychiatry August 2015 by Dr.amjad. Compiled by :Dr Rabeea Iesar.(72)


IN THE NAME OF ALLAH THE MOST GRACIOUS THE MOST MERCIFUL.

Psychiatry august 2015 online:

1.regarding ach synthesis

From post ganglionic sympatheticsto sweat glands

2.true regarding norepinephrine


139

Present in most of post ganglionic sympathetic.

3.which neurotransmitter is present in most post ganglionic sympathetics.

Norepi

4.olfactory sensation

Soluble in both water and lipid

5.regarding Wallerian degeneration

Axonal fragmentation

6.schwann cels for pns and for cns

Oligodendrites

7.subdural hematoma structure involved

Bridging veins

8.ataxia,dementia,urinary incontinence

NPH

9.vibration carried by dorsal column

10.temperature carried by spinothalamic

11.which tract carry sharp pain,

Lat.spinothalamic

12.organ of corti is frequency analyser.

13.athetosis and corea combination in

Huntington

14.thalamus to globus pallidus

Direct relation.

15.which structure damage in huntington?

16.bradykinesia,resting tremor,rigidity>parkinsonism.

17.left homonymous hemianopia>rt.optic tract

18.pituitary tumor>bitemporal hemianopia

19.loss of motor functions which structure involved


140

Genu+ant.part of post.limb.

20.reloxifen >in breast cancer.

21.confabulation,ataxia,nystagmus>wenikes encephalopathy.

22.sensory ataxia>thalamus involved.

23.trigeminothalamic tract>VPM nucleus.

24.feeding center(VPM)…..OPTION OF VPL not given.actual answer should be VPL.

25.imprinting>in birds only.

26.cultural syndrome ‘’latah’’>common in middle aged women

27.physical dependence caused by>heroine

28.stimulant for aldosterone>hyperkalemia

29.common in alzheimer and frontotemporal dementia>loss of memory

30.teratogenic>lithium

31.enkephalins>basal ganglia

32.endorphins>hypothalamus.

33.aphasia>temporal lobe

34.heart shaped vertebrae>thoracic

35.internal capsule>carries somatosensory fibers in anterior limb.

36.clozapine

Agranulocytosis is side effect.

37.neuroleptic malignant syndrome

Occurs when haloperidol used with anaesthesia.

38.tardive dyskinesia>haloperidol

39.hypothalamus lesion has no effect on memory.

40.oldman maniac depression,receive lithium,which is not side effect>hypothyroidism

41.unilateral hearing loss>organ of corti

42.oldman,saw tooth EEG,sleep pathology>REM

43.lesion of thalamus>hyperesthesia.
141

44.myasthenia gravis>neostigmine

45.most common cause of dementia in elderly>ALZHEIMER

46.pituitary adenoma>bitemporal hemianopia

47.nocturnal enuresis>imipramine

48.wenike enceph all occurs except>confabulation(occours in Wernicke psychosis)

49.tourette syndrome>haloperidol

50.regarding agoraphobia>defined as fear of open places.

51.regarding kluver bucy syndrome all occurs except>rage attacks

52.mydrisis occurs due to lesion of>third nerve

53.during sleep all raised EXCEPT

Melatonin,prolactin,ach,GH,serotonin(increase)

Testosterone,TSH,dopamine(decrease)

54.structure destroyed in chronic alcoholics>mammillary body

55.all sensations via thalamus except>visual

56.all interact with monoamine oxidase inhibitors except;

Morphine

57.beta blocker with intrinsic sympathomimetic activity


pindolol

58.regarding restmann syndrome,all true except>lesion in non dominant parietal lobe(actual lesion occurs in
dominant parietal laobe)

59.not a feature of UMN

Pendular knee jerk

60.center of rem sleep

Pons

61.suxamethonium caused hyperkalemia

62.priapism caused by

TRAZADONE

63.secreted from ant.pituitary>prolactin


142

64.pregnancy and hyperthyroidism,drog og choice>propylthiouracil

65.phenobarbitone>enzyme inducer

66.35 years old man ,sailor by profession,had a vsist of southeast asia few days before,her wife started noticing
that he forgets more than usual,a state of dementia established,what is the most usual probabiliyty

HIV –RELATED DEMENTIA

67.dementia most commonly occurs in >ALZHEIMER

68.true about septum pellucidum (SEQUENCE)

Fornix>ant.commisure>thalamus

69.fundoscopy of diabetic reveals soft cotton wool spots,most probable cause is>DM

70.middle age teacher,tremors,drug of choice>propranolol

71.medial geniculate body>attatched to main mass of thalamus

72.carbidopa inhibits dopamine formation in periphery acting as>dopa decarboxylase.

Courtesy by Dr.amjad.

Compiled by :Dr Rabeea Iesar.

Remember in prayers.

=-=-=-=-=-=-=-=-=-=-=-=-=-=-=-=-=-==-=-=-=-=-=-=-=-=-=-=-=-=-=-=-=-=-==-=-=-=-=-=-=-=-=-=-=-=-=-=-=-=-=-=
143

13.ENT 18th November 2015 by Zehra Aqeel Nizami(59)

Some paper 2 ent questions that I could recall!


1-Blood supply of trachea
Internal carotid
Sup thyroid inf thyroid
Subclavian

2-Nerve supply of ant part of respiratory mucosa on lat wall of nose

3-Acessory meningeal artery arises ffrom?


Foramen cecum
Foramen rotundum
Foramen spinosum
Foramen ovale

4-Nerve supply of tip of the nose from branches of?


Maxillary nerve
Opthalmic nerve
Facial nerve

5-Decompression of orbit done in thryoid opthalmopathy by removing lateral wall! Content of


eye herniate in which space?
Infratemporal fossa
Temporal fossa
Pterygopalatine fossa

6-Content in intimate relation with sphenoid sinus


Internal carotid
Trigeminal
Dnt remember other options

7-Sinuses present at birth


Sphenoid frontal
EThmoid maxillary
Maxillary sphenoid

8-About maxillary sinus


Floor is formed by alveolar part of maxillary bone
Opening lies low in the medial wall
Develops from sphenoid sinus

9-Opening of maxillary sinus


In hiatus semilunaris
In middle meatus
In sphenoid recess
144

10-About external auditory meatus


Directed downward and backward
Post wall is supplied by facial nerve
Contains ceruminous glands which secrete brown wax

11-About incus
Smallest bone of the body
Attaches to handle of malleus
Lies in mesotympanum
Lies in attic

12-One more about incus


Has a big body and two processes or limbs
Lies posteriorly and Attaches to post
wall

23-1gram lignocaine in 2% solution, volume would be?


50ml
100ml
250ml

24-Mother brought her child with a language problem, he speaks well and has a good auditory
perception but conjure words, lesion in?
Angular gyrus
Arcuate fasciculus
Hessle gyrus

25One more on conduction aphasia can't recall

26-Patient with rhinosinusitis, endothelial invasion


Aspergillis
Mucor
Rhizopus

27-Lymph drainage of ant part of floor of the mouth


Submental
Submandibular
Parotid
Deep cervical

28-Structure most likely to be injured in the lesion at ligamentum arteriosum?


Left Recurrent laryngeal
Right recurent
Vagus
145

29-Patient with wheezes on examination multiple hilar nodular lesions


Invasive aspergillosis
Non invasive aspergillosis
others options can't recall

30-Rocky mountain spotted fever caused by ?


Ricketsia ricketsi
Ricketsia prowazeki
Ricketsia 2 other types as well
Coxsackie

31-Shortest incubation period


Measles
Rubela
Influenza
Mumps

32-Injury to semicircular canals would cause


Tendency to Fall on the same side
Can't recall other options

33-During spin, eye moves quickly to the left ,this fast movement of eye is called?
Nystygmus
Post rotatory nystygmus?

34-On rotatory movement to the right


Right medial rectus contracts in whole of the movement
Left medial rectus contracts in whole of the movement
Right horizontal semicircular canals depolarize and send afferents to cortex

35-Patient with rhinosinusitis, causative organism?


Hemophilus influenzae
Pnemococcus

36-Impedence mechanism is responsible for


Transference of sound pressure
Transfer of sound waves to internal ear

37-Hot sauses activates which types of receptors on tongue?


Nociceptic?
Thermal receptors?
Touch receptors

38-Taste sensations travel to cortex via


Thalamus?
146

39-Patient with adrenalectomy threshold of which taste would increase?


Salty
Bitter
Sweet

40-Patient with bilateral glosopharyngeal damage which taste sensation would be lost?
Sweet
Bitter
Salty
Sour

41-Auditory stimulus used in brain evoked potential


Clicks
Speech

43-Vocal cords are involved in


Consonants
Resonance
Vowels and pitch of the sound

44-Masking is done in?


Only above 60db
At 45db
At 50db

45-Aryepiglottic folds serves to close the inlet of larynx in


Straining
Weight lifting
Swallowing

46-Superficial laryngeal nerve enters the larynx thru?


Thyrohyoid membrane

47-One question on anatomical dead space


Values were given alveolar ventilation is 6000 VT is 600 tell the anatomical dead space?
150ml
250ml
100ml

48-Patient with HB 6 platelets count elevates hyperplastic bone marrow diagnosis?


Acute leukaemia?
Hemolytic anemia

49-In lactation which nsaid is appropriate?


Ibuprofen
147

Neproxen
Piroxicam

50-Kalman syndrome
Lesion in Arcuate nucleus
Kal1 gene on y chromosome

51- posterior fibers of temporalis muscle causes?


Protrusion
Retrusion
Opening
Closing

52- lateral pterygoid and digastric muscles both bellies contraction would cause?
Protrusion
Opening
Closing
Retraction

53- Which antibody is responsible for myesthenia gravis?


IgM
IgG
IgE
IgD

54- about keiselbech plexus


Formed in the anterior part of the nose in Little's area
Frequent site of bleeding in hypertensive patients

55- muscle that keeps eustacian tube open


Tensor tympani
Tensor vali palatini
Palatooharyngeus

56- nerve supply correctly paired


Palatopharyngeus- pharyngeal plexus
Salphingopharyngeus- glosopharyngeal
Stylopharyngeus- pharyngeal plexus

57- about parotid gland


Duct pierces masseter and opens near incisor teeth
Has no relation with pharynx
Has a thin loosely attached fascia
148

58- salivary calculi present most commonly in?


Warthins duct of submandibular
Stenson's duct of parotid

59- otoconia
Produced by precipitation of calcium on a substance(that u dnt remember)
Supporting cells

=-=-=-=-=-=-=-=-=-=-=-=-=-=-=-=-=-==-=-=-=-=-=-=-=-=-=-=-=-=-=-=-=-=-==-=-=-=-=-=-=-=-=-=-=-=-=-=-=-=-=-=

14. Psychiatry 16th August 2015 by Dr Amjad Dahani(47)


Psychiatry 16 august 2015 Paper 2
1.regarding acetylcholine synthesis
Ans..from postganglionic sympathetic to sweat glands

2.true regarding norepinephrine


Ans.. A.present in all postganglionic sympathetic
B.present in most of postganglionic sympathetic (ans)

3.which neurotransmitter is synthesized from postganglionic sympathetic


Ans...norepinephrine Note ...these three questions were there

4.olfactory sensations
A.soluble in water
B.lipid soluble
C..both lipid and water soluble (ans)

5...regarding wallerian degeneration


A.axotomy
B...axonal fragmentation(ans)

6..Schwann cells for PNS and for CNS


A.astrocytes
B.microglia
C.oligodendrocytes (ans)

7...scenario on subdural hematoma structure involved


A..great veins
B.Middle meningial artery
C.bridging veins(ans)

8..scenario symptoms given ataxia,dementia,urine incontinence


Ans...Normal pressure hydrocephalus

9..vibration by
A.spinothallamic tract
B.dorsal column(ans)

10..hottness carried by
149

A.dorsal column
B.spinothalamic(ans

11..structure carrying sharp pain


A.ant spinothalamic
B.dorsal column
C.lat spinothalamic(ans)

12..frequency analyser
Ans..organ of corti

13..scnerio features given athetosis,chorea


Ans..Huntington disease

14...Huntington disease structure damaged


Ans..caudate nucleus

15...thalamus to globus pallidus


Ans..direct relation

16..scenerio given features bradykinesia, Resting tremors,rigidity


Ans...Parkinson's disease

17..left homonomous heminopia structure damaged


A.optic chiasm
B.optic nerve
C.optic tract left (ans)

18..scenerio on pituitary tumor ,,mostly cause


A..homonomous heminopia
B..bitemporal hemenopia(ans)

19...bitemporal heminopia caused by involving


Ans..pituitary These were different two questions

20..loss of motor functions,,which structure involved


A.anterior limb
B.posterior limb
C.genu D.ant part of post limb (ans)

21..Reloxifen,given in
A..ovarian cancer
B..breast cancer(ans)

22..confabulations, Ataxia,nystgmus ( Wernicke)


Ans..alcohol

23..regarding trigeminothalamic tract


A..VPM(ans)
B..VPL
C..MGB
150

24..sensory ataxia,,structure involved


A.hypothalamus
B.thalamus (ans)

25..feeding centre(lateral hypothalamus not given)


A..anterior hypothalamus
B..post hypothalamus
C.VPM(ans)

26..Imprinting
A..psychological problem
B. in birds (ans)
C.learning method

27..cultural syndrome Latah


A.occurs in middle aged women
B.severe disorder

28..stimulant for aldosterone


A.hypokalemia
B.acidosis
C.hyperkalemia(ans)

29.same feature of Alzheimer disease and fronto temporal dementia


A..loss of confidence
B..loss of memory (ans)

30..physical dependence caused by usually


A.cocaine
B.alcohol
C.heroine(ans)

31..drug which is teratogenic


A..diazepam
B..fluphenazine
C..lithium (ans)

32..carbidopa inhibits dopamine formation by levadopa ,,acting as


A.monoamine oxidase
B.dopa decarboxylase (ans)

33..medial geniculate body


A.lateromedially to thalamus
B.not related to thalamus
C.attached to main mass of thalamus (ans)

34..teacher feel tremors during writing,,drug should be given


A..diazepam
B..methyldopa
C..propranolol (ans)

35..child during sleep grinds teeth,condition called


151

A.somnabulism
B.narcolepsy
C..bruxism (ans)

36..diabetic patient has soft woolly spots in eyes,,cause


A.diabetes mellitus(ans)
B..retinal detachment
C..vessel occlusion

37..septum pellucidum true is


Ans..fornix+anterior commissure+ thalamus

38..drugs inducing rapid eye movement sleep


A.benzodiazepines
B..barbiturates
C..hypnotics
D..no drug can do it(ans)

39..scenario features like dementia given


Ans..Alzheimer disease

40..person brought his 35 year old son with history of dementia,,patient came after a visit from south Asia..most
likely he has
Ans..HIV related dementia

41..phenobarbitone
Ans..liver enzyme inducer

42..pregnant lady safest drug


Ans..propylthiouracil

43..lady with 22 weeks gestation,,has tremors,,cousin has same problem,,drug should be given
Ans..propylthiouracil

44..secreted from anterior pituitary


Ans..prolactin

45..drug causing priapism


Ans..trazadone

46..hyperkalemia caused by
A..furosemide
B..suxamethonium
C.severe burns

47..tricyclic antidepressants ,,also effect receptors


A.beta1
B..betA2
C.alpha (ans)

Regards ...dr Amjad Dahani (psychiatry Larkana)


152

=-=-=-=-=-=-=-=-=-=-=-=-=-=-=-=-=-==-=-=-=-=-=-=-=-=-=-=-=-=-=-=-=-=-==-=-=-=-=-=-=-=-=-=-=-=-=-=-=-=-=-=

=-=-=-=-=-=-=-=-=-=-=-=-=-=-=-2014=-=-=-=-=-=-=-=-=-=-=-=-=-=-=-=-=-=-=-=
1. Medicine 12th Feb 2014 by Dr .Fahad (192)

1. High Pressure in Glomerular capillaries is due to

a. Afferent is long

b. Afferent is short

c. Efferent is low resistance

d. Efferent is down stream

2.Regarding SLE followin are true except

a. ANA is +ve

b. Fibrinoid necrosis is common histological feature

c. Glomerulonephritis is commonest cause of death

d. more common in women

e. risk is increased by inheritance of HLA B27

3.Edema in nephrotic syndrome

a. Na retention

b. Hypoproteinemia

c. Hyperlipidemia

4.Bronchopulmonary segment

a. each has its own blood supply

b. different number on both sides

5.Recently diagnosed fatal disease, most appropiate

a. tell family but nt patient

b tell patient but nt family

c. tell family and patient as soon as knwn

d. crisp, logical and evidence based accurate information to the patient and family accordin to demand
153

6.Young lady having anemia, MCV 78 , MCH 25, MCHC 29

a. Hypochromic

b.Hypochromic and Normocytic

c.Hypochromic and microcytic

d. normocytic and normochromic

7.WHICH OF THE FOLLOWING SHIFTS THE O2-HGB

DISSOCIATION CURVE TO RIGHT:

A.ACIDOSIS

B.ALKALOSIS

C.INCREASE pH

D. NO

E. FETAL HEMOGLOBIN

8.A 75 year old man with an acute MI suddenly dies 6 days after the infarct . At autopsy, there is a

large amount of blood in the pericardial sac . The most likely cause of death is ;

A. Cholesterol pericarditis

B. Amyloidosis

C. Cardiac tamponade

D. Codsackie B pericarditis

E. Constrictive pericarditis

9.most active gluconeogenesis results from

metabolism of ?

a.fatty acid

b.protein

c.cholesterol

d.glycogen

e. TG

10. Severe magnesium deficiency

a. Hypocalcemia

b. Hypercalcemia

c. Hypokalemia
154

d.Hypophosphatemia

11. Langerhans giant cells found in

a.Tuberculosis

b.Sarcoidosis

c.Wegners granulomatosis

D)Syphlis

12. parasympathetic inervation

a.sweat gland

b.salivary gland

13. A pt has multiple polyps on colonoscopy.His father died of same disease.These types of polyps are

14. Dysarthria is due to a lesion in?

a.Brocas area

b.Cerebellum

c.Thalamus

d.Sensory motor cortex

e.Hypothalamus

15.Hypothyroidism

a. Increased cholesterol

16. During 2nd week which is appropriate test for

Typhoid fever ?

a) Widal+Blood culture

a-Hemartomatous

b-Adenomatous polyp

c-Metaplastic

b) Blood culture only

c) Stool culture only

d) Bone marrow

17. A woman havin fever for few days developed chest pain which aggravated by lying down . Reason

is?

a.Costochondral junction
155

b.Mi

c. pericarditis

d.due to plurea

18. purpura is most likely asaociated with

a. leukemia

b. von wilbrand disease

c. hiv

19. a lady using diethylesterbestrol what should be most likely in her daughter

a. squamous cell carcinoma

b. clear cell carcinoma

20. Lady with osteoporosis,having family h/o cardiac disease and breast ca. Treated by

a. Calcium plus vit D

b. Raloxifene

c. bisphosphonates

d. HRT

21. Which disease involves a carrier stage ?

a.Measles

b.Mumps

c.Polio

d.Typhoid

22. H+ ion

secretion in proximal tubule is related with?

A. Bicarbonate reabsorption

B. Bicarbonate excretion

C. K secretion

D. Na secretion

23. Hamartoma is ;

A. A Metaplasia

B. A neoplasia

C. Has a capsule
156

D. Same as cholestoma

E. Totally benign

24. 40 yr old woman, middle class,3 kids ,pap smear shows cervical dysplasia..cause

A HPV

B IUCD

25. 10% Formaldehyde is used for sending Biopsy

specimens,Why?

a. Sterilisation

b.To prevent autolysis

c. TO stain cells

d. TO denature proteins

e. to cause lysis of cell

26. Metastatis occur in all except

a.Liver

b. Brain

d. Spleen

d. lung

27.Fast, sharp pain from mechanical origin is mediated by which fibers;

A. A alpha

B. A delta

C. C fiber

D. High myelinated

28. Fat necrosis occurs in ;

A. Acute pancreatitis

B. Heart

C. Liver

29. Which tumor is derived from all the 3 germ layers;

A. Adenoma

B. Teratoma

C. Rhabdomyosarcoma
157

D. Chancroid

30. A 20yr old male has dyspnea on lying down. The structure most likely to cause this is ;

A. Enlarged isthmus of thyroid

B. Puberty

C. Retrosternal goiter

31. A FISHER MAN PRESENTED WITH SLOWLY DEVELOPING LETHARGY, EASY FATIGUE AND

PALPITATIONS. HIS DIET COMPRISES OF FISH AND RICE MOSTLY PHYSICAL EXAM SHOWED
PALLOR

AND LOSS OF TOUCH SENSATION IN BOTH FEET AND LOWER LIMB. HIS CBC SHOWED HB 7.5G/DL
MCV

132, PLATELETS 110 AND WBC 3450 . MOST LIKELY CAUSATIVE PARASITE:

A.ANKYLOSTOMA DUODENALE

B.DIPHYLLOBOTHRIUM LATUM

C.STRONGYLOID STERCOLIS

D.ECHINOCOCCUS GRANULOSIS

E. TAENIA SAGINATA

32. Long term elevated BP is due to

a.cardiac output

b.renal function

c.TPR

33. The vitelline duct forms;

A. Ductus venosus

B. Duodenum

C. Meckel, s diverticum

D. Portal vein

E. Urachus

34. Which is correct Regarding Axillary artery

a-It Lies behind pectoralis minor

b-Extend upto the lower border

of pectoralis major

35. a. the components of the cell containing microtubules in there structure are;
158

a. cell membrane

b. centrioles

c. endoplasmic reticulum

d. lysosomes

e. mitrochondria

36. a chronic smoker from 30 years working in a tyre factory presented with painless hematuria n on

histopathology transitional cell carcinoma of bladder causative agent is ..?

a.aromatic amines

b.tobaco smoking

c.nitrites/nitrates

d.shistosomia hematobium

37. most common manifestation of autoimmune

disease ;

a. Fever

b.arthralgia

c.haematologic symptoms.

d.fatigue

38. ACTH effectively controls ?

a. Aldosterone

b. Adrenal androgens

c. Adrenal estrogen

d. Hydrocortisone

39. what defines the acute inflammation ?

1.local response of living tissue to injury

2.systemic response of living tissue to injury

3.migration of leukocytes

40. aortic pressure lowest during which phase of cardiac cycle??

a.atrial systole

b.isovolumic contraction

c.isovolumic relaxation
159

41. The vision lost is left temporal and right nasal field of both eyes, the visual pathway involved in

this lesion is

A. Rt optic nerve

B. Left optic tract

C. Rt optic tract

D. Left optic nerve

42. the affect of L dopa in parkinson pt dcreease gragually due to

1.antibodies develop against l dopa

2.gradual degeneration of neurons in sunstantia nigra

43. Glucose transport across a membrane is directly proportional to concentration graddient . Type of

transport is ;

A. Simple diffusion

B. Facilited

C. Active

D. Na countertaransport

44. sebacous glands are absent on

1.corner of lips

2.palms

3.glans of penis

45. Blood transfusion reaction is likely to occur when a person having blood group A+ is transfused

with blood of group;

A. A+

B. AC.

ABD.

O+

E. O-

46. C0mplicti0n of diabetic?

a.Wet gangren

b. dry gangren

c. ranyod phen0mena
160

d. thrmbroagints

47. true regarding genetic inhertance of SLE is ;

a. high incidence in monozygotic twins

b. incidence is high in females

c. if its in family, more chances to occur to all family members

48. Langhans giant cells are found in;

a.Tuberculosis

b.Sarcoidosis

c.Wegners granulomatosis

d. Leprosy

49. most active gluconeogenesis results from metabolism of ?

a.fatty acid

b.amino acids

c.cholesterol

d.glycogen/carbohydrates

50. Esophago gastric junction competence is

maintained by

A. lying supine

B. increased intra abdominal pressure

C. diaphragm paralysis

D. use of metoclopramide

E. use of morphine

51. Diminished jaw jerk due to lesion of

A. Mesencephalic nucleus of V nerve

B. Facial motor nucleus

C. Glossophyrngeal injury

D. Vagus injury

52. Co-trimoxazole action in.hiv pt on pnumocystc carini?

a.Inhibit cellwall syn

b.Inhibit folic acid synth


161

c. inhibit DNA synth

53. Which of the following structure doesn't pass posterior to the flexor retinacum

A. Ulnar nerve

B. Median nerve

C. Flexer digitorum superficialis

D. Flexer digitorum profundus

54. Cardiac surgeon during open heart surgery found bleeding vessel accompanying LAD in anterior

interventricular groove . Most likely involved vein is ;

A.Great cardiac vein

B. Anterior cardiac vein

C. Oblique vein

D. Small cardiac vein

E. Marginal vein

55. An upper motor lesion of facial nerve will result in ;

A. Excessive lacrimation

B. Hyperacusis

C. Loss of taste sensation in ant 2 /3 of tongue

D. Ability to wrinkle the forehead on the affected side

56. SQUAMOUS CELL CARCINOMA

a.Actinic keratosis

b. marjulian ulcer

c. Bowen Disease

57. Regarding histology of gastrointestinal tract, which is inappropriate ;

A. Peneth cells contain Eosinophilic granules in apical cytoplasm

B. Peyer's patches are present in submucous layer of duodenum

C. Parietal cells of human stomach secretes intrinsic factor

D. Striated brush border is composed of microvilli

E. The longitudinal coat of muscularis externa is arranged in 3 bands in colon

58. Which one.is.malignant

a.Glioma
162

b.Meningioma

c.Psedolymphoma

d.Chondroma

59. which cranial nerves r parasympathetic

A. III, VII, IX, X

B. V, VIII, II, XII

60. Severly dehydrated man comes in er what to gv

a.5percent glucose

b.Ten precent glucose

c.Hypertonic albumin

d. 0.9% nacl

61. Fibrous dysplasia commonly occurs in

a.Maxilla

b.Occipital bone

c.Frontal bone

d.Temporal bone

62. Weight of prostate

a.14 gr

b.18 gr

c.30 gr

d.40 gr

63. Edema is caused by;

A. ↓ hydrostatic pressure

B. ↑ osmotic pressure

C. Lymphatic obstruction

64. Growth hormone secretion is increased ;

A. Exercise

B. Hperglycemia

65. Drug action wont b affected bound to alpha

glycoprotein in
163

a. Neoplasm

b.Trauma

c. Hepatic disease

d. MI

66. Clinically GFR is measured by

a. Creatinine

B. Inulin

67. 16 yr child with cola color urine since one day , Hb 6 , high grade fever , was given antimalarial 1

day back ..

a.g6pd

b.falciparum(blackwater fever)

c. PNH

68. The cause of death in gas gangrene is due to ;

A. Bacteremia

B. Toxic shock

C. Pneumonia

69. which is not the content of mediastinum...?

a.lungs

b.heart

c.trachea

d.vagus nerve

e.large vessels

70. Diseases transferred thru vertebrate animals to

humans:

a)zoonosis

b)zonoarthosac

c)Zoncosis

d) anthropozoonosis

71. about calcitonin...?

a.must b given parenteral


164

b.may b used in intoxication of vid D

c.hypoparathroidism

d.a polypeptide of 32 aminoacid

72. most common conginital heart dsz?

a)asd

b)vsd

73. A man with AIDs who has no symptom with CD4 count 800 presented with suppurative otitis

media, causative organism is:

A. Mycobacterium

B. Strep pneumoniae

C. Pseudomonas aeruginosa

d)stap aureus

74. A pt can survive without mechanical ventilation, if the lesion is ;

A. Above C2

B. At C3

C. At C4

D. Below C5

75. Preganglionic sympathetic nerve fibers release ;

A. ACH

b. Norepinephrine

C. Dopamine

76. 5th Post intercostal artery

a. Thoracic aorta

77. Which one of the following is a COX II inhibitor ;

A. Aspirin

B. Indomethacin

C. Meloxicam

78. Fasting blood sugar of pt is 6.8 mmol (122 mg/dl) after 1 hr of OGTT it is 10.9 mol and later after 2

hr postprandial it was 10.7 mol ( 194 mg/ dl ) what is the diagnosis

A. DM
165

B. impaired glucose tolerance

C. Long lag phase

79. Not recorded on ECG normally

A. SA NODE

b.LEFT ATRIUM

c.AV NODE

d.RIGHT ATRIUM

80. After 48 hr of starvation, wht will be broken down to provide energy,;

A. Carbohydrate

B. Fat

C. Protein

81. Which of the following causes increased interstitial pressure;

A. ↑ capillary permeability

B. ↑ colloid osmotic pressure

82. Post op wound causes greenish pus discharge . Which organism is involved ;

A. Psudomonas

B. Klebseilla

C. E. Coli

83. Infection spreads retroperitoneally, which will b infected

a) Spleen

b) Jejunum

c) Transverse colon

d. Ascending colon

84. Sprain of ankle leads to eversion of ankle, which ligament is damaged .

a. Deltoid

b. Talofibular

c. peroneal

d. tendocalcaneous

85. A pt, wd DK didnt respond to treatment and died, Autopsy show inflammatory lesion in kidney

brain and lung, with some fungus. Most likely organism is ;


166

A. Mucormycosis

B. Candida

86. Person naked in room, temperature 21 degree centigrade, humidity present, loss of heat by ;

A. Insensible perspiration

B. Sweating

C. Conduction and radiation

87. Microscopic feature of polyarteritis nodosa is ;

A. Granuloma

B. Fibrinoid necrosis

C. Associated with hepatitis c

88. Which of the following structure damages in thyroidectomy ;

A. Recurrent laryngeal nerve damage

B. External laryngeal nerve damage

C. Laryngeal oedema

D. Sore throat

E. Vagus nerve damage

89. ACTH effectively controls ?

a.Aldosterone

b.Adrenal androgens

c.Adrenal estrogen

d.Hydrocortisone

e.NE

90. which cannot b measured by spirometry

1.residual volume

2.expiratory reserve volume

3.inspiratory reserver vol

4.tidal vol.

91. the following diseas occur in iv abuser

1)infective endocarditis

2)llibmansack endocarditis
167

92. Normal FEV1/ FVC ratio is ;

A. 0.6

B. 0.8

C. 1

D. 5

93. Pt unable to void after fall which of the following section of spinal cord is affected;

A. L2, L3, L4, L5

B. L1, L2

C. S1, S2, S3

D. S2, S3, S4

94. osmotic diuretic effects on which part of nephron

a.proximal tubule

b.collecting duct

c.asc loop of henle

d.glomerular memb

95. After rapid loss of 2 liter of fluid . person drinks plain water . it ll lead to

A. Increase in interstitial fluid only

B. Increase in intracellular fluid volume

C. Plasms volume increase

96. A patient known smoker in laryngoscopy found laryngal nodule what is most unlikely

1 neoplasm

2 hyperplasia

3 keratosis

4 atrophy

97. secondary center of ossification

a.epiphysis

b.epiphyseal plate

c.diaphysis

98. .liquefactive necrosis.

a.brain
168

b.kidney

c.spleen

99. 1st heart sound

a.isovolumetric contraction

100. A 40 year old man has pain felt deep in the face and nose between the mouth and orbit to the

ear and temporal region . the pain may be due to the lesion of the ;

A. auricular nerve

b. facial nerve

c. hypopharyngeal nerve

d. maxillary branch of the trigeminal nerve

e. opthalmic branch of the trigeminal nerve

101. EBV

nasopharyngeal carcinoma

102. inverse stretch reflex(resulting relaxation)

golgi tendon organ

103. fast sharp pain

A delta fibers

104. 3rd ventricle

bounded laterally by 2/3 thalamus (Rabia Ali

MCQ)

105. glycogen breakdown

aerobic and anaerobic break down to pyruvate

106. fatty acids co enzyme

biotin

107. %age

units/total units X 100

108. The diagnostic lab test for the infection by streptococuss beta haemolyticus is ;

A. ASO titre

B. Blood culture

C. ESR
169

D. Leukocytes culture

E. Urine for bile pigment

109. disclosing pt's secret info

A. only with pt. consent

B. Insurance Claim

110. Most common cause of inc bleeding time

a.thrombocytopenia

b. Aspirin

111. opsonin

C3b

112. plasma cells produce

antibodies

113. black water fever

a.p.falciparum

114.Sulphonylurea?? Don’t remember the options but question was asking about mechanism of action

115. alkalinize urine for removing

a.phenobarbitone

116. which one is true for

parasympathetic sys on Heart

a.inc the AV nodal delay

b. Decrease K ion outward current

c. Increase Ca++ inward Current

d. Decrease Inward Na+ current

117. .parasympathetic action

a. sweat glands

b.salivary glands

118.ectopic pregnancy most

common site

a. uterine tube

119. both central n peripheral chemoreceptors respond to


170

a..arterial pCO2

b. Increase pH

c. Decrease pO2

120. meckel's diverticulum

a.vitelline duct

121. .insulin dec by

a.beta blockers

122. most sensitive cardiac marker

a.trop T

b.Ckmb

123. Hypothyroidism

a. increased TSH

124. left adrenal vein drain into

a.left renal vein

125. fastest B.P regulation

a.baroreceptor

126. ESR inc in

a.infections

b. polycythemia

127.local factor delaying wound healing

a.foreign body

b. local infection

128.actin covered by

a.tropomyosin

129. turner

a.44xo

130.klinefilters syndrome

a.46XXY

131. S2,S3,S4 lesion

a.rectal incontinence
171

b.no pain in labour

c. anorgasmia

132. pain referred to shoulder

a.c2c3

b.C3,C4,C5

133. compression of S1

a.dec ankle jerk

134. not related to rt. kidney

a.descending colon

135. external carotid

a.lateral to retromandibulr vein

136.symphisis pubis

a.sec. cartilaginous joint

137.fibrous and parietal pericardium supplied by

a.phrenic

138. corona radiata

a.granulosa cells

139. .cimetidine with sucralfate

a.decrease metabolism

b. Sucralfate decreases cimetidine absorption, give it after 2 hours

140. Cimetidine may significantly prolong the prothrombin time and causes bleeding in pt who has

been treated with warfarin by ;

A. decreasing the hepatic clearance of warfarin

B. Displacing warfarin from plasma protein

C. Increasing the oral bioavailability of warfarin

D. Inhibiting renal tubular secretion of warfarin

141. .sternocleidomastoid supplied by

a.accessory nerve

142.regarding collagen

a.present in all types of connective tissue


172

b.unique protein

145.organ for metabolism

a.liver

b kidney

146. Regarding Portal vein following statement are correct ; Except

A. Develops partly from the cranial part of right vitelline vein

B. is connected to veins of anterior abdominal wall through paraumbilical vein

C. Is formed by union of superior and inferior mesenteric vein

D. lies in the free margin of lesser omentum

E. tributaries include left gastric and superior pancreatoduodenal vein

147. compact bone...

a.Regularly arranged lamelae

148. An initial left to right shunt before reversal is most commonly a feature of ;

A. Coarction of the aorta

B. Ebstein' s malformation

C. Eisenmenger's complex

D. Transpostion of the great arteries

E. Truncus arteriosis

149. Digoxine unlikly

1.inc Ca intracellular

2.inc K intracellular

3.inc Na intracellular

4.inc sympathetic to heart

150. u wave is associated with

A..atrial depo

B .atrial repo

C. ventricular repor

4. ventricl depo

5. slow repolarization of papillary muscles

151. A 25year old sprinter developed acute leg pain while running. the next day he noticed
173

ecchymosis around ankle . he can stand on his toes through it hurts . the most likely cause is ;

a. acute arterial embolism

b. deep vein thrombosis

c. herinated lumbar disc

d. ruptured Achilles tendon

e. ruptured plantaris tendon

152.A 30 year male came to a consultant clinic with history of progressive right facial numbness and

headach for 3 months, on examination, facial nerve palsy was confirmed on right side along with VIII

nerve involvement of same side, and right sided Intention tremors, the most likely site of lesion is ;

A. Cerebello pontine angle

B. Pons

C. Rt lobe of cerebellum

D. Rt side of mid brain

153. Wilsons disease

A. Ceruloplasmin

154. 16 yr old boy with diarhea,biopsy showin villous atrophy, not improved by gluten free diet, most

likely diagnosis

a. Celiac disease

b. Giardiasis

c. Whipples disease

155. IM ketorolac given in

a. Renal failure

b. Pregnancy

c. Suspected drug addicted complaining of pain

156. Water and salt reabsorption is maximum at

a. Duodenum

b. Stomach

c. jenunum

d. Ileum

e. Colon
174

157. wht happens in patient gettin TPN

a. Hyperglycemia

b. hyperphospatemia

158. epinephrine given slowly???

a. Decrease in TPR

b. Increase in diastolic pressure

159. Premalignant oral lesion

a. Lichen planus

b. Submucosal fibrosis

160. Question from biostatistics (Asked

about Wrong Statement)

a.statistical data is of no significance

b. Chi square

161. Nerve loops around arch of aorta

A. recurrent laryngeal nerve

162. Hemolytic anemia question?? Scenario was given and Correct Answer was Hemolytic anemia

163. Regarding atonic bladder, it is caused by

A. damage to parasympathetic efferent pathway

B. Damage to sympathetic efferent pathway

C. Cerebral cortex damage

D. Pudendal damage

164. Toxoplasmosis is inection

caused by

a. Parasite

b. Virus

C. Fungus

D. Bacteria

165. transmitted from eating undercooked meat

a. tapeworm

b. Liver fluke
175

166. false statement about Carbimazole

a. increases T4 to T3 conversion

167. lady with exophthalmos, thyrotoxicosis

a. Graves

b. MNG

168. Mitral stenosis, absent waves are

a. c waves

b. a waves

169. What wil decrease in hypotension

a. Carotid sinus discharge

170. RBC fragility

a. 0.35 % NaCl cause complete hemolysis

b. Hypotnonic cause swelling

c. Hypertonic causes shrinkage

171. Standing from 1 hour

a. Decrease CVP

172. something about Gamma motor neuron

a. Relaxes muscle

b. controls fiber 1a afferent

c. contract the intrafusal fibers

173. Arteriolar dilation leads to

a. increase in arterial blood volume

b. increase in venous volume

c. something about capillaries

174. Cause of Squamous Cell Carcinoma bladder

a. Shchistosoma hematobium

175.Post Commnicating artery

a. Internal carotid and Post Cerebral artery

176. Sjogrens Syndrome

a.dry mouth and eyes


176

b. Dry mouth only

c. Dry nasal mucosa

177. Which one is not epithelial tumour

a. Adenocarcinoma

b. Liposarcoma

c. Squamous cell Ca

178. Diphtheria toxoid given n thn aftr 2 weeks skin rashes , most likely reaction

a. Type 1

b. type 2

c. Type 3

d. Type 4

179. Cardiac output low in

a. beri beri

b. Cardiac failure

c. AV fistula

d. hyperthyroidism

180 Labetalol

a. Beta blocker

b. Alpha blockr

c. Alpha plus Beta blocker

181.Monocytes likely

A. Can transfer into large multinucleated giant cells in chronic infection

B. Increase in allergy

C. Produce IgM

D. Formed from precursor cells in lymph nodes

E. Do not migrate across capillary wall unlike granulocytes

182. not true abt surfuctant

a.increase surface tension

183.Lipid diet

a. Cholecystokinin
177

184. Reflex involving cerebrum

a Light reflex

b. Consensual light reflex

c.Accomodation reflex

d.Visual body reflex

185.Ocular myasthenia

a. Decreased postsynaptic receptors

186.Oral anticoagulant

a.PT

b. aPTT

187.Cushing syndrome

a.ACTH

188.Renal excretion of drugs???

a.Something about Plasma proteins

189.graft rejection

a. Acute Cellular

190.Ventilation of lungs

a.increased at base

Good Luck for your Part-1 Exam

191. Failure of aorta to separate from Pulmonary artery

a. Truncus Arteriosus

b. TOF

192. Peptic ulcer pain

a. Greater splanchnic Nerve

b. lesser splanchnic nerve

=-=-=-=-=-=-=-=-=-=-=-=-=-=-=-=-=-=-=-=-=-=-=-=-=-=-=-=-=-=-=-=-=-=-=-=-=-=
178

2. Medicine August 2014 Online by Dr.Zara Bashir (148)

Q1.. One of the following chemical mediator is for pain

a.Bradykinin (ANS)

b.Leukotrines

c.Nitrous oxide

Q2..One of the following is used in non neoplastic diseases and act as an immunosupressant

a.Methotrexate

c.Cisplatin

Q3..GFR increases due to

a.Dilatation of afferent arteriole(Ans)

b.Dilatation of efferent arteriole

Q4..Patient brought with history of heavy drinking and ascites lab investigations show

impaired LFTs most probable finding on histological exam of needle biopsy of liver show

a.Constricted central vein

b.Disorganized liver architecture(Ans)

Q5..Mitotically active liver cells

a.Normal portal triad

Q6..Patient with peptic ulcer has epigastric pain the referred visceral pain is mediated by

a.Greater splanchnic nerve(Ans)

b.Lesser splanchnic nerve

c.T-9 spinal nerve

Q7..Regarding epithelia and cells

a. Holocrine type of secretion occurs in mammary gland

b. An endocrine gland is marked by the presence of a number of ducts

c. the simple squamous epithelium is usually found at places where exchange of gases takes

place(Ans)

Q8..A man with head injury has difficult in uttering words and his speech has also become

slow damage is in

a.Frontal lobe

b.Brocas area(Ans)

c.Wernickes area

d.Occipital lobe
179

Q9..Action potential in SA node is caused by

a.Na influx

b.Ca influx(Ans)

c.Increase K+ conductance

Q10. High output cardiac failure is found in

a. anemia

b. beri beri(Ans)

c. hyperthyroidism

d. high blood pressure

e. total peripheral resistance

Q11. following lesion is considered as premalignant lesion

a. compound naevues|(Ans)

b. contact dermatitis

c. psoriasis

d. seborrhic keratosis

e. intradermal naevus

Q12..During collection of blood sample from medial cubital vein which nerve is most likely

to be pierced

a.Median cutaneous nerve of arm

b.Median nerve(Ans)

c.Musculocutaneous nerve

d.Radial nerve

Q13..Mean systemic filling pressure regulates

a.Mean arterial pressure

b.Pulmonary vascular resistence

c.Right arterial pressure

d.Systemic vascular resistance

e.Venous return

Q14..Percentage of lichen planus was asked in the paper options were

a.1-10%(Ans)

b.5-10%

c.10-15%

d.15-20%

e.20-25%
180

Q15..A hypertensive 40 year old man developed severe chest pain and died at autopsy

there was extensive dissecting aneurysm of aorta what is the most likely cause

a.Atherosclerosis of aorta

b.Medial necrosis of aorta(ANS)

c.Saccular aneurysm of aorta

Q16..A 7 year old boy had acute onset of fever and sorethroat he responded well to

antibiotics however 3 weeks later he started to have generalized edema painful

micturation and hematuria these symptoms indicate that he probably has

a.Good pasture syndrome

b.Henoch shonlein purpura

c.Post streptococcal glomerulonephritis(Ans)

Q17..urine creatinine 144mg/dl, 24 hour urine volume 1.4 litre, Plasma creatinine

1.2mg/dl..... find creatinine clearance 168 litre/day is ans... 31 st aug medicine mcq

Q18..Protein catabolism in extrahepatic tissue and protein synthesis in liver is action of:

a. cortisol(Ans)

b. epinephrine

c.growth hormone

d.insulin

e.thyroxine

Q19..Metaplasia is characterized by

a. functional change in cell

Q20..An increase in nuclear cytoplasmic ratio of cells

a.Variation in the size and shape of cell

Q21..A 12 year old boy has frequent episodes of pain in the abdomen chest and joints he is

usually febrile and has jaundice at the time of these attacks the spleen is mildly enlarged

HB varies 6 to 8 and the reticulocytes are raised the most likely cause of his

hemoglobinopathy is hemoglobin

a.C

b.D

c.E

d.S-C

e.S-S(ans)

Q22..A 55 year old insulin dependent diabetic female had a major abdominal surgery on

the 8th postoperative day she complained of sudden onset of dyspnea chest pain and
181

hemoptysis the most likely cause of her symptom would be

a.Myocardial infarction

b.Pneumonia

c.Pulmonary embolism(Ans)

d.Septicemia

e.Spontaneous pneumothorax

Q23..Subacute bacterial endocardititis after tooth extraction is caused by

a.Strep viridians(ans)

b.Staph aureus

Q24..P53 is a tumor suppressor gene when it is absent or mutated it may indicate

a.Apoptosis

b.Cell survival(ans)

c.DNA repair

d.Mitosis

e.Short G1 phase

Q25..Most commonly fracrured long bone

a.Tibia(Ans)

b.Fibula

c.Humerus

d.Femur

e.Radius

Q26...A 60 years old man presents with 8 month history of progressive generalized

lymphadenopathy his HB is 9.8 TLC is 59×10*9 and platelet count is 94×10*9 the peripheral

film mostly show mature looking lymphocytes the most likely diagnosis in this patient is

a.Chronic lymphocytic leukemia(ans)

b.Hairy cell leukemia

c.Malignant non hodgkin lymphoma

d.Sarcoidosis

e.Tuberculous lymphadenitis

Q27..Functional residual capacity is

a.IRV+RV

b.ERV+RV(Ans)

c.IRV+ERV+RV

Q28..Superficial temporal artery is accompained by which nerve


182

a.Auriculotemporal nerve(Ans)

b.Glossopharyngeal nerve

c.Deep petrosal nerve

Q28.. Injury to common peroneal nerve will lead to

a.Loss of eversion(Ans)

b.Loss of inversion

Q29. Oblique fissure of the lung

a.T3-T6(Ans)

b.T2-t4

Q30..The maximum effect (emax) achieved by a drug is a measure of its

a) bioavalability

b) efficacy(Ans)

c) intrinsic activity

d) potency

e) safety

Q31.Nutmeg liver seen in

a.Chronic-acute viral hepatitis

b.Chronic passive congestion(Ans)

Q32.DNA replicate in which phase

a.Prophase

b.Metaphase

c.Anaphase

d.Interphase(ans)

Q33..Alcohol intake associated with

a.Vitamin A

b.Vitamin C

c.Vitamin B1(ans)

Q34..Posterior 1/3 of interventricular septum is supplied by

a.LAD(Ans)

b.LCX

c.RCA

Q35..Left axis deviation occurs in

a.Right bundle branch block

b.Left bundle branch block(ans)


183

Q36..Patient with hyperthyroidism tsh is low.TRH is given ths will increase tsh where is

the problm

a.Pitutary

b.Hypothalmus(Ans)

c.Thyroid

Q37..Patient with ptosis,anhidrosis,mydriasis aftr fever

a.Oculomotir nerve palsy(Ans)

b.Horners

Q38..True abt cell

a.Nucleolus makes ribosomal dna

b.Hemotoxtlin is an acidic dye(ans)

c.Cell is in euchromatic phase if its trabscritionaly active

Q39.Medial fasiculus formed by decusation of

a.Pyarmids

b.Intrnal arcuate(ans)

c.Post extrnal arcuate

d.Ant ext arcuate

Q40.. .Deep sleep hornore

a.Serotonin

Q41...True abt NREM

a.Delta waves(Ans)

b.Night nares

Q42..Alpha waves

a.disapear upon opening eyes

Q43.2nd hrt sound producd by

a.closure of tricuspid n mitral valve

b.closure of aortic n pulmonary valve(Ans)

Q44..Patietn with low third heart sound

a.rapid refilling of ventricles

few other options dont remember

Q45..Butterfly rash and nephritis

a.Sle nephritis

Q46..How to diagnose SLE specificaly

a.Anti ds dna
184

Q47..Patient smoker psycotic with vomitin abdominal pain n visible perstalsis in upper

portion of stomach

a.Pyloric stenosis

b.Psycogenic(Ans)

c.achlasia

Q48..complete Ventricular depolarization state

a.P wave

b.St

c.Qrs(Ans)

Q49..Saw tooth ecg

a.Firing rate 250-300(ans)

b.More thn 450

c.Trtmnt is defibrilator

Q50..First line defense in tissue

a.Macropage,neutrophils

Q51..Ige ataches to

a.Eosiphils(ans)

b.Basophils

Q52..Coarctation of aorta

a.narrowing of aorta Proximal to subclavian artry origin

Q53...Ecf grealy increasd by

a.100ml isotonic sol

b.100ml hypertonic(ans)

c.100ml hypotonic sol

Q54...Most common facturd bone

a.Tibia(ans)

b.femur

c.humerus

d.radius

e.fibula

Q55..Most common cause of mutliple fractures

a.Cushing

b.Osteoporirsis(ans)

c.Hyperpara
185

d.Hypo para

Q56...Mgso4 toxicity reversd by

a.Cacl2

Q57..,Leprosy diagnosis in early stage

a.nasal or lesion scraping(Ans)

b.sputum tests for lepra bacili

c.blood culture

Q58..Cervix with large irregular cells, cells have large nuclei n prominent nucleolus after

2yrs of hpv

a.Carcinoma

b.Metaplasia(Ans)

c.Warts.

d.dysplasia

Q59...collagen fibers

a.are of yelow colour

b.freely devide

c.staiined with eosin(ans)

d.prominent in ligament flavum

3 questions abt muscle spindle n gamma fibers abt gamma and alpha

Q64.a lady wid round face fat on neck and abdomen with hirsutism and raised ACTH

a.cushing disease

b.cushing syndrom(Ans)

c.conns syndrom

Q65..a pt with hyper pigmented elbows hyperkalemia hypotension like snerio

a.addisons(ans)

b.cushing

c.conns syndrom

Q66...who r true hermaphrodites

a.xxy(Ans)

b.xyy

c.yy

d.xo

Q67..wheat is a rich source of

a.vit c
186

b.thiamine(Ans)

c.riboflavin

d.vit a

e.vit d

Q68...oblique fissure of lungs

a.t3 t6(Ans)

b.t3 t4

Q69...trauma to peroneal nerve

a.loss of eversion(ans)

b.loss of inversion

Q70...epithelim covering palatine tonsils

Q71..progressive weaning of muscles in evening related to mysthenia gravis.confirmatory

test is

a.antibodies to ach receptors(ans)

b.emg

Q72..a pt injured in periphary is transfused with 2 week old saved blood he is

predominately transfused with which type of cells

a.rbcs(ans)

b.lymphocytes

c.platelets

Q73.thyroid isthmus is related to

a.recuuret larayngeal nerve

Q74..some drug used for mysthenia gravis.uska mech of action poocha hoa tha

a.it act as anticholinestrase

b.raises choline

c.as cholinestrase

Q74.frontal area is supplied by

a.basilar artery

b.arteries dont anastmose once they enter brain

c.artries dont anastomos on surface of cerebral area(ans)

Q75.foot area on motor humonculus is supplies by

a.middle cerebral artery

n other options dont remember

Q76..pt with po2 60 hb 15.what type of hypoxia will occour


187

a.anemic hypoxia

b.methemoglobiinemia

c.cyanide poisioning

d.hypoxic hypoxia(ans)

Q77.which is absorbed passively

a.urea(ans)

b.glucose

c.k

d.na

Q78..thiazide causes

a.hyperkalemia

b.hypo kalemia(Ans)

Q79.a pt with thorn snerio

a.staph aureus

Q80.effect of parasympthtic

a.on git

Q81.parasypmthtic action

a.inc salivary secretio

b.decrease sweating

Q82..vasodilation is caused by

a.alpha1 adrenergic

b.alpha 2 adrenergic(ans)

c.beta 1 n beta 2 adrenergic

Q83.transverse colon is supplies by

a.right rectal artery esa kuch tha dont remember exactly

Q84...amniocentesis can be done

a.in week 4-5

b.weeks before 10

c.befor 14 weeks(ans)

Q85...which is not malignant

a.adenoma

b.lymphoma

c.hamartoma(ans)

d.melanoma
188

Q86..which is rich source of cholesterol

a.egg(ans)

b.red meat

Q87..a man with firearm injury to axila.he is unable to flex index fingure and there is loss

of sensation from lateral palmer surface of skin.nerve involved is

a.axillary

b.median(ans)

c./muskulocutaneous

d.ulnar

Q88..pituitary tumour snerio wid blindness what are other finding in a centraly located

pituitary tumour

a.hypothermia

b.anorexia(ans)

c.salvage

Q92.alcohlic pt.with anorexia n vomitin.liver biopsy shows

a.fatty liver

b.mallory bodies(ans)

Q93.action potential in sa node is due to

a.influx of na

b.influx of ca(ans)

c.eflux of k

d.eflux of cl

Q94..sever pain around chin and erusption around that nerve.reason

a.herpes zoster

b.trigeminal neuralgia(ans)

Q95..a child wid fetal disease askes u is he gonna die?what wil u do

a.tel me what ur parents told u(ans)

b.no u r going to be fine

c.yes u r going to die

d.ur prents told me not to talk abt it

Q96.p53 is a suppresor gene its defect or deficiency means

a.mitosis

b.cell survival(ans)

c.apoptosi
189

d.short g1 phase

Q97...a lady after abdominal surgery develop dyspnea after 8 days

a.pnemonia

b.pulmonary embolism(Ans)

c.sepsis

Q98...metaplasia

a.is a fuctional change in epithelim

Q99...rt border of heart on ap view of xray is made by

a.right atrim(ans)

b.rt ventricl

c.ivc

d.svcc

Q100.lp done at

a.below l4(ans)

b.below l5

c.below l2

q101..git motility is decreased by

a.gastrin

b.cck(ans)

c.secretin

Q102..exudate means

a.protein >3g(ans)

b.more lyphocytes

Q103.adult polycystic is

a.autosomal dominant(ans)

b.autosomal ressesive

Q104..bp 200/130 which is damage

a.jg cel(ans)

b.renal cells

c.pct

Q105...blood born disease is

a.hept b

Q106..human is a intermediate host in

a.hydatid disease(ans)
190

b.hook worm infestation

Q107..blood supply for post 1/3 of interventricular septum

a.rca

b.lad(ans)

c.lca

Q109.maximum increase of ecf occour in

a.100ml hypertonic salin(ans)

b.100ml hypotonic normal saline

Q110..a pt wid difficulty in swalowing from 6 months.lost 5kg in 2 weeks

on endoscopy middle part of esophagus showed ulcer.causes are

a.ch alcholism

b.carcinoma(ans)

c.gastric ulcer

Q111..pain in epigastrim in pt with gastric ulcer is due to

a.greater splanchnic nerve

Q112.effects of turminal ileostomy

a.free fatty acids in stool(ans)

b.dec/inc water in stool dont remember exatly

Q113...max anteropsterior diamete of chest is achied by

a.diaphragbucket handel like movement of intercosta; muscles

Q114.in 2nd week of typhoid fever test for diagnosis is

a.widal n blood(ans)

b.blood culture

c.widal test

Q115.ige on

a.basophils

b.eosinophils(ans)

Q116.phase 1 reaction is

a.methylation

b.oxidation(ans)

c.glucronidation

d.acetylation

Q117.granulocytes related qs dont remmeber.options were

a.lymphocyte
191

b.eosiophil

c.baophil

d.neutrophil

Q118.wheat is a rich source of

a.thiamine (ans)

b.riboflavin

c.vit c

d.vita

e.vit d

Q115..pt with pco2 decreased,more oxygen raised ph.what wil happen

a.hypoventilation(ans)

b.hyperventilation

Q116.a child with sore throat hematuria

a.post streptococal glomerulonephritis

Q117.psedomonas aeinosa effects by

a.endotoxin (ans)

b.erytrogenic toxin

Q118..pt wid g6pd deficiency

a.heinz bodies(ans)

b.howel jolly body

Q120.sjogren is related to

a.rehumatoid arthritis(ans)

b.sle

Q121.pt with broken words after injury.area damaged is

a.brocas(ans)

b.wernicks

c,part of frontal lobe

Q122.most common premalignant lesion

a.erytroplakia

b.leukoplakia

c.lichen planus(ans)

Q123.eversion n inversion occur at

a.subtalar joint

Q124.a line drawn btw tip of both scapula will pass from
192

a.t10

b.t7(ans)

c.t9

d.t7

e.t6

Q125.bladder carcimoa cause

a.smoking

Q126..hard plate epithelim

a.st sq non keratinised

b.simple coloumnar

Q127..1 qs wid edema

a.increased hydrostatic pressure

Q128..2nd qs wid edema

a.lyphatic obstruction

Q129..nrem sleep

a.delta waves(ans)

b.night mares

c.deranges bp n pulse

Q130..deep sleep neurotransmitter

a.serotonine(ans)

b.norepinephrine

Q131..tru abt dna

a.euchromatic is expressed in rna synthesis

Q132..breast atrophy in young female

a.estrogen(ans)

b.progesteron n estrogen

Q133..most common cause of pul embolism

a.thromus in deep legs veins

Q134..micturation reflex

a.is self regenrative(ans)

b.related to sacral area

c.inhibited if parasympthetic supply cut

Q135..100perecnt o2 for

a.decompression sickness
193

Q136..bronchogenic ca due to

a.smoking

b.asbestosis(ans)

Q137.in adult polycytic disease most common cause of death

a.cerebral hemmorhage

b.renal failure(ans)

Q138..factor delaying wound healing

a.infection

Q139..p.falciparum causes

a.black water fever

Q140.heparin works by acting on

a.factor 12a

b.factor 9

c.factor 10a n thrombin(ans)

Q141.angina worsen by

a.theophylin

Q142..which muscle [protrudes jaw

a.lat pterygoid(ans)

b.ant digastric

c. temporalis

d. medial pterigoid

e. platysma

Q143..apex of lung

a.max ventilation

b.max dead space(ans)

Q144.pt e vulvar itching.biopsy taken .which of the following change is premalignant.

a.Chancre

b.Condyloma

c.Intradermal nevus

d.Pagets(ans)

e.Non specific ulcer

Q145. In Pneumothorax following happens...

a. Lung collapses and chest expands(ans)

b. Lung collapses and chest in-draws


194

c. Lung expands and chest expands

d. lung expands and chest in-draws

e. lung expands and chest remains same

Q146.. Which tumor does not have bone metastasis....?

a) Thyroid

b) Liver(ans)

c) Lung

d) Breast

e) Kidney

Q147.. Natural anticoagulant in the body...

a) Heparin(ans)

b) Warfarin

c) Aspirin

d) Plasminogen

e) Fibrin

Q148.. Regarding NEURAL TUBE...

a) Forms primitive streak

b) Forms notochord

c) CNS develops from it(ans)

d) Forms three germ layers

ALL THE BEST

=-=-=-

=-=-=-=-=-=-=-=-=-=-=-=-=-=-=-=-=-=-=-=-=-=-=-=-=-=-=-=-=-=-=-=-
195

3. Medicine 12th November 2014 by Amin Raza (197)


MEDICINE & ALLIED PAPER 12TH NOVEMBER 2014 WITH ANSWERS

1. Diabetic pt develops sensory abnormalities of left leg & ulcer of big toe...
Ans. Angiopathy with Neuropathy.

2. Which of the f/w is not a tumor marker:


Ans. Alpha 1 anti-trypsin

3. THIRST IS REDUCED BY...


ANS. INCREASE EXTRACELLULAR VOLUME

4. ANOTHER FUNCTION OF SMOOTH ENDOPLASMIC RETICULUM:


ANS. DETOXIFICATION

5. AN OLD WOMEN PRESENTED WITH WEAKNESS, PALPITATION & BREATHLESSNESS ON EXERTION, TINGLING OF
FEET, HB LOW TLC & PLATELET LEVEL GIVEN, DIAGNOSIS:
ANS. VITAMIN B12 DEFICIENCY

6. INVOLVED IN HYDROXYPROLINE & HYDROXYLYSIN DURING COLLAGEN SYNTHESIS...


ANS. ASCORBIC ACID

7. MALE PT WITH FACIAL NERVE PALSY ALONG WITH INTENSION TREMORS ON SAME SIDE, SITE OF LESION...
ANS. CEREBELLO PONTINE ANGLE

8. PHARMACOLOGICAL BLOCK OF HCL BY H2 ANTAGONIST...


ANS. INHIBITS BOTH GASTRIN INDUCED & VAGALLY MEDIATED SECRETION OF ACID.

9. CARDIAC VALVE LESION, ANGINA COMMON IN...


ANS. AORTIC STENOSIS

10. STIMULATION BY INJECTION OF PARASYMPATHETIC, DEC IN...


ANS. SIZE OF PUPIL

11. ALDOSTERONE SECRETION GREATLY STIMULATED BY RISE IN..


ANS. PLASMA K" LEVEL

12. GH SECRETION INCREASE IN


ANS. HYPOGLYCEMIA

14. IN MENOPAUSE...
ANS. DEC ESTROGEN INCREASE IN FSH & LH.

15. AFTER INABILITY TO VOID URINE...


ANS. S2,3,4

16. PT PRESENT WITH PAIN & INABILITY TO ABDUCT SHOULDER JOINT, TENDERNESS OVER GREATER TUBEROSTY,
AFTER INITIAL ABDUCTION OF 40°, HE IS ABLE TO ABDUCT HIS SHOULDER...
ANS. SUPRASPINATUS INJURY
196

17. HYDROCEPHALUS MOST COMMONLY DUE TO OBST OF...


ANS. AQUEDUCT OF SYLVIUS.

18. LOSS OF VISUAL ACUITY & FIELD IN THYROTOXIC OPTHALMOPATHY...


ANS. OPTIC NERVE COMPRESSION

19. COMPRESSION OF OPTIC CHIASMA...


ANS. BITEMPORAL HEMIANOPIA

20. PT WITH BITEMPORAL HEMIANOPIA LESSION IS AT...


ANS. OPTIC CHIASMA
(2 QS 1 IN EACH PAPER)

21. WASTING OF THENAR EMINENCE & WEAKNESS OF ABDUCTION & OPOSITION OF THUMB...
ANS. MEDIAN NERVE

22. PREGNANT LADY SUFFERED FRM MEASELS, ANOMELY EXPECTED IN BABY...


ANS. CONGENITAL CATARCT

23. MALE DEVELOP JAUNDICE AFTR USE OF DRUG GIVEN BY QUAKE, ENZ RAISED...
ANS. ALANINE TRANSAMINASE

24. PT OF DKA NOT RESPOND TO THERAPY & DIED, FUNGAL INFECTION, MOST LIKELY ORGNSM...
ANS. MUCOR

25. GRANULOMA WITH CASEATING NECROSIS, MOST COMMONLY FOUND IN...


ANS. TB
26. Splenectomy done chance of infection...
Ans. Strep: Pneumoniae

27. Fisherman develops lethrgy, fatigue & palpitation,


Taking fish in diet, O/E Pallor, loss of touch......
Ans. Diphylobothrium latum

28. Pregnant lady present with jaundice, total bilirubin 11, direct 9, indirect 2, has Gall stones, which enzyme ll be
raised...
I STUCK B/W ALKALINE PHOSPHATASE & GGT..
BUT I PREFR:
ALKALINE PHOSPHATASE.

29. A BOY PRESENTED WITH SKIN RASH, FEVER, JOINT PAIN, GROUPBOF LOCALIZED LYMPH NODE ENLARGMENT
AFTER THE 7 DAYS OF IVG INJECTION...
ANS: TYPE-3 HSR

30. OLD FEMALE HAS NECROTIC LESSION IN NOSE, URINE EXAM REVEALED PROTEINUREA, BIOPSY SHOWS
GRANULOMA WITH NECROSIS & VASCULITIS, DIAGNOSIS...
ANS. WEGNERS GRANULOMATS

31. OLD MALE PRESENTED WITH SORE THROAT & CERVICAL LYMPHADENOPATHY. BLOOD PICTURE SHOWS
ATYPICAL LYMPHOCYTES, USEFUL INITIAL TEST WOULD BE...
ANS. MONOSPOT TEST
197

32. A LADY FRM VILLAGE HX OF DIFICULT LABOUR, PRESENTED IN ER WITH BLEEDING P/V & OOZING FRM GUMS
FR LAST 7 HRS.
ANS. DIC

33. CHRONIC HEMOLYSIS, ENDOGENOUS PIGMENT...


ANS. HAEMOSIDERIN

34. MOST POTENT CHEMOTACTIC AGENT...


ANS. C5A

35. PARASITE ASSOCIATED WITH CHOLANGIO-CARCINOMA...


ANS. CLONORCHIS SINENSIS

36. MOST RELIABLE MICROSCOPIC FINDING OF CHRONIC HEPATITIS...


ANS. FIBROSIS

37. A CHILD PRESENTED WITH RASHES STARTED AROUND EARS THEN SPREADS OVER FACE THEN TRUNK & LIMBS,
POSIBLE DIAGNOSIS...
ANS. MEASLES

38. MOST RELIABLE INVESTIGATION/FINDING IN THE DIAGNOSIS OF TB...


I DID: CASEATING GRANULOMA
(ASIM SHOAIB KEY FR THIS BCQ IS "DEMONSTRATION OF AFB)

39. DIAGNOSTIC FEATURE OF PLASMODIUM FALCIPARUM...


ANS. BLACK WATER FEVER

40. CEREBELLUM MALARIA...


ANS. PLASMODIUM FALCIPARIUM

41. GAS GANGRENE CAUSE OF DEATH...


ANS. TOXEMIC SHOCK

42. METASTATIC CHANGE...


ANS. REPLACEMENT OF EPITHELIUM

43. METAPLASIA...
ANS. CHRONIC IRRITATION

44. MOST COMMON ROUTE OF INFECTION OF HBV & HCV...


I STUCK B/W BLOOD TRANSFUSION & CONTAMINATED NEEDLES SOME THING LIKE THAT...
BUT
I PREPHR BLOOD TRANSFUSION

45. MOST COMMON PROMOTING FACTOR FOR THE DEVELOPMENT OF LIVER CIRRHOSIS IN ADULT MALE.
I DID: ALCOHOLISM
(ASIM SHOAIB KEY HEP.C)

46. YOUNG PT WITH GENERALIZED EDEMA & PROTEINUREA 6GM/DAY, DEFECT AT WHICH RENAL STR..
ANS. BASEMENT MEMBRANE

47. MOST USEFUL PARAMETER FOR DIAGNOSIS OF SLE...


ANS. ANTIBODIES AGAINST DS DNA
198

48. OLD FARMER WHILE DOING STERNEOUS WORKBIN VERY HOT SUNNY DAY SUDDENLY COLAPSED
PLASMA NA LEVEL WAS LOW, CAUSE...
ANS. EXCESSIVE SWEATING

49. BOY BROUGHT BY HIS MOTHER WITH COMPLAIN OF LOSS OF APITITE, VOMATING & YELLOW SCLERA, INV OF
CHOICE...
ANS. BILIRUBIN+ALT

50. TYPE 3 HYPERSENSITIVITY REACTION...


ANS. POST STREPTOCOCAL GLOMERULONEPHRITIS

51. Cause of localized edema.


Ans. Allergic condition

52. Most likely complication in mouth due to chewing Betal nut & pan...
Ans. Submucous fibrosis

53. Alleles are...


Ans. Non-identical genes at same locus.

54. OT TECHNICIAN HAS ACUTE ECZEMATOUS DERMATITIS ON HANDS & WRIST IN THE AREA COVERED BY LATEX
GLOVES...
ANS. TYPE-4 HSR

55. HEMOLYTIC DISEASE OF NEWBORN CAUSED BY RH BLOOD GROUP INCOMPATIBILITY...


THE MEDIATER OF DISEASE...
ANS. IGG ANTIBODY

56. LOSS OF WHICH ONE OF THE F/W CLASS OF MOLECULES ON THE SURFACE OF A TUMOR CELL TARGET WOULD
RESULT IN LOSSBOF SUSXEPTIBILITY TO KILLING BY HOST IMMUNE CELLS...
ANS. MHC-CLASS 1

57. ORAL CONTRACEPTIVE PILLS


COMMON COMPLICATION...
ANS. THROMBOEMBOLISM

58. K/C OF TUBERCULOSIS ON ATT THERAPY DEVELOPS BIG TOE SWELLING WITH PAIN, DRUG CULPRIT...
ANS. PYRAZINAMIDE

59. ANTICOAGULANTS CONTRAINDICATION...


ANS. THROMBOCYTOPENIA

60. BARR BODY TEST FRM CELLS OF BUCCAL SMEAR IS DONE MAILY FOR THE DIAGNOSIS OF...
I DID: KLINEFELTER SYNDROME
(ASIM SHOAIB KEY FR THIS QS IS TURNERR SYNDROME)

61. EPIGASTRIC PAIN DUE TO GASTRIC ULCER, NERVE...


ANS. GREATER SPLAANCHIC

62. MONITORING OF BLOOD LEVEL FOR ANTIBIOTIC, ROUTENLY INDICATED FR...


ANS. VENCOMYCIN
199

63. STAPHYLOCOCUS AUREUS INFECTION DOC...


ANS. CLOXACILLIN

64. SOURCE OF INFECTION LIES INFRONT NECK ANTERIOR TO PRE-TRACHEAL FASCIA, INFECTION CAN SPREAD
UPTO...
ANS. ANTERIOR MEDIASTENIUM

65. REGARDING KALLMAN SYNDROME...


I DID: ANOSMIA IS COMMON FEATURE. (DNT KNW CORRECT ANS)

66. BARRIER SEPRATES BLOOD OF MOTHER FROM FETUS ETC...


I DID: CYTOTROPHOBLAST & FETUS ENDOTHELIUM (DONT KNOW EXACT WORDING OF THE OPTIONS)

67. SECRETION OF ADH IS INHIBITED BY...


ANS. ALCOHOL

68. DURING EXERCISE VENOUS BLOOD RETURN FRM LEGS BY...


ANS. CONTEACTION OF MUSCLES

69. BLOOD VESSELS CONTAIN MOST OF THE SYSTEMIC BLOOD...


ANS. VEINS & VEINULES

70. REGARDING POST: FONTANELLE...


ANS. AREA IS SMALLER AS COMPARED TO ANTERIOR FONTANELLA.

71. BASAL GANGLIA...


I DID: INITIATE MOVEMENTS (NOT SURE)

72. DIAGNOSTIC MICROSCOPIC CRITERIA OF MALIGNANT TUMOR...


ANS. INVASION (METASTASIS)

73. DIAGNOSTIC CRITERIA FR MALIGNANT OR PREMALIGNANT LESSION (DNT REMEMBER EXACT WORDING)
I DID: PLEOMORPHISM

74. RH ANTIGLUTININS...
I DID: RH+ VE FETUS IN RH-VE MOTHER

75. REGARDING PITUATRY GLAND..


ANS. SEND VEINOUS DRAINAGE TO DURA.
76. Damage to tissue by Radiations mostly depends on...
Ans. Duration of exposure.

77. Trauma case pelvic fracture with fracture of femor & tibia, pulse 110, BP 60/40mmHg, immediate
management...
Ans. IV Fluids..

78. Heparin given to pt of DVT it will prevent..


Ans. Propagation

79. MAJOR CAUSE OF ATROPHY..


ANS. LOSS OF INNERVATION

80. EARLY STAGE OF ASCHOFF BODY SHOWS...


200

I DID: EOSINOPHILS..
(CORRECT ANS: IS FIBRINOID NECROSIS)

81. SECONDRY HYPERALDOSTERNISM...


ANS. RENIN

82. AT THE END OF MERATHON RACE...


ANS. DEC INSULIN, INCREASE GLUCAGON.

83. VASOCONSTRICTION CAUSED BY...


ANS. ANGIOTENSIN

84. LEFT VENTRICULAR HYPERTROPHY...


ANS. AORTIC VALVULAR DISEASE

85. MOST COMMON POSITION OF APPENDIX...


ANS. R.C

86. +VE TENDNBRG TEST...


ANS. SUPERIOR GLUTEAL NERVE

87. A PT COMES AFTER 4 HRS WITH ALLERGIC REACTION AFTER BEE STUNG...
I DID: IM ADRENALINE

88. ALPHA FETOBPROTEIN RISE IN..


ANS. TESTICULAR CARCINOMA

89. HYPOCAMPUS REGION DAMAGE...


ANS. UNABLE TO CONVERT SHORT TERM MEMORY INTO LONG TERM MEMORIES

90. PROTEINUREA, BASEMENT MEMBRANE DISRUPTION...


I STUCK B/W AG RATIO & ALBUMINUREA
DID: ALBUMINUREA

91. AFTER ADRENECTOMY TEST FR.


I STUCK B/W NA & GLUCOSE
DID: NA

92. MULTIPLE SCLEROSIS CASE


BOY HAS COMPLAIN OF BED WETTING, TREATMENT...
I DID: OXYBUTYRIN

93. PROZONE:
I DID: HIGH TITER ANTIBODIES

94. INDIRECT PSYCO-PHYSIOLOGIC BRAIN ACTIVITY...


I STUCK B/W PUPILLARY SIZE/REACTION & SWEAT GLAND ACTIVITY...
DID: PUPILLARY SIZE OR REACTION

95. ASHMATIC + IHD PT ON OT BP SHOULD BE MANAGE WITH...


I DID: IV NITROPROSIDE
(CORRECT IS IV NITROGLYCERINE, RABIA ALI BCQ)
201

96. REGARDING PAIN...


I DID: AT EXTREME TEMPRATURE PAIN RECEPTORS STIMULATED

97. WIDE SPREAD PULMONARY INFARCTION, DYSPNEA, SHOCK...


I DID: THROMBOEMBOLISM

98. BIOAVAILABILITY DEFINATION


(STATMENT ME CLEAR NAHI HORHA THA K PUCHA KYA JARHA HAI)
I DID: ORAL ROUTE

99. INIDENCE...
ANS. NEW CASES

100. PREVELENCE...
I DID: TOTAL NO OF CASES EXCEPT SUDDEN OUTBREAK CASES (DNT REMEMBER EXACT WORDING)

101. Diarrhea...
Ans. Metabolic acidosis with normal anion gap.

102. Hyline cartilage what is visible...


I did: collagen & elastic fibers (dnt know correct ans)

103. After accident of young boy 55% lung collapsed...


Stuck b/w High V/Q ratio & atmospheric pressure equal to intrapleural pressure
DID: HIGH V/Q RATIO
(DNT KNW WHICH 1 WAS CORRECT)

104. DIC INITIATED BY...


ANS. THROMBOPLASTIN

105. MOST SENSETIVE TEST FR DIABETIC NEPHROPATHY...


I DID: URINE ALBUMIN

106. ARTERY ACCOMPANY PHRENIC NERVE MAY BE AT RISK DURING SURGERY...


I DID: CARDIOPHRENIC

107. REGARDING APEX OF HEART..


ANS. 8CM AWAY FRM MID STERNUM

108. 6CM MASS INFRNT THIGH


BIOPSY MOST COMMONLY SHOWS...
I DID: VIMENTIN

109. A MAN STAND STRAIGHT FROM 1 HR..


I DID: INC CVP (DONT REMEMBER EXACT WORDING)

110. SEVERE HYPOMAGNESEMIA...


I DID: HYPOCALCEMIA

111. HARD KERATIN FOUND IN...


I DID: NAIL

112. HYPERTHYROIDISM SNERIO


202

INC T4 LEVEL ETC WHAT LL DECREASE...


STUCK B/W TSH & REVERSE T3
DID: RT3

113. DEC LEVEL OF 0XYGEN IN ARTERIAL BLOOD BUT O2 TENSION NOT AFFECTED...
I DID: ANEMIA

114. DNA ALSO FOUND IN...


ANS. MITOCHONDRIA

115. LARGE NO OF MITOCHONDRIA FOUND ON THE SURFACE OF...


I DID: CILIARY MUSCLE (DNT REMEMBER EXACT OPTION)

116. PLATELATE ADHESION..


I DID: ROUGH ENDOTHELIUM

117. REGARDING DEAD SPACE..


I DID: HUMIDIFICATION, WARM THE AIR ETC

118. ANAPHYLACTIC & HYPOVOLMIC COMMON FINDING..


I DID: TACHYCARDIA

119. CARDIAC CYCLE PHASE B/W S1 & S2...


ANS. VENTERICULAR SYSTOLE

120. HERNEATED DISC COMPRESS...


I DID: POSTERIOR ROOT

121. REGARDING ROOT VALUE..


I DID: KNEE JOINT L4,5 (ACCORDING TO 1ST AID THIS OPTION CORRECT)

122. MAJOR CARCINOGEN...


ANS. BENZIDINE

123. PRODUCT ENTER KREBS CYCLE...


I DID: ACETYL COA

124. FACTOR NOT INVOLVE IN INTERNSIC PATHWAY...


ANS. FECTOR VLL

125. REGARDING BLOOD SUPLY OF HEART WHICH STATEMENT IS CORRECT...


ANS. LCA GIVES LAD & LCX
126. Spinal anesthesia...
Ans. Loss of vasomotor tone

127. Drugs absorption icreased by...


I did: Inc Lipid solubility

128. Secretion of Insulin stimulated by...


Ans. GIP

129. OLD MAN DIED AFTER SCHEMIC ATTACK, ATOPSY OF BRAIN SLICE SHOWS...
203

ANS. LIQUIFECTIVE NECROSIS

130. BROCAS AREA


ANS. MCA

131. MENINGITIS DEC LEVEL OF GLUCOSE FOUND IN...


ANS. PYOGENIC MENINGITIS

132. MUCUS ACINI...


ANS. SUBLINGUAL GLAND

133. DEC NO OF T CELLS FOUND IN...


I DID: AUTOIMMUNE DISEASE

134. ABOUT MAJOR/ DANGEROUS DISEASE OF CHILD DR TOLD THE PARENTS OF THE CHILD, THEIR RESPONSE...
I DID: DENIAL

135. REGARDING CCK ALL ARE CORRECT EXCEPT...


ANS. INHIBIT PANCREATIC SECRETIONS

136. SNERIO: LOSS OF MOTOR FUNCTION ON RIGHT SIDE...


LESSION AT...
I DID: LEFT INTERNAL CAPSULE

137. LOSS OF PAIN & TEMP ON LEFT SIDE, & LOSS OF TOUCH, VIBRATION ON RIGHT SIDE.
I DID: RIGHT HEMISECTION OF CORD

138. TRANSPORT THROUGH CELL MEMBRANE INVOLVING ACTIN, MYOCIN & CLATHRIN...
I DID: PINOCYTOSIS

139. JAUNDICE ON 2ND DAY AFTER BIRTH TOTAL BILIRUBIN 9...


ANS. PHYSIOLOGICAL JAUNDICE

140. REGARDING RECURRENT LARYNGEAL NERVE...


ANS. BRANCH OF VAGUS NERVE.

141. REGARDING ECG ALL ARE CORRECT EXCEPT...


ANS. P WAVE ABSENT IN ATRIAL FLUTTER

142. REGARDING CARDIAC BLOOD FLOW..


I DID: REGULATED BY LOCAL METABOLITES (DNT REMEMBER EXACT WORDING OF QS & OPTIONS)

143. REGARDING VAGUS NERVE


I DID: LEFT RECURRENT LARYNGEAL NERVE HOOKS AROUND LIGAMENTM ARTERIOSIS

144. THYROID GLAND ENCLOSED IN...


ANS. PRETRACHEAL FASCIA

145. HYPOKALEMIA...
ANS. U WAVES ON ECG
146. Complet Heart Block...
Ans. Fainting may occur bcz ateria unable to pump blood into ventricles.
204

147. Parknson pt on L-Dopa treatment eventually wears off bcz...


Ans. Dopamenrgic neurons in Subtncia nigra continue to degenerate.

148. Sq cell carcinoma on buttock lymph drain into...


I did: horz group of superficial inguinal lymph nodes

149. HARMONE OF SLOW WAVE SLEEP..


ANS. SEROTONIN

150. NEONATE WITH SMALL EYES ETC...


ANS. TRISOMY 13

151. REGARDING KIDNEY RELATIONS...


I DID: LEFT RENAL VEIN IS LONG & ANTERIOR TO RENAL ARTERY

152. DIAZEPAM+ CHLORPROMAZEPINE...


I STUCK B/W SADATION & MUSCLE RELEXATION
DID: MUSCLE RELEXATION

153. C-ANCA...
ANS. VASCULITIS

154. PT WITH UREMIC NEPHROPATHY, WHAT WOULD NOT SEEN (DNT REMEMBR EXACT WORDING OF STATMENT)
I DID: HYPOPHOSPHOTENEMIA

155. CHILD OF 6 WEEK AGE PRESENTED WITH JAUNDICE, HIS DIPPER STAINED WITH BILIRUBIN...
ANS. GILBERT SYNDROME

156. ATHELETIC AFTER INJURY UNABLE TO INVERT THE FOOT BUT ABLE TO EVERT, RUPTURE OF....
ANS. TIBIALIS ANT & TIBIALIS POSTERIOR.

157. WHICH OF THE F/W HAS HIGHEST PH...


ANS. PANCREATIC JUICE

158. LOSS OF TASTE POST 1/3 OF TOUNGE...


ANS. GLOSSOPHARYNGEAL

159. TEUE REGARDING RECEPTORS..


I DID: GOLGI TENDON ACTIVATED BY MUSCLE TENSION

160. DEEP SEA DIVER RAPIDLY ASCEND...


ANS. EMBOLI IN ARTERIES

161. ALL ARE THE FINDINGS IN THE PT OF CONGESTIVE HEART FAILURE EXCEPT...
SPLEENOMGLY, DEC PULMONARY COMPLIANCE ETC
I DID: CAPILLARIES REFILL (DON'T REMEMBER EXACT WORDING)

162. 6 YRS BOY 17-KETOSTERIOD IN URINE, ENLARGE ADULT SIZE PENIS, HAIRS ON BODY...
I DID: CONGENITAL ADERNAL HYPERPLASIA

163. REGARDING STAGING OF TUMOR...


I DID: EXTENT OF INVESION
205

164. O2 DELIVARY TO THE TISSUES DECREASE BY...


ANS. INCREASE PH

165. REGARDING CIRCLE OF WILLS..


I DID: INTERNAL CAROTID + POSTERIOR COMUNICATING ARTERY
(DON'T REMEMBER EXACT WORDING)

166. GASTRIC ACID ACTIVATE...


ANS. PEPSINOGEN

167. REGARDING ESOPHAGUS...


ANS. BEGINS AT THYROID CARTILAGE

168. WHICH STR SEPRATES OROPHARYNX FROM GUT...


I DID: STOMEDIUM

169. REGARDING YOLK SAC...


I DID: CONTAIN INITIAL BLOOD MARROW (DON'T REMEMBER EXACT WORDING)

170. HIV PT INITIAL STAGE LYMPH NODE BIOPSY SHOWS...


DON'T REMEMBER EXACT OPTIONS
171. Catecholamine derived from...
Ans. Tyrosine

172. N.Meningitis DOC...


Ans. Pencillin.G

173. Cholera, Path physiology...


I did: Activate adenyl cyclase

174. ALPHA-1 RECEPTOR ACTIVATION...


ANS. MYDRIASIS

175. THYROID CANCER, TUMOR MARKER...


I DID: CALCITONIN

176. ECTOPIC ACTH...


ANS. SMALL CELL CARCINOMA

177. PROTON PUMP INHIBITOR..


ANS. OMEPARAZOLE

178. RECURRENT ABORTIONS, THROMBOSIS ETC NOT RESPOND ON TREATMENT...


I DID: FACTOR V LADEN
(I THINK ANOTHER OPTION ANTIPHOSPHOLIPD THAT WAS CORRECT)

179. ASPIRIN OVERDOSE SIDE EFFECT...


I DID: HYPOTHERMIA (NOT SURE)

180. LESION IN MIDDLE CRANIAL FOSSA...


I DID: TRIGEMINAL

181. REGARDING CAVERNOUS SINUS...


206

ANS. ABDUCENT NERVE

182. REGARDING SEROTONIN SYNDROME...


I DID: TCA+SSRI

183. ASTHMA..
I DID: FEV1 RATIO
(2 BCQS WAS THERE NOT REMEMBER EXACT STATEMENT & OPTIONS)

184. DEC GFR EXCEPT...


I DID: CONSTRICTION OF EFFERENT ARTERIOLE

185. SCENARIO DIAGNOSE..


I DID: POLYCYSTIC OVARIAN DISEASE

186. BCQ ON PROBAND...


DON'T REMEMBER OPTIONS
THERE WAS: START OF NEW GENETIC DISEASE IN THE FAMILY

187. TRACHEA ENTERS IN THORAX RIGHT SIDE RELATION..


I DID: LYMPHATICS..

188. CONGENITAL ANOMALY FAILURE OF SEPARATION OF AORTA & PULMONARY ARTERY...


ANS. TRUNCUS ARTERIOSIS

189. REGARDING END ARTERIES...


ANS. PRESENT IN VITAL ORGANS.

190. SCENERIO ON VITAL CAPACITY

191. NEUROVASCULAR BUNDLE..


ANS. LOWER BOARDER OF RIB

192. COUNSELING: HELP THE PEOPLE TO HELP THEMSELVES.

193. SHARP PAIN NOT INCLUDED:


I DID: MEDIAL LAMINSCUS..

194. PANCREATIC DUCT : HAVE SPHINCTER..

195. ACTIVE TRANSPORT:


I DID: NOT FOR ALL DRUGS.

196. VOLUNTARY MICTURATION...


I DID: BY PARASYMPATHETIC

197. INC AIRWAY RESISTANCE...


INC R.

=-=-=-=-==-=-=-=-==-=-=-=-==-=-=-=-==-=-=-=-==-=-=-=-==-=-=-=-==-=-=-
207

4. Medicine November 2014 by Mitral Valve(195)


I Hav tried my best to remember all options..
This iz max I cud do 
(Medicine & Allied)

1. Diabetic pt develops sensory abnormalities of left leg & ulcer of big toe...
A. Angiopathy with Neuropathy.
b.angiopathy
c.arteriosclerosis

2.Pt has pain in region of auricle wch bcums worse on chewing.nerve involved?
a.auriculotemporal nerve
b.glossopharangeal nerve
c.infraorbital nerve
d.ant sup alveolar nerve

3.Rh agglutinin
a.is presnt on surface of RBCs
b.is presnt in Rh +ve blood grps
c.is produced in Rh-ve mother when she conceives as Rh+ve baby
d.Rh-ve individual has D antigen

4. Which of the f/w is not a tumor marker:


a.Alpha 1 antitrypsin
b.vesmin
c.desmin
d.AFP

5.the main cause of increased blood flow to exercising muscles


a.raised b.p
b.vasodilatation due to local metabolites
c.inc sympathetic discharge to peripheral vessels
d.inc heart rate

6.Fight n flight response results in


a.dec airway resistance
b.inc resistance of airway
c.pupil constrction
d.none of above

7.Growth hormone secretion is increased by


a.exercise
b.cortisol
c.glucose
d.REM sleep

8. Thirst is reduced by...


Increase Extracellular volume
.inc rennin
Dec extra cellular volume
208

9. Another function of Smooth Endoplasmic Reticulum:


a.Detoxification
b.protein synthesis

10. An old women presented with weakness, palpitation & Breathlessness on exertion, tingling of feet, Hb
low Tlc & platelet level given, Diagnosis:
a. Vitamin B12 deficiency
b.folic acid deficiency
d.cobalamine def

11. Involved in Hydroxyproline & hydroxylysin during collagen synthesis...


a.Ascorbic Acid
b.vit D
c.vit B12
d.vit B6

12. Male pt with facial nerve palsy along with intension tremors on same side, site of lesion...
A.Cerebello pontine angle
b.pons
c.internal capsule
d.midbrain
e.cerebral cortex

13. Pharmacological block of HCL by H2 antagonist...


A.Inhibits both gastrin induced & vagally mediated secretion of acid.
b.inhibits gastrin induced secretion of acid only
c.inhibits vagally mediated secretion only

14. Cardiac valve lesion, Angina common in...


A.Aortic stenosis
b.aortic regurgitation
c.mitral stenosis
d.mitral regurgitation

15.Stimulation by Injection of Parasympathetic, dec in...


A.Size of pupil
b.intestinal motility

16. Aldosterone secretion greatly stimulated by Rise in..


A.Plasma K" level
b.renin level
c.acth level

17. In menopause...
A. Dec Estrogen Increase in FSH & LH.
b.dec estrogen decrease Fsh n Lh
c.inc estrogen decrease in FSH & LH
209

18. After Inability to void urine...


A. S2,3,4
b.S1,2,3
c.S1,2
d.S3,4

19. Pt present with pain & inability to abduct shoulder joint, tenderness over greater tuberosty, after initial
abduction of 40°, he is able to abduct his shoulder...
a.Supraspinatus injury
b.deltoid injury
c.teres major
d.serratus ant

20. Hydrocephalus most commonly due to obst of...


a.Aqueduct of sylvius.
b.foramen of monro
c.foramen of luschka
d.foramen of megendie

21. Loss of visual acuity & field in thyrotoxic opthalmopathy...


A.Optic nerve compression
b.muscles trapping

22. Compression of optic chiasma...


A, Bitemporal hemianopia
b.rt homonymous hemianopia
c.lt homonynomous hemianopia

23. Pt with Bitemporal hemianopia lession is at...


A. Optic chiasma
b.optic tract
c.optic nerve

24. Wasting of thenar eminence & weakness of abduction & oposition of thumb...
A. Median nerve
b.musculocutaneous nerve
c.ulnar nerve

25. Pregnant lady suffered frm measels, anomely expected in baby...


A. Congenital catarct
b.pneumonia
c.PDA

26. Male develop jaundice aftr use of drug given by quake, enz raised...
A. Alanine transaminase
b.ASt
c.ALP
d.GGT
210

27. Pt of DKA not respond to therapy & died, fungal infection, most likely orgnsm...
A.Mucor
b.candida

28. Granuloma with caseating necrosis, most commonly found in...


A. TB
b.sarcoidosis

29. Splenectomy done chance of infection...


A. Strep: Pneumoniae
b.H.influenza

30. Fisherman develops lethrgy, fatigue & palpitation,


Taking fish in diet, O/E Pallor, loss of touch......
A.Diphylobothrium latum
b.echhinococcus

31. Pregnant lady present with jaundice, total bilirubin 11, direct 9, indirect 2, has Gall stones, which
enzyme ll be raised...
a.GGT
b.Alkaline phosphatase.

32. A boy presented with skin rash, fever, joint pain, group of localized lymph node enlargment after the 7
days of IVG injection...
A: Type-3
b.Type 2
c.type 1
d.type 4

33. Old female has Necrotic lession in nose, urine exam revealed proteinurea, biopsy shows granuloma
with necrosis & vasculitis, diagnosis...
A.Wegners Granulomats
b.leprosy
c.beurgers

34. Old male presented with sore throat & cervical lymphadenopathy. Blood picture shows Atypical
lymphocytes, useful initial test would be...
a. Monospot test
b.biopsy

35. A lady frm village hx of dificult labour, presented in ER with bleeding P/V & Oozing frm gums fr last 7
hrs.
A.DIC
b.pph
c.APH

36. Chronic hemolysis, endogenous pigment...


A. Haemosiderin
b.melanin
c.bilirubin

37. Most potent chemotactic agent...


211

A. C5a
b.c3a
c.c3b

38. Parasite associated with cholangio-carcinoma...


A. Clonorchis sinensis
b.

39. Most reliable microscopic finding of Chronic hepatitis...


A.Fibrosis
b.inc leukocytes

40. A child presented with rashes started around ears then spreads over face then trunk & limbs, posible
diagnosis...
a.Measles
b.chicken pox

41. Most reliable Investigation/finding in the diagnosis of TB...


a. Caseating granuloma
b.demonstration of AFB

42. Diagnostic feature of plasmodium falciparum...


A. Black water fever

43. Cereberal malaria...


a.Plasmodium Falciparium
b.pl.vivax

44. Gas gangrene cause of death...


A. Toxemic shock
b.necrosis
c.septic shock

45. Most common cause of Metaplasia...


a.ultrasonography
b. Chronic irritation
c.infections
d.radiations

46. Most common route of infection of HBV & HCV...


a.Blood transfusion
b.contaminated needles

47. Most common promoting factor for the development of liver cirrhosis in Adult male.
a.Alcoholism
b.hep B
c.Hep C
d.Hep A
212

e.hep D

48. Young pt with generalized edema & proteinurea 6gm/day, defect at which area..
A. Basement membrane
b.glomerulus

49. Most useful parameter for diagnosis of SLE...


A.Antibodies against Ds DNA
b.ANA
c.biopsy

50. Old farmer while doing sterneous working in very hot sunny day suddenly colapsed
Plasma Na level was low, cause...
A. Excessive sweating
b.dec sodium
c.dec osmolarity

51. Boy brought by his mother with complain of loss of apitite, vomiting & yellow sclera, inv of choice...
A. Bilirubin+ALT
b.ALT
c.AST
D.ALP

52. Type 3 hypersensitivity reaction...


A. Post streptococal glomerulonephritis
b.hemolytic anemia
c.contact dermatitis

53. Cause of localized edema.


A. Allergic condition
b.ccf
c.nephrotic syndrome

54. Most likely complication in mouth due to chewing Betal nut & pan...
A. Submucous fibrosis
b.mouth CA

55. Alleles are...


A. Non-identical genes at same locus.
b.identical genes at diff locus

56. OT technician has acute eczematous dermatitis on hands & wrist in the area covered by latex
gloves...
A. Type-4
b.type 1
c.type 2
d.type 3
213

57. Hemolytic disease of newborn caused by RH blood group incompatibility...


The mediater of disease...
A. IgG antibody
b.IgM antibody
c.IgA antibody

58. Loss of which one of the f/w class of molecules on the surface of a tumor cell target would result in
lossof susxeptibility to killing by host immune cells...
A. MHC-Class 1
b.MHC-Class 2

59. Oral contraceptive pills


Common complication...
A.Breast Ca
b. Thromboembolism

60. K/c of Tuberculosis on ATT therapy develops Big toe swelling with pain, drug culprit...
A.isoniazid
b.rifampicin
c. Pyrazinamide

61. Anticoagulants contraindication...


A.Thrombocytopenia
b.MI

62. Barr body test diagnostic for


A. KlineFelter syndrome
b.Turner Syndrome

63. Epigastric pain due to Gastric ulcer, nerve involved?.


A. Greater splaanchic
b.lesser splanchnic

64. Monitoring of Blood level for Antibiotic, routenly indicated fr...


A. Vancomycin
b.cyclosporine
c.omeprazole
d.metoclopramide

65. Staphylococus aureus infection DOC...


A.Ampicillin
b. Cloxacillin

66. Source of infection lies infront Neck anterior to Pre-tracheal fascia, infection can spread upto...
A. Anterior mediastenium
b.superior mediastinum

67. Regarding Kallman Syndrome...


a.hyperosmia
214

b.hypergonadism
c.arcuate nucleus involved
d.kall gene on y chromosome

68. Barrier seprates blood of mother from fetus etc...


a.syncitiotrophoblast & fetus endothelium
b.cyto & fetus endothelium

69. Secretion of ADH is inhibited by...


A. Alcohol
b.inc osmolarity

70. During exercise venous blood return frm legs by...


A. Contraction of muscles
b.gravity

71. Blood vessels contain most of the systemic blood...


A. Veins & veinules
b.arteries
c.capillaries

72. Regarding Post: fontanelle...


A. Area is smaller as compared to anterior fontanella.
b.can b felt in occipito post position

73. Basal Ganglia


a. initiate movements

74. Diagnostic microscopic criteria of malignant tumor...


A. Invasion
b.(metastasis)
c.pleomorphism

75 difference b/w Malignant n premalignant lession


a. Pleomorphism
b.invasion

76. Regarding Pituatry gland..


a.Send venous drainage to Dura.
b.ant develops from oral ectoderm

77. Damage to tissue by Radiations mostly depends on...


A. Duration of exposure.
b.type of radiation

78. Trauma case pelvic fracture with fracture of femor & tibia, pulse 110, BP 60/40mmHg, immediate
management...
A. IV Fluids..
b.secure airway
215

c.tracheostomy

79. Heparin given to pt of DVT it will prevent..


A. Propagation

80. Major cause of Atrophy..


A. Loss of innervation
b.disuse

81. Early Stage of Aschoff body shows...


a. Eosinophils..
b.is Fibrinoid necrosis

82. Secondry Hyperaldosternism...


A. Renin
b.aldosterone
c.angiotensin

83. At the end of Merathon race...


A. Dec insulin, increase Glucagon.
b.inc insulin dec glucagon
c.both inc
d. both dec

84. Vasoconstriction caused by...


A. Angiotensin
b.renin

85. Left ventricular hypertrophy...


a.Aortic valvular disease
b.pulmonry valvular disease

86. Most common position of Appendix...


A.RetroCecal
b.pelvic
c.paracecal

87. +Ve tendnbrg test...


A. Superior gluteal nerve
b.inf gluteal nerve

88. A pt comes after 4 hrs with allergic reaction after bee stung...
a. IM Adrenaline
b.iv cortisone

89. Alpha fetobprotein rise in..


A. Testicular carcinoma
b.embryonal ca

90. Hypocampus region damage...


216

A. Unable to convert short term memory into long term memories

91. Proteinurea, basement membrane disruption...


a.AG ratio
b. Albuminurea

92. Multiple sclerosis case


Boy has complain of bed wetting, treatment...
a. oxybutyrin

93. Prozone:
a.high titer Antibodies

94. Indirect psyco-physiologic brain activity...


a.sweat gland activity...
b. Pupillary size or Reaction

95. Ashmatic + IHD pt on OT BP should be manage with...


a.iv nitroproside
b. iv nitroglycerine

96. Regarding pain?


a.at extreme temprature pain receptors stimulated

97. Wide spread pulmonary infarction, dyspnea, shock...


a.embolisation
b.Thromboembolism

98. Bioavailability taken for


a. Oral route
b.iv route
c.im route

99. Inidence...
a.New cases
b.new plus old cases

100. Prevelence...
a.new cases
b.total no of cases except sudden outbreak cases

101. Diarrhea...
A.Metabolic acidosis with normal anion gap.
b.metabolic acidosis with increased anion gap

102. Hyline cartilage what is visible...


a.collagen & elastic fibers (dnt know correct ans)
b.collagen fibers only
c.elastic fibers only
d.none

103. After accident of young boy 55% lung collapsed...


217

a.atmospheric pressure equal to intrapleural pressure


b. High V/Q ratio

104. DIC initiated by...


a.Thromboplastin

105. Most sensetive test fr Diabetic nephropathy...


a.urine albumin
b.urine for proteins

106. Artery accompany phrenic nerve may be at risk during surgery...


a.pericardiophrenic
b.

107. Regarding apex of heart..


A. 8cm away frm mid sternum

108. 6cm mass infrnt thigh


Biopsy most commonly shows...
a.vimentin
b.desmin

109. A man stand straight from 1 hr..


a. Inc CVP
b.dec cvp

110. Severe hypomagnesemia...


a.hypocalcemia
b.hypercalcemia

111. Hard keratin found in...


a.Nail bed
b.hair

112. Hyperthyroidism snerio


Inc T4 level etc what ll decrease...
a.TSH
b. reverse T3

113. Dec level of 0xygen in arterial blood but O2 tension not affected...
a.Anemia
b.co poisoning
c.hypoxia

114. DNA also found in...


A.Mitochondria
b.ribosomes
c.nucleolus

115. Large no of Mitochondria found on the surface of...


a. Cilia
b.liver cells
218

116. Platelate adhesion..


a.Rough endothelium

117. Regarding Dead space..


a. humidification, warm the air etc
b.inc in tracheostomy

118. Anaphylactic & hypovolmic common finding..


a.tachycardia
b.hypotension

119. Cardiac cycle phase b/w S1 & S2...


A Ventericular systole
b.atrial systole

120. Herneated disc compress...


a. Posterior root
b.postero laterally

121. Regarding root value..


a. Knee joint L5
b.biceps jerk c6
c.triceps jerk c8

122. Product enter Krebs cycle...


a.Acetyl CoA

123. Factor not involve in internsic pathway...


A. Factor Vll
b.factor X

124. Regarding blood suply of heart which statement is correct...


A. LCA gives LAD & LCX

125. Spinal anesthesia.given person bcums unconscious.cause?


A. Loss of vasomotor tone

126. Drugs absorption icreased by...


a. Inc Lipid solubility

127. Secretion of Insulin stimulated by...


A. GIP
b.glucagon

128. Old man died after schemic attack, atopsy of Brain Slice shows...
A. Liquifective necrosis
b.coagulative necrosis

129. Brocas area


A. MCA
b.ACA
c.PCA

130. Meningitis dec level of Glucose found in...


A. Pyogenic Meningitis
219

131. Mucus acini highest in.?..


A. Sublingual Gland
b.gastric glands
c.parotid gland

132. Dec no of T Cells found in...


a.: Autoimmune disease
b.bacterial infections

133. About Major/ dangerous disease of child admitted in ward.. parents of the child response...
a. Denial
b.depression

134. Regarding CCK all are correct except...


A. Inhibit pancreatic secretions
b.inc gall bladder secretions

135. Snerio: Loss of motor function on Right side...


Lession at...
a. Left internal capsule
b.right internal capsule

136. Loss of pain & temp on Left side, & loss of touch, vibration on Right side.
a.Right Hemisection of cord
b.left hemisection of cord

137. Transport through cell membrane involving Actin, myocin & clathrin...
a. Pinocytosis
b.secondary active transport
c.facilitated diffusion

138. Jaundice on 2nd day after birth total bilirubin 9...


a. Physiological jaundice
b.breast milk jaundice

139. Regarding Recurrent laryngeal nerve...


a.Branch of vagus nerve.

140. Regarding ECG all are correct except...


a.P wave absent in Atrial flutter
b.St segment is from peak of S wave to start of T wave

141. Regarding cardiac blood flow..


a.regulated by local metabolites
220

142. Regarding Vagus nerve


a.Left recurrent laryngeal nerve hooks around ligamentm arteriosis
b.

143. Thyroid gland enclosed in...


a.Pretracheal fascia
b.investing facia
c.vertebral fascia
d.carotid fascia

144. Hypokalemia...
Ans. U waves on ECG

145.in Complet Heart Block...


a.Fainting may occur bcz unable to pump blood into ventricles.
b.atrial rate is lower than ventricular rate
c.ventricular fibrillation is common
d.fainting occurs bcuz of prolong periods during wch ventricles fail to contract.

146. Parknson pt on L-Dopa treatment eventually wears off bcz...


A. Dopamenrgic neurons in Subtncia nigra continue to degenerate. (ganong mcq)

147. Sq cell carcinoma on buttock lymph drain into...


a. horizontal group of superficial inguinal lymph nodes
b.vertical grp of superficial inguinal nodes
c.popliteal nodes
d.deep inguinal nodes

148. Hormone of slow wave sleep..


A. Serotonin
b.Ach
c.epinephrine

149. Neonate with small eyes etc...


A. Trisomy 13
b.trisomy 21
c.trisomy 16

150. Regarding Kidney relations...


a. Left renal vein is long & anterior to renal artery
b. right renal vein is long & anterior to renal artery
221

151. Diazepam+ chlorpromazepine...


a. sadation
b muscle relexation

152. C-ANCA...
A. Vasculitis
b.arthritis

153. Pt with uremic nephropathy, what would not seen ?


a. Hypophosphotenemia

154. Child of 6 week age presented with jaundice, his dipper stained with Bilirubin...
A.Gilbert Syndrome
b.biliary atresia

155. Atheletic after injury unable to Invert the foot but able to evert, rupture of....
A. Tibialis Ant & Tibialis posterior.
b.peroneus longus& flexor hallucis longus

156. Which of the f/w has highest PH...


A. Pancreatic juice
b.gastric juice
c.salivary secretions

157. Loss of taste Post 1/3 of tounge...


A. Glossopharyngeal
b.vagus
c.facial

158. True Regarding receptors..


a: Golgi tendon activated by muscle tension
b.muscle spindle activated by golgi tendons

159. Deep sea diver rapidly ascend pain in joints...why?


A. Emboli in Arteries
b.emboli in veins

160. All are the findings in the Pt of congestive heart failure except...

a. capillaries refill
222

b.splenomegaly
c.hepatomegaly
d.pulmonary compliance

161. 6 yrs boy 17-ketosteriod in urine, enlarge adult size penis, hairs on body...
a: congenital adernal hyperplasia
b.adrenogenital syndrome

162. Regarding staging of tumor...


a: Extent of invesion

163. O2 delivary to the tissues decrease by...


A.Increase PH
b.dec pH
c.inc H+ conc

164. Regarding circle of wills..


a.connects internal carotid + posterior comunicating artery
b.connect ant communicating to post

165. Gastric acid activate...


A Pepsinogen
b.trypsinogen

166. Regarding Esophagus...


A. Begins at thyroid cartilage

167. Buccopharangeal memb separtess


A.nasal cavity from oral
b.Stomedium from pharynx/ foregut

168. Regarding yolk sac...


a. contain initial blood marrow

169. Catecholamine derived from...


A. Tyrosine

170. N.Meningitis DOC...


a.ceftriaxone
b. Pencillin.G

171. Cholera, Path physiology...


a. Activate adenyl cyclase
b.activates chloride channels

172. Alpha-1 receptor activation...


A. Mydriasis
b.meiosis
c.inc gi motility

173. Thyroid cancer, tumor marker...


223

a: Calcitonin
b.PTH

174. Ectopic ACTH.from?


a.squamous cell Ca
b. Small cell carcinoma

175. Proton pump inhibitor..


a. Omeparazole

176. Recurrent abortions, thrombosis etc not respond on treatment...


a: Factor V laden
b,Antiphospholipd syndrome

177. Aspirin overdose side effect...


a.coma
b.metabolic acidosis

178. in middle cranial fossa...


a.Trigeminal ganglion residing thre

179.wch nerve present inside Cavernous sinus...?


a.Abducent nerve
b.occulomotor N
c.trochlear N

180. Regarding serotonin syndrome...


a: TCA+SSRI

181.bst test for Asthma..


a: FEV1
b.PEFR

182. Dec GFR except...


a. constriction of efferent arteriole

183. Pt with hirsuitism.on scan multiple cysts


a.polycystic ovarian disease
b.polycystic kidney disease

184.Proband...
a.start of new genetic disease in the family

185. Congenital anomaly failure of separation of aorta & Pulmonary artery...


A.Truncus Arteriosis

186. Regarding end arteries...


A. Present in vital organs.
224

187. Max amount of air inspired n normaly expired


a.Vital capacity
b.IVC
c.IRV

188. Neurovascular bundle..


A.Lower border of Rib
b.upper border of lower rib

189. Counseling:
a.help the people to help themselves.
b.help ppl to help others

190. Sharp pain not include


a. medial laminscus..
b.spinothalemic tract

191. Pancreatic duct :


a. Have sphincter..

192. Active transport:


a.not for all drugs.
b.non saturable

193. Voluntary micturation...


a. by parasympathetic
b.

194.Wch one of following is chemical carcinogen?


a.alpha anti trypsin
b.benzidine
c.ethyl alcohol
d.propyl alcohol

195.pt has undergone surgry for adrenal gland tumor.He will have increasd threshold for wch of following
taste sensation?
a.salty
b.sweet
c.bitter
d.sour
JIZAKALLAH
BEST OF LUCK!!
REGARDS
MITRAL VALVE!!!!!! 

=-=-=-=-==-=-=-=-==-=-=-=-==-=-=-=-==-=-=-=-==-=-=-=-==-=-=-=-==-=-=-
225

5. Medicine Online 28th August,2014 by Dr.Sidra Majeed (27)

1. A 40 year old teacher while writing on blackboard is suffering from shaking of his hands. His cousin has same problem…Pipe lead rigidity and
other sign not reported…all other signs are unremarkable…Drug should be given is ROPINIROLE.

2. Which has got soft keratin with having no nucleus


A. St. Corneum (ANSWER)
B. St. Basalis
C. St. basalis & st. spinosum
D. Hairs follicle
E. base of the nail

3. In anemia , blood becomes turbulent due to


Decrease blood viscosity (ANSWER)

4. HIV drug causes pacreatitis and peripheral neuropathy is


Didanosine (Answer)

5. Anaphylactic shock comparison with Hypovolumic shock has


Increase cardiac output (answer)

6. Adenoma has
Glandular cells ( answer)

7. Anterior SURFACE of the heart is formed by Right Ventrilcle (anwser)


( remember the question was about surface NOT about border…anterior border of heart is formed by Right atrium)

8. A scenario of VMA found in yrine …Pheochrocytoma (ANSWER)


9. Ovarian cancer marker…CA 125
10. A scnerio of 5 hydroxyindoleacetic acid ……Carcinoid tumor
11. A young man having dysphagia, horness of voice ….case of medullary cell carcinoma…marker is CALCITONIN
12. Increase carbohydrate consumption will increase the demand of
A. Thaimine (Answer)
B. Riboflavin
C. Niacin
13. Scnerio about tremors and rigidity…..are affected…Substantia Nigra
14. Protooncogene is converted to oncogene by
PONT mutation
15. The most common cause of mental retardation is
A. Cri du chat syndrome
B. Fragile X chromosome ( Answer)
C. Edward syndrome
D. Turner syndrome
16. The length of right bronchus is
2.5 cm (answer)
17. Creatinine is important in assessing renal failure…it arises maily from
A. Liver
B. Brain
C. Kidney
D. Skeletal muscle ( answer)
18. Muscle carrying capacity with hemoglobin is
A. Actin
B. Troponin
C. Myoglobon (Answer)
B. Tropomyoglobin
19. Increase GFR due to
Dilation of afferent arterioles
20. Hamartoma is defined as ?
A. mass of mature disorganized tissue indigenous to that organ (Answer)
B. ectopic rest of normal tissue
C. a tumor that can metastasize
D. a malignant tumor that can cause serious complications
226

E. Totipotent cells
21. Flare due to arteriolar dilation is due to AXONAL REFLEX
22. A new born baby is having complain of passing urine from umbilicus to urachus…
A. ectopic ureter
B. Ectopic urethera
C. persistent allantois (ANSWER)
23. A pateient having CVP line in subclavian vein & on parental nutrition. He develop fever.Blood CP shows cluster of gram +ve cocci. The drug
of chaoice for this organism is
A. Amoxicillin + Clavulanic acid (ANSWER….Rabbia ali mcqs PAGE 443..qs # 246…..2ND EDITIOIN)
B. Vancomycin
C. Ceftazidim
D. Imepenam
E. Piperacillin + Tanzobactam
24. True about cerebellum
A. Connected to medulla through middle cerebellar peduncle
B. Resting tremors occur after injury
C. Lies below tentorium cerebellum (ANSWER)
D. Ataxia not occur after this lesion
25. Melanocytes are derived from
A. Endoderm
B. Ectoderm
C. Neural crest cells ( Answer)
D. Pharyngeal arch
E. Lateral plate
26. Primordial germ cell of yolk sac are derived from
A. Ectoderm
B. Mesoderm
C. Endoderm (answer) ….Rabbia ali BCQ page 140
27. Which of the following would produce maximum excitation of the hair cells in the right horizontal semicircular canal?
A. Hyperpolarization of the hair cells
B. Bending the stereocilia away from the kinocilia
C. Rapid ascent in an elevator
D. Slow ascent in an elevator
E. Rotating the head to the right (ANSWER

=-=-=-=-==-=-=-=-==-=-=-=-==-=-=-=-==-=-=-=-==-=-=-=-==-=-=-=-==-=-=-
227

6. Surgery 12th Febuarary 2014 by Dr. Asad-Ur-Rehman (200)


FCPS PART-1 12TH feb 2014 Sugery paper 1 and 2 compiled by Dr. ASAD-UR-REHMAN
No body knows the exact answer in CPSP key and every one is defending his answers but I have
compiled paper with best suitable answers on basis of discussions in this page. Those questions
in which more than one answer is written the option written first is answered by most of
candidates according to group discussion.
1. Percentage…dividing unit by total units and multiply by 100
2. Confidentiality can be breached….when patient allow it
3. patient developed a fatal disease…give clear crisp evidence based information according to
patient need
4. Glycolysis…glucose breakdown to pyruvate aerobically and lactate anaerobically
5. True about collagen….present in all type of connective tissues
6. turner…….44xo
7. klinefilters syndrome……..46XXY
8. vitamin in oxidation of fatty acids….biotin
9. ESR…………..increase in infection
10. BSF=122 1hr gtt=192…..impaired GTT
11. diagnostic test of beta hemolytic streptococcus……………ASO titre/ blood culture
12. not of epithelial origin……liposarcoma
13. decrease B.P……decrease carotid sinus firing
14. dysarthria is feature of lesion in ……….cerebellum/ brocas area
15. secondary center of ossification represent….epiphysis
16. common complication of diabetes in limb…..dry gangrene
17. definition of acute inflammation….local response of living tissue to injury/systemic response
to injury
18. 1st heart sound during…….isovolumetric contraction
19. cause of oedema………..lymphatic obstruction
20. supply of sternocleidomastoid….acessary nerve
21. most drugs metabolized in….liver
22. dyspnea on laying….retrosternal goiter
23. cause of edema in nephrotic…hypoproteinemia
24. better hearing in noisy environment….otosclerosis
25. sebaceous gland not in which hairless area of skin…..palm and sole
26. vein accompanying LAD artery……..great cardiac vein
27. pain between eye and angle of mouth and temporal….maxillary division of trigeminal nerve
28. EBV cause…….nasopharyngeal carcinoma
29. microtubules present in…….centrioles
30. stretch of muscle resulting relaxation……….golgi tendon
31. fast sharp pain carried by…….A delta fibers
32. 3rd ventricle….lateral boundary by ant 2/3 rd of thalamus
33. parasympathetic cranial nerves…III,VII,IX,X
34. 16 yr female mcv 78, mch 25, MCHC…..MICROCYTIC HYPOCHROMIC
35. Right shift of hb-O2 curve in …….acidosis
36. fever,pain on lying down……..pericarditis
228

37. primary opsonin………C3b


38. symphisis pubis…………………sec. cartilaginous joint
39. pain from both fibrous and parietal pericardium by………………phrenic
40. least metastasis to………….spleen
41. fat necrosis…………….acute pancreatitis
42. severe hypomagnesemia cause………….hypocalcemia
43. H+ secretion in PCTs coupled with…….bicarb reabsorption
44. compression of S1 root……loss of ankle jerk
45. not related to rt. Kidney…………..descending colon
46. heat lost in an uncovered pt. in OTat 23C……………..radiation and conduction
47. 67.S2,S3,S4 lesion………….rectal incontinence
48. tumour comprising all the germ layers……………teratoma.
49. at rest actin covered by………tropomyosin
50. GFR in clinical setting……….creatinine clearance
51. greenish discharge from wound…….. pseudomonas
52. cause of death in gas gangrene………….toxemia
53. diagnosis of typhoid in 2nd week ……widal test + blood cultures
54. antibodies formed by …..plasma cells
55. plasmodium falciparum……..black water fever
56. langarhans giant cells……..T.B
57. hamartoma……totally beningn
58. nodule on voccal cord unlikely……..atrophy
59. alkalinization of urine done in overdose of……..phenobarbitone
60. pre ganglionic sympathetic fibers secrete………Ach
61. parasympathetic promote……salivary gland secretions
62. common site of ectopic pregnancy…………. fallopian tube
63. vitelline duct…………meckel diverticullum
64. both central n peripheral chemoreceptors respond to……increase in arterial pCO2
65. death on 6th post MI day……………cardiac tamponade
66. most sensitive cardiac marker……..trop-T
67. primary hyperthyroidism……increase TSH
68. left suprarenal vein drain in…….left renal vein
69. PAIN OF PLEURA to shoulder…..C3 4 5
70. fastest B.P Control mechanism………baroreceptor regulation
71. severe reaction when A+ patient transfused with………….AB-
72. single common local factor delaying wound healing………infection
73. energy fuel after 48hrs of starvation…..triglycerides/liver glycogen
74. 2L of volume lost in sweat and 2L distilled water given result is …….inc ICF volume
75. most active gluconeogenesisis from…..proteins
76. compact bone histology……lamella regularly arranged
77. 60 yr chronic smoker working in tyre factory and transitional S C…..cigarette
smoking/aromatic amines
78. false about statistics….statistical values have not influence practical values
79. fluid given in severe dehydration………………5% glucose solution/ 0.9 normal saline
229

80. concentration gradient mediated tranport of glucose……..diffusion/facilitated


81. not found in dermis………melanocytes/reticulocytes
82. ACTH effectively control…….hydrocortisone/adrenal androgens
83. true about bronchopulmonary segments…..each having independent blood supply
84. effective dose of drugs bound to alpha 1 acid glycopro not affected in….. MI/hepatic
disease/trauma
85. parasympathetic cause……inc the AV nodal delay/ increase k effluxs in sa node
86. inhibit insulin secretion…………beta blockers/glucagon/secretin
87. RBC fragility……completely destroy in 0.35% solution/certain drugs cause RBCs hemolysis
88. dilation of arterioles…increase venous volume
89. Monocytes….can transform in to multinucleated gaint cell in chronic infections
90. 10 %formaldehyde used for sending biopsy specimen becoz……… it prevent of
autolysis/denature proteins
91. post communicating artery……connect internal carotid n post cerebral passing under
occulomoror nerve
92. lowest pressure in aorta durng cardiac cycle… isovolumetric contraction /isovolumetric
relaxation/atrial sysole
93. true about hydatid cyst…transmitted to man by infected water
94. corona radiataformed by……………….granulosa cell
95. female vaginal discharge having growth in vagina received diethylstilbestrol during 1st
trimester….clear cell carcinoma
96. which one is malignant…………………glioma
97. external carotid artery……………lateral to mandibular vein
98. loss of 2L by in marathon replaced by pure water…..decrease extracellular osmolarity
99. increase interstitial fluid pressure……………….increase capillary permiability
100. increase in venous return not increase…. pulmonary pressure/left ventricular tension
101. fibrous dysplasia most common in…….maxilla/temporal/frontal/spenoid/ethmoid
102. liquefactive necrosis…..brain
103. STERILIZATION….121C FOR 3 MINUTES KILL ALL ORGANISMS
104. ATYPICAL MICOBACTERIUM……NOT RESPOND TO ROUTINE ANTITUBERCULOUS
105. PSEUDOMEMBRANOUS COLITIS……C.difficile
106. CARRIES….LACTOBACILLUS
107. FILAMENTOUS ORGANISM…….ACTINOMYSIS
108. By blood transfusion…HCV
109. Broodie abcess…..osteomyelitis
110. UTI NO PENILE LESIONS LEUKOCYTE ESTERASE +VE ORGANISM…CHALMYDIA
111. SEPTIC ARTHITIS…..STAPH AUREUS
112. OCULAR TOXICITY…ETHAMBUTOL
113. SEVERE BLOOD TRANSFUSION REACTION…MISMATCHED BLOOD
114. BAD SIGN OF GRAFT…..THROMBOSIS and vasculitis
115. RADIATION….HEMOPOITIC SYSTEM
116. DYSPLASIA…HPV
117. PROSTATIC INTRAEPITHELIAL NEOPLASIA….MONITOR PSA
118. AFLATOXIN…HCC
230

119. LIVER ADENOCARCINOMA…alpha fetoproteins


120. Tumor.Increase cathecolamines in urine child…neuroblastoma
121. Hypothalamus,….not control respiration
122. Fainting episodes investigation tobe done….insulin and c peptide
123. Renal stones,punched out lesions on x ray…hyperparathyroidism
124. hyperthyroidism in pregnancy….propylthiouracil
125. increase ca absorption from kidney….parathyroid
126. Residual volume…1200
127. not in CRF….hypophosphetemia
128. most common TEF….blind esophagus and distal communicate with trachea
129. cloacal membrane….ectoderm and endoderm open at 7th weak
130. 2nd part of duodenum anteriorly…..transvere colon mesentary
131. Lower body of epiploic foramen…1st part of dudenum
132. Right suprarenal vein…IVC
133. True about abdominal aorta…related to left sympathetic trunk
134. Left gastroepiploic artery….splenic artery
135. Parasympathetic….increase stomach muscular contraction
136. Relation of femoral artery to vein in femoral sheat…lateral
137. Internal oblique…cremasteric fascia
138. Potent stimulator of salivary secretion….proteins
139. best histocompatibility sample….wbcs
140. thalasemia…..factor viii
141. bleeding circum….increase APTT
142. MENORAGIA PURPURA MEGAKARYOTYPES….ITP
143. OLD LYMPHOCYTES ON SMEAR…CLL
144. DIFIICULT LABOUR INCREASE PT APTT….DIC
145. MCV 128…..MEGALOBLASTIC ANEMIA
146. CRF…ERYTHROPOITIN
147. MOST COMMON DAMAGED CARPAL BONE…SCAPHOID
148. ANATOMICAL SNUFFBOX…RADIAL ARTERY
149. THENAR WASTING…..MEDIAN NERVE
150. WRIST ADDUCTION…both extenser and flexer carpi ulnaris
151. LATRAL BREAST…ANT AXILLARY
152. NOT BY SCIATIC….GLUTEUS MAXIMUS
153. SUBDURAL HEMMORHAGE…superior cerebral vein
154. Common peroneal gamage….LOSS OF EVERSION
155. S1 COMPRESSION….loss of ankle jerk
156. PAIN KILLER FOR ASHAMATIC…paracetamol
157. NOT GEGENERATE MYELIN SHEAT BECOZ NO ….SCHAWAN CELLS
158. OVARIAN….PARAoRTIC
159. LEFT SAGITAL SENSARY AREA damage….RIGHT LEG
160. LACRIMAL GLAND…PARASYMPATHEYTIC BY FASCIAL
161. POST TRIANGLE NECK disection…SPINAL ACESSARY
162. Cricothyroid…….EXTERNAL LARANGEAL
231

163. RENAL VASCULAR SEGMENT…5


164. Conserve blood volume….RENIN
165. SEMINAL VESICLE…join vasferens to form ejaculatory duct
166. CRF SOB….metabolic acidosis
167. HYDROGEN SECRETION IN PCTS…HCO3 REABSORPTION
168. Main support uterus…..TRANSVERSE CERVICALL LIGAMENT
169. HISTOLOGY…PROSTATE
170. RIGHT SHIFT…DEC PH
171. Not related to aortic arch….RT RECURRENT LARANGEAL
172. Pleural effusion on standing…..COSTODIAPHARAMETIC RECESS
173. Embed in Ant wall of carotid sheat………ANSA ERVICALIS
174. FEMORAL SHEAT …FASCIA ILIACA AND TRANSVERSALIS FASCIA
175. Correct about FEMUR…….. SHAFT NECK ANGLE IS 125
176. Tetnus….exotoxin has lethal effect
177. SEMIOMA…poor prognosis inpulmonary metastasis
178. Cast applied to leg Ant shin syndrome…….ANT TIBIAL ARTERY
179. Small cell carcinoma…..level of cortisol/decrease Na in plasma
180. True about left coronary artery…give circumflex artery
181. true about histology of GIT…Striated border microvilli 4 options were correct question was
wrong
182. HEALTHY MALE EJACULATE… bulk formed by seminal vesicle secretions/ no bacteria
183. DEFECATION involve ….parasympathetic sacral segment/intrinsic myenteric reflex
184. PULSE OXIMETERY give wrong reading…. Methemoblobinemis/black coloured people
185. CNS TUMOR CHEMO RADIO DEATH AFTER 8 MONTH PULMONARY THROMBUS….cardiac
metastasis
186. ADVANCE DRUG TO REVENT FORM RENAL HEART LIVER
TRANSPLANTATION…MURONOMAB CD-3 ANTIBODY OKT3/cyclosporine/ANTILYMPHOCYTIC
GLOBULINS
187. Safe dose of local anesthetic….200mg lignocain/50 mg bupivacain
188. Medial necrosis on autopsy…. disecting aneurysms/ berry aneurysm
189. Peripheral golden brown mass.macropage filled alveoli….malignant
mesothelioma/SILICOSIS
190. locally malignant…ameleoblastoma/plemorhic tumors
191. VERTEBRAL ARTERY….pass from foramen transversarium
192. BREAST SAMPLE FOR FLOW CYTOMETER to determine….malignant cell/ kary
otype/aneulpoidy
193. Achalasia cardia…loss of vagal neurons
194. Continuous Pain chest not affected by respiration fever 10
day…myocardium/costochondral junction/ pleura/pericardium
195. NOT PASS FROM GREATER SCIATIC FORAMEN…OBTURATUS INTERNUS
196. FLOOR OF MIDDLE CRANIAL FOSSA…greater wing of sphenoid
197. SPARING OF LATERAL THENAR EMINENCE….ulner nerve
198. LIPS BUCCAL MUCOSA LESIONS…..HSV TYPE I
232

199. HIV related lymphoma…EBV


200. NOT TRANSMITTED BY ARTHROPODS….dracunculous
=-=-=-=-==-=-=-=-==-=-=-=-==-=-=-=-==-=-=-=-==-=-=-=-==-=-=-=-==-=-=-

7. Radiology 12th November 2014 by Sheeraz Ahmed Sheikh(159)


1)intrvel b/w 1st and 2nd heart sound eaquls to?

A) cardiac cycl
B) ventriculr diastol
C) ventriculr systole ANS

2)Loss of sensations of dura in midle cranial fosa due to lesion of?


A.facial nrve
B.glossophayrngeal
C.trigerminal ANS
D.vagus

3)Severe hypomagnesimia?
A.hypocalcemia ANS
B.hypercalcemia

C.Hyperphosphatemia

D.Hypocalcemia

Ehypophosphatemia ans a

4) fracture of pterion causes demage


a. middle meningeal artey ANS
b. sup cereberal atry
c. maxillary artery
d. sup temporal A

5)Apex of heart?
A.Most superior part
B.Directly downward to ryt side
C.In 3rd intrcostal space
D.8cm away from midline ANS

6)other than steroid synthesis smooth ER also helps


A. detoxification ANS
B.protein synthesis

C.

D.

7)thirst inhibited by
a. Ag2
b.alcohol
c. inc plasma osmolarity
d. Increase ecf Volume ANS
e.renin
233

8)baby delivered.. develop jaundice next day.. indirect Bilirubin 9 total Bilirubin 10 .. breast fed.. mom blood positive
group
a. erythroblatls fetals
b. physiological jaundice ANS
c. rh incompatibilty
d. neonatal sepsis
e. breast feed jaundice

9. Structure Passing through Fossa ovalis

A.Femoral artery

B.Femoral vein

C,Great saphenous vein

D.small saphenous vein

MANDIBULAR DIVISION OF TRIGEMINAL NERVE AND LESSER PETROSAL NERVE!!

10. After entering Superior mediastinum structure On.right side of trachea


A.Pleura
BVagus nerve

C.Mediastinal pleura

D.Common carotid artery

11.Max blood volume is found in

A.Caplaries
B.Arteries
CVeins n venls ANS

D.Aorta

12.True about posterior communicating artery in circle of willis?


A. Connects ICA & PCA
B. PCA & MCA
C. passes below oculomotor
D. Passes above the occulmotor

E.Connects ICA and basilar ANS

13.barr bodies is seen in buccal smear for diagnosis of:


a> klienfiltr syn
b> turner syn ans

14.Cholelithiasis in a pregnant lady. reliable test?


a. ALT
b. GGT ANS
c. ALP IF GGT X NOT MENTIONED THAN ALP

15)The children admitted to wards for some irrevesible disability or end stage disease will have parents with a normal
physiological response of:
a) Anxiety
234

b) denial ANS
c) depression
d) projection
e) repression ANS B

16) mucinous acinus rich in


brunners
parotid
sublingual ANS

17.Increased Mitochondria in apical layer which?


A.Cardiac muscle
B.Skeletal muscle
C.ciliated cells ANS
C.steroid secreting cells

D.Liver Parenchymal cells.

18. Erythroblastosis by
A.igm
B.igg ANS ANS IGG

19) TB diagnosis
a. AFB ANS
b. Caseating granuloma

20) Whch is correct? Ak option yad nh arha


Bicep jerk c6 ANS
Tricep c8
Ankle jerk s2 s3
Knee jerk l4 l5

21) Phrenic nerve injurd wd


Musclophrenic artry
Pericardiophrenic ANS
Int thoracic

22) Diarrhea causes


a. Metabolic acidosis with normal anionic gap?? ANS
b. Metabolic acidosis with low anionic gap
c. Metabolic acidosis with high anionic gap??

23) Man won race at the end hormonal level


A. Inc insulin drc glucagon
B. Dec insulin inc glucagon ANS
C. Inc insulic inc glucagon
D. Dec insulin dec glucagon

24) other then nucleus , which hav dna?

A.Mitochondria ANS

B.Golgi

C.Lysosome
235

D.Ribosome

E.Endoplasmic reticulum

25) incusura if uncinate proces is for which artey


a. sup mesenteric? ANS
b. inf mesenteric
c. celiac

26) pancreas ki major artery


a. sup mesenteric
b. inf pancreatosudenal
c. left gastroepiploic
d. gastrduodenal

E.Splenic ANS

27)in ingunal hernia repair most probable structure seen


a. pempino plexus ANS
b. inguinal nerve
c. coopers gland

28) cause of death in gas gangrene ...extensive necrosis/

toxemic shock ANS

29) comon method to detect cancer cell in circulation


a. tumor markers ANS
b. peripheral culture

30 liver disease chronicity is detected by fibrosis na? bqi options yad nai

31. functiona unit of liver


a. hepatocytes
b. portal triad
c. kupffer cels

D.Hepatic acini ANS

32. in portal htn .. dilatation of


a. left colic vein ANS
b. inf epigastric
c. left ovarian
bqi option nai ya option nai yad

33. bare area of liver kahan drain krta…..Celiac Nodes

34. Hyaline cartilage?


Collagen and elastic fibers visiber

/ collagen fiber visible/ ANS

fibers absent

/ fibwrs not visible


236

35. patient with meningitis cured. afyet one mnth came with complaint of heache.. papiledema..
hydrocephalus i chose.. dnt know other options.. is it right? ANS HYDROCEPHALUS

36. av bundle
a. it is nervous tissue
b. only conection btw atria n ventricle ANS

37.Intercostal space.

A.1 anterior intercostals artry

B.1 post Intercostal artery

c.2 ant int costal arteries ANS

2 ANT ARTERIES, 1 POST ARTERY!!

Fall on out streched hand at greater tuberosity can't abduct ,muscle injured?

DELTOID ANS

Kallman syndrome major complain.?

HYPOGONADISM AND HYPOSMIA

Cranial nerve in cavernous sinus?

6TH

Fall on back ,urine incontinence spinal segment involve

S2,3,4

Sore throat lymphadenopath,atypical lymphocytes ...next investigation?

MONOSPOT

Decrease cardiac output


1 ccf ANS

2 beri beri

3 av fistula

4 hyperthyroidism

Extensive fat embolism per tha aik

Early ashkoff presentation


237

FIBRINOID NECROSIS ANS

,esinophil,

giant cell,

fibrosis

Which is a vasoconstrictor
Prostaglandin

Angiotensin ANS

kallerin

Dic initiated by?

THROMBOPLASTIN ANS

Factors in intrinsic pathway?

Headache and facial nerve palsy on right sight with intension tremors? Nerve arise from?

Ponds

cerebellopontine ANS

right cerebellum

,right medulla

right midbrain

Hakim gave some medicine patient came with hepatic necrosis enzyme elevated?

ALT

Pregnant lady with biliary stone enzyme elevated?

GGT

Mcq on pneumothorax

Meningitis in child after autosplenctomy organism responsible?

STREP PNEUMONIA ANS

There was something about prozone?!!


238

Pedigree mein Kuch pro band per tha

Investigation of choice on diabetic nephropathy?

URINE MICROALBUMINEMIA

Child with generalised edema investigation of choice?

URINE ALBUMIN

38. which virus has greater chance of transmision even if the instruments are strlized with alcohol.
a. EBV
b adeno
c HIV ANS
d. herpes zoster
e. herpes simplex

39. which part of duodenum is retroperitoneal


a. first
b. secnd
c. third
d. fourth
e. all excpt first part ANS

40. there are two axilay lymph paths


a. ventral n lateral
b. anterior n superior
c. hepatic n abdominal
d. anterior n posterior

41. Chemical carcinogen....

methyl alcohol/

propyl alcohol/

bezidine?? ANS

42urine protein krwate hain i think.. nephrotic kiun k zada comon bcho ma cause

43. Hep b n c transmitted commonly...

.direct contact,

blood transfuson,

contaminated needle use ANS

44. volume of air expired forcible after deep inspiration is


vital capacity ANS
inspiratory capacity
total breathing capacity
239

45. Surface metaplasia


Chronic irritant ANS

.sunrays

uv rayz...two other one..

46. pancreas
a. they extend from right kidney to spleen
b. they are inferior to stomach
c. sup mesenteric pas anterior to uncinate process ANS
d. they lie at L4 vertebral body level

47.

About coronry artry?


A)Ryt coronry suply av,bundl branches,bundl of hs
2) left gves anterior dcsendng n circumflex branches ANS

48. During exercise decreased TPR is due to


Skeletal muscle

arterioles dilation

due to local metabolites ANS

49. Which factor does not take part in intrinsic pathway


Factor10
Factor7. ANS
Factor 8

50. pituitary
A partly derived from foregut ANS
B veinous drainage directly in dural sinus

C.anterior to optic chiasma

52. folowing not opening in pterygopalatine


a. inf orbital fissure
b. ptrygoid canal
c. sphenopalatine foramen

53. estrogen containing ocp will cause mostly:


a> thromboembolism ANS
b> breast cancer

54. Local edema? Allergy is true?

55. Infection outside pretrachial layer goes to


A.superior mediastinum
B.posterior
C.anterior ANS
D.middle

56. Diabetic nephropathy


240

urinary albumin ANS


Ultrasonography

57. Reed sternberg cells and lacunae cells are seen in


A mixed cellularity Hodgkin lymphoma
B nodular sclerosis Hodgkin lymphoma ANS

58.t rue about ecg,

u waves due to hypokalemia, ANS

inverted t waves due to hyperkalemia

,increase ext cellular na causes depolarization

59. which lesion is associated wth eating betel nut and paan?
a. submucosal fibrosis ANS
b. keratosis
c. ulceration

60. Pyogenic meningitis option thi kisi question ki??

61. CRF..

mormocytic normochromic anemia??

62. Counciling help the people

help themselves ANS

understanding their problems..

63. pt hand abducted.. asked to move hand slowly n smothly downward.. bt he droped hand sudenly by his side..
muscle torn
a. deltoid ANS
b. supraspinatus
c. subscapularis
d. teres major
e. pectoral

64. Mitral valve area..


1-2cm Square
2-4
4-6
6-8

65. Radiation induced CNS Tumor.

A.Glioblastoma multiforme

B.Lymphoma

C.Meningioma

D.Astrocytoma.
241

66. Sensory nerve supply to Uterus via which ligament

A.Broad ligament ANS

B.Round ligament

C.Uterosacral

67. Diabetic patient with loss of sensations of feet n ulcer on left big toe. Cause is

a. Angiopathy with neuropathy ANS

b. Arterial thrombosis

c. Arterial thrombosis with infection

d. Atherosclerosis???

68. glans penis lymphatics to

deep inguinal L.n

69.Genitofemoral Nerve in

posterior relation with Ureter

70,71Pancreata magna.....Splenic..
Incisura pancreas....SMA

72.Incidence

73.Prevalence

74.Pulmonary ligament

75.Urinar bladder which surface not covered by peritoneum…..

BASE ANS

76.5 HIAA in urine….

Carcinoid syndrome

77.Metastasis…

most imp for malignancy

78.Invasion to adjacent structure

79.Cubital tunnel syndrome

MEDIAN NERVE ANS

80. median Nerve gives no branch in Arm??

81. Cephalic vein pierces clvipectoral facia?


242

82. Sibbon facia

83. Right sided leg and upper limb weakness Plus left angle of mouth deviation ?

ANS PICA

84. Intrnal capsule blood supply ..

.chaocot(lenticulostriat Branch of MIDDLE cerebral arter

85. which is anatomically classified as largest prostate lobe?

PERIPHERAL LOBE

86. T12 vertebra?

87. middle ear tegmen tempani

88.Pelvic kidne…asim and chadkin question

89.IVC…3rd Part of duodenum

90.Anterior spinal artery

91. S1...lateral surface of foot

92.Brocas area blood supply MIDDLE CEREBRAL ARTERY

93. Posteroir decending arter...apical part of interventricular septum

94. Sphenous vein 20 valve

95.Breast….pect major under

96. pect major flap arterial supply...

THORACOACROMIAL ANS

97. Pulmonary embloism...

DVT ANS

98..CD4 helper T cell

99.autoimmune disease

100..Wegener glomerulonephritis

101..adrenalectomy…

Sodium chloride prefence

102. yes....left umbilical vein..ligamentum teres

103. Superior Gluteal nerve

104.Prematurity….

.PDA
243

105. yes...left kidney anteror relation...

.Body of pancreas

106. HIV ---even after alchol strlization

108. lower anal canal.....inguinal L.N

109. beta blockers

110. Bioavalability is for oral drugs

111.Hepatic enzyme induction

112. gastroduodenal artery

113Festigeal..

.most medial

114. BAsal ganglia...

.cogintive motor control

115..cavrnous sinus

116.IVC……

L5

116.Epidural space….foramen magnum….SACROCOCCEGEAL MEMBRANRE

117..Hepatic veins….IVC

118. bucopharyngeal memb .

. stomodeum n foegut

119. bitemporal hemianopsia…

.vertical lesion on optic chiasma

120.Vas deferens

121.liquefactive necrosis

122.apoptosis

123.Inguinal hernia in contact…pampiniform plexus

124.Glycocalyx??

125.glutamate?

126. hernited disc compress posterior root oR spinal nerve?

127…Alpha..puillary dilatation
244

128.. han pancreatic disease related to directly . CBD

129..structure not passing through pterygoplatine fossa

130.meso.Ovarium

131. Radiation....Duration of exposure

132.sodium loss..excessive Sweating ..person working in garmi

133. Drugs lipid solubility

134. Pneumothorax

136.Barium follow through

137. Hemosiderin....secondry Iron overload

138.Amyloidosis…………..Light chain

139.Basement membrane

140.vaginal white discharge ,no blodd ..age 30 organism? CANDIDIA ALBICANS

141.asthma…..eosinophils

142.dead space

143.IV fuids

144.Prectrachel facia….thyroid

145.S23…detrusor bladder

146..undecnded testes….

147. actin mysin clathein.. secondary active???pinocytis??

148..urogenital fold..hypospadias

149.epidural space….foramen magnum to S1?

150. parasympathetic...decresae AV nodal delay?

151Hystrectomy..ureter

152.Superficial ring..external oblique qponeurosis.

153.Carotid sinus..blood pressuer

154..macula densa ..rennin?

155..cystic hygroma

156.trineuclotide expansion

157.high ketosteroid in urine…adrenogenital syndrome?

158?tyrosine…catecholamines
245

159.brain fuel…glucose

=-=-=-=-==-=-=-=-==-=-=-=-==-=-=-=-==-=-=-=-==-=-=-=-==-=-=-=-==-=-=-

8. Pathology 4th June 2014 Paper by Umbreen Hashim (80)

Pathology 4th June 2014 Paper


1.Most common type of hyperlipidemia
A.type 1
B.type 2 (IIb is MOST COMMON with incidence of 1/200)
C.type 3
D.type 4
E.type 5
2.Senerio ov multiple myloma ...wt will b the bone marrow finding
A.numerous plasma cells
B.hyper cellular marrow with blasts
3.Wts absent in gall bladder
A.musculris mucosa
B.peri muscular layer.
4.Respiratory alkalosis
Low pco2 low po2
Low pco2 high ph
5 yrs old male bma shows more than 20% blasts acid phosphatase positive sudan black negative
Aml
T lymphocyte leukemia
Histiocytosis
6.Shwartzmann reaction ?
a.antibody mediated
B.cell mediated
C.complement mediated.
Not involve complement
7. Most specific for acute hepatitis
Alt
Ast
Gamma gt
8. H.pylori suspected on endoscopy of patient suffering from gastritis specific diagnosis
A.urea breath test
B.histopathology
(on endoscopy, biopsy of tissue stomach is most accurate)
9. Fungal infection causing extensive endothelial damage
Crypto
Cocoido
Histopl
Mucor (If Sinuses endothelial damage was there in question stem then Mucor is appropriate choice)
10.Itp not responding to steroids
A.megadoses of plt transfusion twice weekly
B.immunoglobulin
246

11.Most common Hospital acquired UTI


A.staph
B.proteus
C.serratia
D.strep
E.e.coli
12.Vibrio cholera
Anaerobe
Grows on alkaline media
13Highest content of protein.
A)Idl.
B)Ldl
C)hdl
)Chylomicrn
14.Most common pyogenic peritonitis
A.bacteroids
B.e.coli
C.spirochets
D.proteus
15.Factory worker presented with motor peripheral neuropathy
Zinc (as zinc comes in category of heavy metals)
Mangnese
Magnesium
16.Bacteria adhere to cell
Techoic acid
Pilli
Capsule
17.Uv light cause
Cataract (uv light mostlyy damage skin leads to skin cancers, skin related disorders thn eyes are more
sensitiv leads to CATARACT, pterygym ang pingucela)
Peripheral neuropathy
Teratogenesis
lung cancer
18.Short limbs .normal trunk . Normal intelligence .
Autosomal dominan
Auto recessive
X linked domint
X linked recessive
A vessel cntaining thick circular layer of muscles
Mediun size artery
Medium size vein
Large vein
Burkit lymphoma oncogenesis by EBV
A. protoncogen c-myc amplification
B. Protoncogen c-myc transcription
247

C. Ras protooncogene
D.bcl2 protooncogene
Staphylococal toxin is
Exotoxin
endotoxin
Enterotoxin
Common tumor after transplant
Hcc
Rcc
Myeloproliferative
A scenerio with occupational history..i.e working ina factory n presentationwith renal impairment.causative
agent
Mercury
Lead
Not a tumour marker
Cea
AFo
P
HCG
Amylase
After Gastrectomy
A.microcytic Anemia
B.megaloblastic
C.pernicious
D.anemia of chronic diseas
Klienfilter syndrm.
Xo
Xxy
Xxx
Trachea ( wrong one)
Stratified squamous epithelim
Contains muscles in post wall
Patient with Arthralgia .utricura
A.typ1 hypersensitvty
B.type 2
C.type 3.
D.type 4
A pt with a lesion on penis no other symptoms most def lab test
Vdrl
Tpa
Dark field microscopy
Fas ab
Antitumor cells express
Mhc 2
248

Cd8 +cytotoxix t cell


Most sensitive tumor to radiation
Brain
Lymph node
Bone
Plague transmited by
A.flea
B.mites
C.tick.
D.mosquito
food poisoning by staph aureus mediated by
A.endotocin
B.enterotoxin
C.leucidin
D.exotoxin
Patiens csf shows v high protiens n v low glucose plus turbidity...
A.meningococci
B.hemophilus
C.pnumococci
D.viral
Cystope and endoscope sterlization at 100 for 20 min
A.tindilization
B.autoclaving
C.pastrurization
68 yrs old lady...complain of bluish spots no other symptoms labs normL
Senile purpura
Vit k def
Liver disease
Least compensated acid base disorder
Normal anion gap
Increased anion gap
Primigravida with complain of headache and blueish spots on skin .SHISTOCYTES on peripheral film
A.pre eclampsia
B.HELLP
C.HUS
D.TTP
E.DIC
Most imp criteria for malignancy on histopathology
A.high nc
B.pleomorphism
C.mitosis
D.invasion
Vaccine is nt available against
A.ebv
Bjapanese encephalitis
C.measles.
249

Most common eitiological agent for tumor in aids


Ebv
Cmv
Hpv
Rhinosinusitis fungus invading endotheliun
A.aspergilloses
B.mucor
C.rhino
d.Histoplas
A boy took chloramphenicol suffered from anemia epistaxis
Aplastic anemia
Itp
Soldier frm Siachen cme bck. B.p 150/90 .. .
Polychythemia vera
2ndry polycthemia.
Spores killed by
Dry heat at 160 temp fr 1hr .
Moist heat at 100 temp fr 20min
Gentoin violet
High content of cholestrol
Chylomicro
Hdl.
Idl
LDL
Hyperpigmentation due to endocrine distrbnce is Not A feature of
A.ACTH THERAPY
B.pregnacy
C.melasma
D.plagra
E.addisons disease
Pigmentatn in uper dermis
Panniculitis ,
Alteratn of bonds
Inc activity of melanocyts
Basal keratinocyts
Most likely for diabetes mellitus
A.increased glucose 6 phosphatase
B. Decreased gluco 6 phosphatase
C.glucose utilization is independent of blood glucose level
D. Increased glycogen synthesis
Hypertensive pt with bp around 210/120...the kidney structure most likely to be damaged
Pct
Macula densa
250

Dct
Juxtaglomrular cells
Carcinogenic free radical injury after radiation takes
A.2 to 5 years
B. 5 to 10 yrs
C.within 2 years
Lymph nodules absent in
Lymph nods
Payer patchs
Tonsils
Thymus
Burkit lymphoma oncogenesis by ebv
A. protoncogen c-myc amplification
B. Protoncogen c-myc transcription
C. Ras protooncogene
D.bcl2 protooncogene
Hypercalcemia in malignancy due to
A. PTh
B.PTH RELATED PEPPTIDE
C.LOCAL INVASION OF BONE MALIGNACY
D.calcitonin
Common tumour after transplant
Hcc
Rcc
Myeloproliferative
Common cause of Addison's
Autoimmune
Tumor
CML can be best differentiated from LEUKEMOID REACTION by
A. Leukocyte alkaline phosphatase
B.sudan b
C. PAS
D.esterase
A boy with more than 100 tubular polyps on colonoscopy n family history of colonic polyps.....cause
FAP
Peutz jehgars
Rapidly induced hypokalemia
Diabetic coma treated with insulin
Ileostomy
Deudenal atresia
Rapidly adopting receptors
A. Free nerv endings
B.pascinian corpusles
C.pain receptors
251

Rta . Multiple frctures 90/50bp .semicomatoseg wat u ll do frst.


Volume replcement.
Reductn and stblize the frcture.
Tracheostomy
Muscle atrophy . N loss of sensation on lateral aspect of limb n loss of muscle stretch reflex
A. Upper moto neuron
B.lower moto neuron
C.lower motor & ant horn
Not a std
..inguinale
Condylomata
Leprosy
Fever malaise macrophages epitheloid cells
Mycobacterium tuberculosis
Mycobctrum tubercle ip
Dosent produce spore.
No capsule.
Non motile
Dificult to stain.
Readily decolorizd wid acid
Not transmitted by fecal oral route
A.trichrius
B.trichnella
E.coli
S.aureus
Scenario splenomegaly without lymphadenopathy TRAP positive
A.aml
B.cll
C.hairy cell leukemia
Fibrocrtilage present in ?
Intervrtbral disc.
Epiglotis
Auricle
epidural space widest at ??
L2 ,
below foramen magnum ,, T7 , T12 , C7
Which cardiac enzyme wud return to normal 80 hrs after MI..I guess this was the right statement
Cpk
Ck mb
Ldh
Dysplasia
Disorganized layering of cells
Increased miosis
252

In which of the following man is the Definitive host ?


A.hookworm
B...hydatid cyst
Nt transmited by sexual contct?
Leprosy.
Herpes simplx 2
Vagnalis inguinale.
Hpv
Undiffrentiated germ cells lining seminiferous tubules
Primary sprmatogonium( spermatogonia A)
Secondary spermatogonium
spermatids
Sertoli cells
35 yaar old man with 20% blast on bma.acid phosphatase positive sudan black negative uska kia tha???
All
Aml
T lymphocytic leukemia
acute undiferentiated leukemia
acute megakaryocytic leukemia
Most common tumor with AIDS
Kaposi sarcoma
Antibodies in serum . . TSH 0.05, T4 increased. T3 normal. Diagnosis?
Graves disease.
Hashimotos thyroidits
Intra nuclear bodies found in pt with pnumonia
CMV
Sudan positive and phosphatase negative
AML
Sudan negative and phosphatase positve
megakaryocytic
Recurrent abortion, hormonal imbalance?
lh
fsh
estrogen
progestrone
Non thrombotic endo carditis ocurs in
SLE
CD 19 postve scnerio based lukemia
ALL
Fluid filled vesicles over backkk
253

VZV
HSV 2
What is used for sterilization?
2% gluteraldehyde (Cold Sterilization)
4% gluter aldehyde
Steam at 100 celcius fr cnsecutve 3 days.. process knwn as
Tyndallization
Pt is Anti lymphocytc globulin, he dvlop pain, arhritc reaction ov site aftr wkk.. type of immunty?
typ1
type 2
type 3
type 4
2 year old boy came wid complain leathargy, poor slow movements, how will diagnosw
TSH with metabolic enzymes
Thyroid profile
IV potassium will cause
metabolic acidosis
aldosterone release
TWO SCNERIOS FROM THYROID
=-=-=-=-==-=-=-=-==-=-=-=-==-=-=-=-==-=-=-=-==-=-=-=-==-=-=-=-==-=-=-

9. Ophthalmology June 2014 by Abdul Sami Abbasi(83)


Fcps part 1, OPHTHALMOLOGY, paper 2, JUNE 2014
Abdul sami abbasi
Ophthalmology past papers
1. Uper lid is not suplied by .
Ans. infraorbital nerve
2. lacrimal gland is derived from
Ans. Surface ectoderm
3. regarding embryology of ciliary muscle
Ans. Derived from neural crest cells
4. Structure passing through optic canal is .
Ans. mandibular nerve
5. the distance between optic disc nd macula is.
Ans. 3mm
6. retinal pigment epithelium is derived from
Ans. Neuroectoderm
7. levator palpabral superioris suplied by
Ans . occulomotor nerve
8 two questions were from oculomotor
nerve . questions not remember
now
9. Histology of corneal epithelium
Ans .stratified squamous non keratinized
10. One question was of horner syndrome ,
11. one scnerio was like . a man presentints with large bones. heart.etc ( asim nd shoaib )
Ans .adenohypophysis
12. structure passing through hyloiad canal
Ans.hyloid artry
13. which is not an aminoglycoside
Ans. Erythromycin
14.question from Timolol
15. Question from pilocarpine
254

16. a patient complains of ptosis mostly at evening, etc


Ans .myasthenia gravis
17.a patient presents with ptosis , he is diagnosed to have myasthenia gravis ,what is used
for its diagnosis ( some what like that )
Ans. edrophonium
18. Which is alpha 1 selective sympathomimic
Ans . Phenylphrine
19. Atropine unlikely
Ans . depress parasympathetic system
20. Atropine less likely
Ans. Activate respiratory centre
21.. A boy feels difficulty in reading book or near work, problem is in
Ans. ciliary muscle contraction.
22. blood supply of fovea is from
Ans. Choriocappilaries
23. local metabolotes of heart is answer of question, its from ( asim nd shoaib )
24. protanopia from ( asim nd shoaib )
25. which is not in hind brain.
Ans. Hypothalamus
26. A patient of myasthenia comes to you ,what drug you shall initially prescribe to this
Ans. neostigmine
27. a patient presents with difficulty in motor speech, lesion is at
Ans. brocas area,
28.maximum reisistance to aqueous flow is through
Ans. trabecular meshwork
29. Corneal transparency is mainted by
Ans. corneal endothelium
30. salt nd water absorption occurs from.
Ans. large intestine
31. collagen fibers in stroma are held together by
Ans. GAG
32. Ratina is thickest at
Ans.optic nerve above optic disk
33. a question related to vastibulcerebellum
34. inf olivary nucleus
35. white light is perceived by
Ans . cones
36. which is steroid hormone
Ans. Thyroid hormone
37. the neurotransmitter secreated by rods and cones .
Ans.glutamate
38. most malignant tumour of eye
Ans . glioma
39. in retinoblastoma what gene is seen…….
Ans . rb gene
40. correct about ciliary body
Ans. 2 epithelium
41. corneal reflex is carried by which nerve
Ans. V1 of trigeminal
42. correct about bowmans membrane
Ans. Present between epithelium and stroma
43. a scnerio on cystic hygroma
44.which is not trinucleotide repeat gene disorder
255

Ans . alzheimers disease


45. a question on geniculocalcrine tract
46.about tears not true is
Ans . 20% is aquous aquous... ( its 30% )
47. a patient presents with history of hypertention ,what ocular sign may be present in the
patient
Ans. arteriovenous nipping
48. superior orbital fissure is present in
Ans. Middle cranial fossa between lesser and greater wing of sphenoid
49. what is most unlikely for visual cortex
Ans. area 19 for pursuit movement
50. corneal reflex infrequent in
Ans . infancy
51. a patient presents in opd with conjunctivitis , he is also suffering from arthritis, he can be
labeled as having
Ans. reiters syndrome
52.lens is thinnest at
Ans . post capsule
53. in snellins chart each component part of letter is separated by a distance of
Ans. 1 min
54. A man after reading a book looks away , what effect will be at ciliary muscle
( Question exactly was not alike that , but concept means this )
Ans .relaxation
55. ANP mechanism of action .
Ans. Cyclic gmp
56. The best method to diagnose a tumour is
Ans.histopathology
57.a patient with TB is on isoniazid , what should be added in treatment …….
Ans . pyridoxine
58. which are animal protein
Ans . vit c vit E .
59. a question from biostatics. ( stratified sampling )
60. a tumour will be graded as fetal when( somewhat LIKE THAT )
Ans It is highly metastaic
61. hiv secreation not found in .
Ans . tears
62. in HIV what does not occur
Ans . hard Ecudate
vasculitis .
cotton wool
63. fast adapting touch receptors are
Ans . pacinian
64. heating is controlled by
Ans. Post hypothalamus
65. most coman old age carcinoma of upper lid
Ans. Basal or squamous ?
66. HLA b27 .
Ans . diabeties malitus
67.calcitonin is secreated from
Ans . thyroid gland
68. diagnostic tumer test is
Ans . biopsy ??
69. chemotaxis defination
256

70. pre tectal area is located


Ans. at level of superior colliculus
71. Choroid fissures faliure to close lead to cleft formation nd coloboma cleft
ANS. Ciliary body
72. In albinism vessels are seen is
Ans . iris
73. coma shaped floater is seen in
Ans. Diabetic retinopathy
74. multi drugs in TB is used because of
Ans. Emergence of resistance
75. retinal detachment mostly occurs at the level of
Ans. RPE ( retinal pigment epithelium )
76. homonymus heminopia occurs in
Ans lesion of optic tract
77. Structures passing through cavernous sinus.
Ans internal carotid artry and abducent nerve
78. pupilary dilation is the function of
Ans. Alpha 1 receptors
79. The inappropriate is .
Ans . rods have low sensitivity
80. About sclera incorrect is
Ans. thinnest at fovea ?
81. scnerio on cenral retinal vein occlusion
82 . regarding visious ring
Ans. Ring formed after trauma
83. A 35 yr old boxer came to you with normal vision supraduction is normal but in primary
position of eye , Right eye tends to adduct . which nerve is damaged.?
Ans.Super devision of oculomotor nerve
=-=-=-=-==-=-=-=-==-=-= The End =-==-=-=-=-==-=-=-=-==-=-=-
=-=-=-=-=-=-=-=-=-=-=-=-=-=-=-=-=-=-=-=-=-=-=-=-=-=-=-=-=-=-=-=-=-=-=-=-=-=-=-=-=-=-=-=-=-=-=-=-=-=-=-=-=-=-=-=-=-

Compiled by : Amlodipine Besylate


=-=-=-=-=-=-=-=-=-=-=-=-=-=-=-=-=-=-=-=-=-=-=-=-=-=-=-=-=-=-=-=-=-=-=-=-=-=-=-=-=-=-=-=-=-=-=-=-=-=-=-=-=-=-=-=-=

You might also like